{"id": 190, "year": 2013, "question_id_specific": 94, "full_question": "An 18-month-old boy, with complete immunization schedule to date, who consults the Emergency Department for right knee swelling after playing in the park, without obvious trauma. In the directed anamnesis, the mother refers that an uncle of hers had similar problems. The ultrasound examination is compatible with hemarthrosis and in the analytical analysis only an APTT lengthening of 52'' (normal 25-35'') stands out. What is the most probable diagnostic hypothesis?", "full_answer": "Although other hemorrhagic diseases can have a prolonged thromboplastin time, due to the intensity of the lesion and the child's sex and family history, the most likely diagnosis is hemophilia.", "type": "PEDIATRICS", "options": {"1": "Marfan syndrome.", "2": "Von-Willebrand's disease.", "3": "Ehlers-Danlos disease.", "4": "Hemophilia A.", "5": "Bernard-Soulier disease."}, "correct_option": 4, "explanations": {"1": {"exist": false, "char_ranges": [], "word_ranges": [], "text": ""}, "2": {"exist": false, "char_ranges": [], "word_ranges": [], "text": ""}, "3": {"exist": false, "char_ranges": [], "word_ranges": [], "text": ""}, "4": {"exist": true, "char_ranges": [[0, 193]], "word_ranges": [[0, 30]], "text": "Although other hemorrhagic diseases can have a prolonged thromboplastin time, due to the intensity of the lesion and the child's sex and family history, the most likely diagnosis is hemophilia."}, "5": {"exist": false, "char_ranges": [], "word_ranges": [], "text": ""}}} {"id": 150, "year": 2012, "question_id_specific": 201, "full_question": "We are consulted by an 84-year-old woman for insomnia of conciliation. After failing sleep hygiene measures, it is decided to initiate pharmacological treatment. Which of the following drugs would you select for the patient?", "full_answer": "Knowing that we have taken measures of sleep hygiene, we have explored what provokes him not to sleep, and he does not sleep nine hours and still wants to sleep more,...then the less harmful answer is the 2. Because it is the one with the shortest half-life and he does not want the lady to fall down the next day.", "type": "PSYCHIATRY", "options": {"1": "Diacepam.", "2": "Lormetacepam.", "3": "Phenobarbital.", "4": "Chlordiazepoxide.", "5": "Chloracepate."}, "correct_option": 2, "explanations": {"1": {"exist": false, "char_ranges": [], "word_ranges": [], "text": ""}, "2": {"exist": true, "char_ranges": [[174, 314]], "word_ranges": [[31, 60]], "text": "the less harmful answer is the 2. Because it is the one with the shortest half-life and he does not want the lady to fall down the next day."}, "3": {"exist": false, "char_ranges": [], "word_ranges": [], "text": ""}, "4": {"exist": false, "char_ranges": [], "word_ranges": [], "text": ""}, "5": {"exist": false, "char_ranges": [], "word_ranges": [], "text": ""}}} {"id": 454, "year": 2018, "question_id_specific": 153, "full_question": "A 65-year-old woman consults for weakness in the right hand that has spread in a few months to other muscle territories of both arms and legs, with distal predominance. On examination there is atrophy and fasciculations in different metameric territories with preserved sensitivity. There is a bilateral Babinski's sign, what is the diagnostic test that would confirm the suspected diagnosis?", "full_answer": "The answer is 2, an EMG, which would be the diagnostic test to confirm the diagnostic suspicion of ALS, the imaging tests both CT, but especially MRI would help us to rule out, but would not confirm the diagnosis.", "type": "NEUROLOGY", "options": {"1": "Cerebral CT.", "2": "Electromyographic study.", "3": "Cerebral MRI.", "4": "Multimodal evoked potentials.", "5": NaN}, "correct_option": 2, "explanations": {"1": {"exist": true, "char_ranges": [[104, 213]], "word_ranges": [[19, 39]], "text": "the imaging tests both CT, but especially MRI would help us to rule out, but would not confirm the diagnosis."}, "2": {"exist": true, "char_ranges": [[0, 103]], "word_ranges": [[0, 19]], "text": "The answer is 2, an EMG, which would be the diagnostic test to confirm the diagnostic suspicion of ALS,"}, "3": {"exist": true, "char_ranges": [[104, 213]], "word_ranges": [[19, 39]], "text": "the imaging tests both CT, but especially MRI would help us to rule out, but would not confirm the diagnosis."}, "4": {"exist": true, "char_ranges": [[104, 213]], "word_ranges": [[19, 39]], "text": "the imaging tests both CT, but especially MRI would help us to rule out, but would not confirm the diagnosis."}, "5": {"exist": false, "char_ranges": [], "word_ranges": [], "text": ""}}} {"id": 541, "year": 2021, "question_id_specific": 5, "full_question": "25-year-old patient, who suffers a motorcycle accident on a Friday night. He is taken to the emergency room and diagnosed with abdominal trauma (negative echo-fast), mild head trauma (Glasgow = 14) and a displaced intracapsular fracture of the right hip. Hemodynamically she is stable, what would be the treatment of choice?", "full_answer": "The management of displaced intracapsular hip fracture in a young patient is surgical and should be done early to reduce the risk of avascular necrosis of the hip. They are not talking about life threatening enough to postpone a necessary surgical procedure or to consider an external fixator following a damage containment policy so we discard option 3. This is a 25 year old patient so we have to pursue reduction and fixation avoiding replacement options (we discard option 2 and 4).", "type": "TRAUMATOLOGY", "options": {"1": "Reduction, open if necessary, and osteosynthesis of the fracture in the first 24-36 hours.", "2": "Total hip resurfacing arthroplasty on Monday on a scheduled basis.", "3": "Wait for improvement of the cranial trauma and schedule the following week a scheduled surgery consisting of reduction and osteosynthesis of the fracture.", "4": "Given the risk of non-union of these types of fracture, I would perform an emergency bipolar hip hemiarthroplasty.", "5": NaN}, "correct_option": 1, "explanations": {"1": {"exist": true, "char_ranges": [[0, 163]], "word_ranges": [[0, 28]], "text": "The management of displaced intracapsular hip fracture in a young patient is surgical and should be done early to reduce the risk of avascular necrosis of the hip."}, "2": {"exist": true, "char_ranges": [[355, 486]], "word_ranges": [[58, 82]], "text": "This is a 25 year old patient so we have to pursue reduction and fixation avoiding replacement options (we discard option 2 and 4)."}, "3": {"exist": true, "char_ranges": [[85, 354]], "word_ranges": [[12, 58]], "text": "surgical and should be done early to reduce the risk of avascular necrosis of the hip. They are not talking about life threatening enough to postpone a necessary surgical procedure or to consider an external fixator following a damage containment policy so we discard option 3."}, "4": {"exist": true, "char_ranges": [[355, 486]], "word_ranges": [[58, 82]], "text": "This is a 25 year old patient so we have to pursue reduction and fixation avoiding replacement options (we discard option 2 and 4)."}, "5": {"exist": false, "char_ranges": [], "word_ranges": [], "text": ""}}} {"id": 602, "year": 2022, "question_id_specific": 112, "full_question": "A 61-year-old woman, administrative, with a history of overweight, hypertension, dyslipidemia and metabolic syndrome, who consults for pain in both buttocks, left trochanteric region, lateral aspect of the left thigh up to the knee and left leg up to the middle third. The pain appears when the lower limb is lifted with the knee extended, but is relieved when the knee is flexed. What is the first clinical suspicion?", "full_answer": "In this case, when speaking of \"reappearance of the symptoms when we lift the lower limb with the knee extended\" we are describing the Lasègue Maneuver. This maneuver is positive in cases of lumbosciatic pain, since it causes a stretching of the sciatic nerve, therefore the correct answer is number 3. The rest of the pathologies do not have a positive Lasègue maneuver, so these options would not be correct.", "type": "TRAUMATOLOGY", "options": {"1": "Gouty arthritis of left hip.", "2": "Left coxofemoral arthrosis.", "3": "Radiated low back pain / lumbosciatica.", "4": "Claudication due to canal stenosis.", "5": NaN}, "correct_option": 3, "explanations": {"1": {"exist": true, "char_ranges": [[303, 410]], "word_ranges": [[51, 70]], "text": "The rest of the pathologies do not have a positive Lasègue maneuver, so these options would not be correct."}, "2": {"exist": true, "char_ranges": [[303, 410]], "word_ranges": [[51, 70]], "text": "The rest of the pathologies do not have a positive Lasègue maneuver, so these options would not be correct."}, "3": {"exist": true, "char_ranges": [[0, 302]], "word_ranges": [[0, 51]], "text": "In this case, when speaking of \"reappearance of the symptoms when we lift the lower limb with the knee extended\" we are describing the Lasègue Maneuver. This maneuver is positive in cases of lumbosciatic pain, since it causes a stretching of the sciatic nerve, therefore the correct answer is number 3."}, "4": {"exist": true, "char_ranges": [[303, 410]], "word_ranges": [[51, 70]], "text": "The rest of the pathologies do not have a positive Lasègue maneuver, so these options would not be correct."}, "5": {"exist": false, "char_ranges": [], "word_ranges": [], "text": ""}}} {"id": 127, "year": 2012, "question_id_specific": 77, "full_question": "A 32-year-old diabetic patient on insulin therapy with good control of his blood glucose levels comes to your office with tingling in both hands, with a sensation of corking and thermal insensitivity of progressive onset over the course of 2 weeks. She does not report visual disturbances, strength deficit, motor clumsiness or other symptoms. On examination she found anesthesia to pain and temperature in both hands and distal forearms; positional and vibratory sensitivity were preserved. There is no muscle atrophy or strength deficit. Muscle reflexes are normal and symmetrical. There is no dysmetria, dysdiadochokinesia or intention tremor. The rest of the neurological examination is strictly normal. Indicate the most likely diagnosis in this case:", "full_answer": "This year's Neurology section's burning question. He presents a diabetic patient on insulin treatment with good control (although diabetic neuropathies do not only develop in patients with poor control) with a sensory picture limited to the upper limbs and of short onset time, without motor symptoms or at any other level. In the examination there is a clear dissociation with thermoalgesic anesthesia and preservation of arthrokinetic and vibratory. Reflexes are normal, neither abolished nor exalted. In addition, the rest of the examination is strictly normal. With all this I believe that the correct answer is 5, that is a syringomyelic lesion, whose initial characteristic is the sensitive dissociation with anesthesia for the thermoalgesic and conservation of the posterior cord. MS and other spinal cord lesions would be accompanied by other exploratory signs such as exaltation of reflexes, for carpal tunnel the exploration exceeds the territory distal to the carpus, and as for answer 1, although there are forms of peripheral polyneuropathy \"pseudosyringomyelic\" they are usually of lumbar onset. The short evolution time (2 weeks), the respect of the lower limbs and the preservation of muscle reflexes make me rule out this response despite the distribution being \"glove-like\".", "type": "NEUROLOGY AND NEUROSURGERY", "options": {"1": "Peripheral sensitvo symmetric distal peripheral neuropathy of diabetic cause.", "2": "Compressive cervical spinal cord injury.", "3": "Bilateral carpal tunnel syndrome.", "4": "Multiple sclerosis type demyelinating disease.", "5": "Central cervical spinal cord injury."}, "correct_option": 5, "explanations": {"1": {"exist": true, "char_ranges": [[1000, 1109]], "word_ranges": [[154, 168]], "text": "although there are forms of peripheral polyneuropathy \"pseudosyringomyelic\" they are usually of lumbar onset."}, "2": {"exist": true, "char_ranges": [[788, 900]], "word_ranges": [[119, 137]], "text": "MS and other spinal cord lesions would be accompanied by other exploratory signs such as exaltation of reflexes,"}, "3": {"exist": true, "char_ranges": [[901, 978]], "word_ranges": [[137, 149]], "text": "for carpal tunnel the exploration exceeds the territory distal to the carpus,"}, "4": {"exist": true, "char_ranges": [[788, 900]], "word_ranges": [[119, 137]], "text": "MS and other spinal cord lesions would be accompanied by other exploratory signs such as exaltation of reflexes,"}, "5": {"exist": true, "char_ranges": [[579, 787]], "word_ranges": [[88, 119]], "text": "I believe that the correct answer is 5, that is a syringomyelic lesion, whose initial characteristic is the sensitive dissociation with anesthesia for the thermoalgesic and conservation of the posterior cord."}}} {"id": 316, "year": 2016, "question_id_specific": 138, "full_question": "A 37-year-old man presents with arthritis of the metacarpophalangeal joints of both hands and right pleuritis. Bilateral malar erythema is seen on examination. Positive antinuclear antibodies were detected (titer 1/640), with anti native DNA antibodies also positive; anti-Sm negative What would be the initial treatment of choice for this patient?", "full_answer": "It is SLE with joint, skin and serositis involvement. Treatment is EC at moderate-high doses. Mycophenolate is never a treatment in acute phase but in maintenance, nor has it proved useful in joint involvement.", "type": "RHEUMATOLOGY", "options": {"1": "Glucocorticoids at high doses.", "2": "Glucocorticoids and mycophenolate.", "3": "Nonsteroidal anti-inflammatory drugs and antimalarials.", "4": "The picture will probably be self-limited and does not require treatment.", "5": NaN}, "correct_option": 1, "explanations": {"1": {"exist": true, "char_ranges": [[0, 93]], "word_ranges": [[0, 15]], "text": "It is SLE with joint, skin and serositis involvement. Treatment is EC at moderate-high doses."}, "2": {"exist": true, "char_ranges": [[94, 210]], "word_ranges": [[15, 34]], "text": "Mycophenolate is never a treatment in acute phase but in maintenance, nor has it proved useful in joint involvement."}, "3": {"exist": false, "char_ranges": [], "word_ranges": [], "text": ""}, "4": {"exist": false, "char_ranges": [], "word_ranges": [], "text": ""}, "5": {"exist": false, "char_ranges": [], "word_ranges": [], "text": ""}}} {"id": 153, "year": 2012, "question_id_specific": 74, "full_question": "A 75-year-old woman consults for violaceous lesions on the hands and neck together with progressive muscle weakness of 3 months of evolution. What diagnostic tests, among those indicated, can be useful for the diagnosis?", "full_answer": "Given this clinical picture, one possibility is dermatomyositis. Of the proposed tests, the determination of serum aldolase may be useful. Its elevation is characteristic of this disease. The biopsy should be muscular and not subcutaneous. Anti-smooth muscle antibodies are not characteristic of this disease.", "type": "RHEUMATOLOGY", "options": {"1": "Determination of serum aldolase.", "2": "Electroencephalogram.", "3": "Biopsy of subcutaneous cellular tissue.", "4": "Determination of anti-smooth muscle antibodies.", "5": "Genetic study of their descendants."}, "correct_option": 1, "explanations": {"1": {"exist": true, "char_ranges": [[0, 187]], "word_ranges": [[0, 27]], "text": "Given this clinical picture, one possibility is dermatomyositis. Of the proposed tests, the determination of serum aldolase may be useful. Its elevation is characteristic of this disease."}, "2": {"exist": false, "char_ranges": [], "word_ranges": [], "text": ""}, "3": {"exist": true, "char_ranges": [[188, 239]], "word_ranges": [[27, 35]], "text": "The biopsy should be muscular and not subcutaneous."}, "4": {"exist": true, "char_ranges": [[240, 309]], "word_ranges": [[35, 44]], "text": "Anti-smooth muscle antibodies are not characteristic of this disease."}, "5": {"exist": false, "char_ranges": [], "word_ranges": [], "text": ""}}} {"id": 369, "year": 2016, "question_id_specific": 122, "full_question": "A 67-year-old man, ex-smoker, with a diagnosis of severe COPD (multidimensional index BODE 5, FEVl 38%, body mass index 23, dyspnea index according to the mMRC 3 scale, distance covered in the 6-minute walk test 260 m) who has had 3 hospital admissions for exacerbation of his COPD in the last 7 months, comes to the consultation. She also has a history of hypertension, ischemic heart disease with AMI 5 years ago and intermittent claudication. In the clinical examination, there is a decrease in vesicular murmur with expiratory wheezing in both lung fields and an oximetry saturation of 88%. Which of the following therapeutic strategies would NOT be recommended for this patient?", "full_answer": "Systemic corticosteroids in these patients should be used primarily in acute exacerbations, not for control of exacerbations.", "type": "PNEUMOLOGY AND THORACIC SURGERY", "options": {"1": "Adjustment of inhaled therapy with long-acting bronchodilators combining anticholinergics and beta-2 adrenergics with inhaled glucocorticoids.", "2": "Start oral glucocorticoids for 6 months to control exacerbations.", "3": "Check that the patient performs the inhalation technique correctly.", "4": "Initiate chronic home oxygen therapy regimen.", "5": NaN}, "correct_option": 2, "explanations": {"1": {"exist": false, "char_ranges": [], "word_ranges": [], "text": ""}, "2": {"exist": true, "char_ranges": [[0, 125]], "word_ranges": [[0, 17]], "text": "Systemic corticosteroids in these patients should be used primarily in acute exacerbations, not for control of exacerbations."}, "3": {"exist": false, "char_ranges": [], "word_ranges": [], "text": ""}, "4": {"exist": false, "char_ranges": [], "word_ranges": [], "text": ""}, "5": {"exist": false, "char_ranges": [], "word_ranges": [], "text": ""}}} {"id": 413, "year": 2018, "question_id_specific": 68, "full_question": "An 87-year-old woman with a history of hypertension was admitted 48 hours ago to the coronary unit for acute myocardial infarction with ST-segment elevation of anterior location. She reported dyspnea. Examination revealed a systolic murmur with fremitus, radiating to the right sternal border, which was not present on admission. What complication do you suspect?", "full_answer": "Rupture of the interventricular septum. Mechanical complications are typical in infarctions in elderly women. Likewise, a murmur that was not present indicates abnormal blood flow: the most frequent complications are mitral insufficiency, which would present with a musical or piante murmur, and VSD, which presents with a murmur radiating to the right border (think of the direction of blood flow).", "type": "CARDIOLOGY AND CARDIOVASCULAR SURGERY", "options": {"1": "Heart failure due to extensive necrosis.", "2": "Anterior aneurysm.", "3": "Left ventricular free wall rupture.", "4": "Rupture in the interventricular septum.", "5": NaN}, "correct_option": 4, "explanations": {"1": {"exist": false, "char_ranges": [], "word_ranges": [], "text": ""}, "2": {"exist": false, "char_ranges": [], "word_ranges": [], "text": ""}, "3": {"exist": false, "char_ranges": [], "word_ranges": [], "text": ""}, "4": {"exist": true, "char_ranges": [[181, 399]], "word_ranges": [[25, 60]], "text": "the most frequent complications are mitral insufficiency, which would present with a musical or piante murmur, and VSD, which presents with a murmur radiating to the right border (think of the direction of blood flow)."}, "5": {"exist": false, "char_ranges": [], "word_ranges": [], "text": ""}}} {"id": 314, "year": 2016, "question_id_specific": 178, "full_question": "A 66-year-old patient diagnosed with stage IV pancreatic adenocarcinoma 8 months ago. He follows treatment with delayed release morphine 200 mg/12 hours orally, kerosene and lactulose. For the last 15 days he has reported paresthesias and occasional lancinating pain in the right lumbar and periumbilical area that does not subside with rescue morphine. The neurological examination is normal. Indicate the most appropriate management:", "full_answer": "We have before us a patient with neuropathic pain, which due to the origin of the tumor, is most likely due to involvement of the celiac plexus. It is important to have a radiological diagnosis in view of a possible local treatment, such as neurolytic treatment or plexus blockade. However, we should not leave the patient without a treatment that can alleviate but rather solve the current situation. He is taking WHO step 3 analgesia, which is clearly insufficient, so he will need adjuvants; although opioids for neuropathic pain are not very effective, we will not reduce the dose but will associate the adjuvants. We would manage the patient with gabapentin and corticosteroids (true answer 4); we would not decrease the dose of morphine (false answer 3), and in the meantime we would ask for a CT scan (or abdominal MRI) to subsequently assess the plexus block (answers 1 and 2 for future management).", "type": "ONCOLOGY", "options": {"1": "Perform computed axial tomography and evaluate nerve compression since it is neuropathic pain.", "2": "Evaluate neurolytic treatment since neuropathic pain is difficult to control.", "3": "Decrease the dose of morphine as it is ineffective in this type of pain.", "4": "Administer amitriptyline or gabapentin, dexamethasone and increase the dose of morphine.", "5": NaN}, "correct_option": 4, "explanations": {"1": {"exist": true, "char_ranges": [[765, 907]], "word_ranges": [[128, 154]], "text": "in the meantime we would ask for a CT scan (or abdominal MRI) to subsequently assess the plexus block (answers 1 and 2 for future management)."}, "2": {"exist": true, "char_ranges": [[765, 907]], "word_ranges": [[128, 154]], "text": "in the meantime we would ask for a CT scan (or abdominal MRI) to subsequently assess the plexus block (answers 1 and 2 for future management)."}, "3": {"exist": true, "char_ranges": [[700, 760]], "word_ranges": [[116, 127]], "text": "we would not decrease the dose of morphine (false answer 3),"}, "4": {"exist": true, "char_ranges": [[495, 699]], "word_ranges": [[84, 116]], "text": "although opioids for neuropathic pain are not very effective, we will not reduce the dose but will associate the adjuvants. We would manage the patient with gabapentin and corticosteroids (true answer 4);"}, "5": {"exist": false, "char_ranges": [], "word_ranges": [], "text": ""}}} {"id": 263, "year": 2014, "question_id_specific": 37, "full_question": "Juan is 60 years old, has been smoking 2 packs/day for years and has reported a persistent cough for the last 6 months. He notes that his left eyelid is more droopy and that the pupil of this eye is smaller. John reports that the medial side of his left hand is numb and with less strength. His physician checks for palpebral ptosis and left miosis; he checks that he can close both eyelids symmetrically and that both pupils respond correctly to light. In addition, he checks that there is no sweating from the left hemiface, that he feels less prickling on the inner surface of the left hand and that he has less strength in the grip of the left hand. Regarding the ocular symptomatology, where is the lesion located?", "full_answer": "The question would be more of neurology. It is a claude bernard horner syndrome and probably the correct one is 1. But for safety's sake it is better to have it checked by a neurologist.", "type": "PNEUMOLOGY", "options": {"1": "Sympathetic fibers, at some level that would span from the hypothalamus to the interinedio-lateral Clark's column of the dorsal medulla.", "2": "Left common ocular motor nerve in the midbrain.", "3": "Edinger-Westphal nucleus above the left common ocular motor nerve nucleus.", "4": "Parasympathetic fibers, at some level ranging from the Edinger-Westphal nucleus to the constrictor muscle of the left pupil.", "5": "Tarsal muscle exclusively."}, "correct_option": 1, "explanations": {"1": {"exist": true, "char_ranges": [[41, 114]], "word_ranges": [[7, 21]], "text": "It is a claude bernard horner syndrome and probably the correct one is 1."}, "2": {"exist": false, "char_ranges": [], "word_ranges": [], "text": ""}, "3": {"exist": false, "char_ranges": [], "word_ranges": [], "text": ""}, "4": {"exist": false, "char_ranges": [], "word_ranges": [], "text": ""}, "5": {"exist": false, "char_ranges": [], "word_ranges": [], "text": ""}}} {"id": 344, "year": 2016, "question_id_specific": 158, "full_question": "A 24-year-old woman, primigestation, suffers a spontaneous abortion at 7 weeks gestation. The anatomopathological study of the abortive remains indicates molar disease. We should inform you that:", "full_answer": "She should not become pregnant until she has undergone regular check-ups and one year has passed with negative beta-HCG levels. Patients will be monitored weekly with hCG dosing until it becomes undetectable, for three consecutive times. After that the monitoring will be monthly for six months and then every two months for another six months before a new pregnancy.", "type": "GYNECOLOGY AND OBSTETRICS", "options": {"1": "The risk of a new molar gestation in a future pregnancy is 50%.", "2": "She should not become pregnant until periodic check-ups and after one year with negative BHCG levels.", "3": "Subsequent controls are not necessary if the evacuation of the trophoblastic tissue was complete.", "4": "Periodic check-ups are necessary since 40% of cases will develop gestational trophoblastic neoplasia.", "5": NaN}, "correct_option": 2, "explanations": {"1": {"exist": false, "char_ranges": [], "word_ranges": [], "text": ""}, "2": {"exist": true, "char_ranges": [[0, 367]], "word_ranges": [[0, 59]], "text": "She should not become pregnant until she has undergone regular check-ups and one year has passed with negative beta-HCG levels. Patients will be monitored weekly with hCG dosing until it becomes undetectable, for three consecutive times. After that the monitoring will be monthly for six months and then every two months for another six months before a new pregnancy."}, "3": {"exist": false, "char_ranges": [], "word_ranges": [], "text": ""}, "4": {"exist": false, "char_ranges": [], "word_ranges": [], "text": ""}, "5": {"exist": false, "char_ranges": [], "word_ranges": [], "text": ""}}} {"id": 43, "year": 2011, "question_id_specific": 151, "full_question": "A 10-year-old girl comes for a health check-up. Physical examination is normal with weight and height in the 50th percentile and BP 109/65. A routine urinalysis shows a specific gravity of 1035 pH6 blood 2+ with no protein. Urine sediment shows 5-10 red blood cells per field. What would be the most appropriate course of action?", "full_answer": "The correct answer is 4. What is not very clear to me is \"routine urinalysis\" because it is not usually done in an asymptomatic healthy child. Microhematuria without associated symptoms may be a finding without pathologic significance and must be confirmed in a subsequent new sediment (although I do not know if at 15 days or later).", "type": "PEDIATRICS", "options": {"1": "Determination of creatinine and nitrogen in blood.", "2": "Refer the child for cystoscopy.", "3": "Determine antinuclear antibodies and complement.", "4": "Repeat urine sediment in 15 days.", "5": "Abdominal computed axial tomography."}, "correct_option": 4, "explanations": {"1": {"exist": false, "char_ranges": [], "word_ranges": [], "text": ""}, "2": {"exist": false, "char_ranges": [], "word_ranges": [], "text": ""}, "3": {"exist": false, "char_ranges": [], "word_ranges": [], "text": ""}, "4": {"exist": true, "char_ranges": [[143, 334]], "word_ranges": [[26, 57]], "text": "Microhematuria without associated symptoms may be a finding without pathologic significance and must be confirmed in a subsequent new sediment (although I do not know if at 15 days or later)."}, "5": {"exist": false, "char_ranges": [], "word_ranges": [], "text": ""}}} {"id": 616, "year": 2022, "question_id_specific": 119, "full_question": "A 13-year-old female, with no relevant history, with menarche 3 months ago, followed since the age of 10 years by idiopathic scoliosis that has worsened. In the physical examination she presents a hump of 7 degrees in the Adams test and in the scoliogram a thoracolumbar curve T4-L1 of 35 degrees of Cobb and a Risser 0. The correct attitude to take will be:", "full_answer": "Orthosis. Cobb angle between 25º - 45º. Immature skeleton (Risser 0).", "type": "TRAUMATOLOGY", "options": {"1": "Recommend swimming and revision in three months.", "2": "Prescribe a corset-type orthosis.", "3": "Refer to physiotherapy for spine elastification.", "4": "Review in 6 months with a new X-ray.", "5": NaN}, "correct_option": 2, "explanations": {"1": {"exist": false, "char_ranges": [], "word_ranges": [], "text": ""}, "2": {"exist": true, "char_ranges": [[0, 69]], "word_ranges": [[0, 11]], "text": "Orthosis. Cobb angle between 25º - 45º. Immature skeleton (Risser 0)."}, "3": {"exist": false, "char_ranges": [], "word_ranges": [], "text": ""}, "4": {"exist": false, "char_ranges": [], "word_ranges": [], "text": ""}, "5": {"exist": false, "char_ranges": [], "word_ranges": [], "text": ""}}} {"id": 489, "year": 2020, "question_id_specific": 105, "full_question": "A 67-year-old patient who in the last 6 months, in two routine analyses, presents progressive lymphocytosis. In the last one, hemoglobin 15.4 g/dL; leukocytes 18.5 x103/μL with 82 % of mature lymphocytes that by flow cytometry express CD5/CD19/CD23 antigens and platelets 240 x103/μL. What do you think is the correct approach?", "full_answer": "The patient is already diagnosed with chronic lymphocytic leukemia B (B-CLLL), presents lymphocytosis that is maintained for more than 3 months and by flow cytometry presents a phenotype compatible with it. Therefore, we must first know the risk factors that will indicate with what priority treatment will need to be initiated, their intensity and prognosis; the presence of TP53 mutations or 17p deletions depends on this. The bone marrow aspirate or biopsy is only indicated if there are other accompanying cytopenias that are not justified by the B-CLL itself, PET-CT is only indicated if Ritcher's syndrome is suspected and we do not have enough data to suspect it, there are no B symptoms, nor is there any mention of LDH elevation.", "type": "HEMATOLOGY", "options": {"1": "Study of TP53 mutations to establish prognosis.", "2": "Bone aspirate/biopsy to confirm diagnosis.", "3": "PET/CT to establish the therapeutic attitude.", "4": "New clinical and analytical control in 6 months.", "5": NaN}, "correct_option": 4, "explanations": {"1": {"exist": false, "char_ranges": [], "word_ranges": [], "text": ""}, "2": {"exist": true, "char_ranges": [[425, 564]], "word_ranges": [[66, 89]], "text": "The bone marrow aspirate or biopsy is only indicated if there are other accompanying cytopenias that are not justified by the B-CLL itself,"}, "3": {"exist": true, "char_ranges": [[565, 738]], "word_ranges": [[89, 121]], "text": "PET-CT is only indicated if Ritcher's syndrome is suspected and we do not have enough data to suspect it, there are no B symptoms, nor is there any mention of LDH elevation."}, "4": {"exist": true, "char_ranges": [[0, 424]], "word_ranges": [[0, 66]], "text": "The patient is already diagnosed with chronic lymphocytic leukemia B (B-CLLL), presents lymphocytosis that is maintained for more than 3 months and by flow cytometry presents a phenotype compatible with it. Therefore, we must first know the risk factors that will indicate with what priority treatment will need to be initiated, their intensity and prognosis; the presence of TP53 mutations or 17p deletions depends on this."}, "5": {"exist": false, "char_ranges": [], "word_ranges": [], "text": ""}}} {"id": 534, "year": 2021, "question_id_specific": 119, "full_question": "A 45-year-old female patient with a history of gastroesophageal reflux has been presenting for the past year with episodes of pallor in some fingers with exposure to cold. She had recently been prescribed prednisone at a dose of 20 mg/day for joint pain and skin induration in the hands and arms. For the last 48 hours, she presented with general malaise and intense headache, for which she went to the emergency department. Examination revealed only a rhythmic tachycardia at 100 bpm, with no neurological focality. Blood pressure was 200/110 mmHg. The blood test shows only a creatinine level of 2.5 mg/dL. Indicate which of the following autoantibodies is best related to the process described:", "full_answer": "This is a clear clinical case of systemic sclerosis (Raynaud's, esophageal involvement, skin induration) presenting with renal crisis. Apart from anti-proteinase 3 antibodies, which are not related to scleroderma (incorrect answer 3), the most likely ones would be antiRNA polymerase III antibodies, which appear in younger patients with renal crises.", "type": "RHEUMATOLOGY", "options": {"1": "Anti-RNA polymerase III antibodies.", "2": "Anti-centromere antibodies.", "3": "Anti-proteinase 3 antibodies.", "4": "Anti-PM-Scl antibodies.", "5": NaN}, "correct_option": 1, "explanations": {"1": {"exist": true, "char_ranges": [[0, 351]], "word_ranges": [[0, 50]], "text": "This is a clear clinical case of systemic sclerosis (Raynaud's, esophageal involvement, skin induration) presenting with renal crisis. Apart from anti-proteinase 3 antibodies, which are not related to scleroderma (incorrect answer 3), the most likely ones would be antiRNA polymerase III antibodies, which appear in younger patients with renal crises."}, "2": {"exist": false, "char_ranges": [], "word_ranges": [], "text": ""}, "3": {"exist": true, "char_ranges": [[146, 234]], "word_ranges": [[20, 32]], "text": "anti-proteinase 3 antibodies, which are not related to scleroderma (incorrect answer 3),"}, "4": {"exist": false, "char_ranges": [], "word_ranges": [], "text": ""}, "5": {"exist": false, "char_ranges": [], "word_ranges": [], "text": ""}}} {"id": 273, "year": 2016, "question_id_specific": 71, "full_question": "18-year-old young man with a history of asthma, allergy to pollens, mites and cat hair, comes to the emergency room referring sensation of food detention at retrosternal level with practical inability to swallow his own saliva. He refers similar episodes on other occasions that have subsided spontaneously within a few minutes. Which of the following is the most likely diagnosis?", "full_answer": "This is intermittent dysphagia. Barrett's esophagus does not necessarily imply peptic stricture, but assuming it does, it is progressive. Infectious esophagitis is more typical of immunocompromised patients. That leaves distal esophageal ring and eosinophilic esophagitis; both are possible, but the insistence on the patient's atopic burden indicates the likelihood of the latter.", "type": "DIGESTIVE SYSTEM", "options": {"1": "Barrett's esophagus.", "2": "Distal esophageal ring (Schatzki).", "3": "Infectious esophagitis.", "4": "Eosinophilic esophagitis.", "5": NaN}, "correct_option": 4, "explanations": {"1": {"exist": true, "char_ranges": [[32, 137]], "word_ranges": [[4, 19]], "text": "Barrett's esophagus does not necessarily imply peptic stricture, but assuming it does, it is progressive."}, "2": {"exist": true, "char_ranges": [[208, 381]], "word_ranges": [[27, 52]], "text": "That leaves distal esophageal ring and eosinophilic esophagitis; both are possible, but the insistence on the patient's atopic burden indicates the likelihood of the latter."}, "3": {"exist": true, "char_ranges": [[138, 207]], "word_ranges": [[19, 27]], "text": "Infectious esophagitis is more typical of immunocompromised patients."}, "4": {"exist": true, "char_ranges": [[208, 381]], "word_ranges": [[27, 52]], "text": "That leaves distal esophageal ring and eosinophilic esophagitis; both are possible, but the insistence on the patient's atopic burden indicates the likelihood of the latter."}, "5": {"exist": false, "char_ranges": [], "word_ranges": [], "text": ""}}} {"id": 56, "year": 2011, "question_id_specific": 76, "full_question": "14-year-old girl who consults for decreased growth for 2-3 years previously normal (provides data) and that other girls her age have greater physical and sexual development. Lately she has had headaches and visual problems that she notices in class and when studying. She has not had menarche or polydipsia or polyuria. Parents with normal height. Examination: short stature at -2.1 standard deviations, normal body proportions, little pubic hair and breast development. Campimetry shows left temporal partial hemianopsia. Bone age: delay of 2 years. General laboratory tests were normal. Gonadotrophins (FSH and LH) and estradiol are low. What do you think is the most appropriate response?", "full_answer": "Answer 2 is false (Turner syndrome: low estradiol and elevated gonadotrophins), brain tumors affecting the hypothalamus-pituitary gland do not give low gonadotrophins (5 false), and it seems obvious that refractive defects do not give hemianopsia (3 and 4 false).", "type": "ENDOCRINOLOGY", "options": {"1": "Decreased growth and sexual development, delayed bone age, headache and visual alteration suggest hormonal deficit and involvement of the optic chiasm.", "2": "As she is a girl of pubertal age, it is most likely that her decreased growth and sexual retardation are due to Turner syndrome.", "3": "She must not have a hypothalamic tumor because of the absence of polyuria and polydipsia. She probably has constitutional delay and her visual problem is refractive.", "4": "A growth hormone deficiency may explain the developmental delay and low estradiol. To evaluate if she needs glasses, due to her headaches and visual disturbances.", "5": "She could have a craniopharyngioma, but it would be rare if she had not shown symptoms before. Also, it would not justify low gonadotrophins and estradiol."}, "correct_option": 1, "explanations": {"1": {"exist": false, "char_ranges": [], "word_ranges": [], "text": ""}, "2": {"exist": true, "char_ranges": [[0, 79]], "word_ranges": [[0, 11]], "text": "Answer 2 is false (Turner syndrome: low estradiol and elevated gonadotrophins),"}, "3": {"exist": true, "char_ranges": [[204, 263]], "word_ranges": [[29, 39]], "text": "refractive defects do not give hemianopsia (3 and 4 false)."}, "4": {"exist": true, "char_ranges": [[204, 263]], "word_ranges": [[29, 39]], "text": "refractive defects do not give hemianopsia (3 and 4 false)."}, "5": {"exist": true, "char_ranges": [[80, 177]], "word_ranges": [[11, 24]], "text": "brain tumors affecting the hypothalamus-pituitary gland do not give low gonadotrophins (5 false),"}}} {"id": 409, "year": 2018, "question_id_specific": 191, "full_question": "The patient comes to the home of an oncology patient whose pain has not been well controlled lately. On physical examination there appears to be no evidence of tumor progression, and no previously known data of interest is revealed. In the anamnesis, the main caregiver states that the patient has pain 8 hours after receiving the prescribed basal dose of morphine every 12 hours. This clinical situation is referred to as:", "full_answer": "The correct answer is 1. The statement perfectly defines a tolerance phenomenon in which the patient has become \"used to\" the dose and needs more in order to have the same analgesic effect.", "type": "PRIMARY CARE", "options": {"1": "Tolerance", "2": "Hyperalgesia", "3": "Dependency", "4": "Ineffectiveness", "5": NaN}, "correct_option": 1, "explanations": {"1": {"exist": true, "char_ranges": [[25, 189]], "word_ranges": [[5, 33]], "text": "The statement perfectly defines a tolerance phenomenon in which the patient has become \"used to\" the dose and needs more in order to have the same analgesic effect."}, "2": {"exist": false, "char_ranges": [], "word_ranges": [], "text": ""}, "3": {"exist": false, "char_ranges": [], "word_ranges": [], "text": ""}, "4": {"exist": false, "char_ranges": [], "word_ranges": [], "text": ""}, "5": {"exist": false, "char_ranges": [], "word_ranges": [], "text": ""}}} {"id": 179, "year": 2013, "question_id_specific": 98, "full_question": "A 33-year-old woman consults for repeated epistaxis, petechiae and ecchymosis. Laboratory tests show thrombocytopenia with a platelet count of 4000 platelets/microliter. The initial presumptive diagnosis is chronic immune thrombocytopenic purpura (ITP). Which of the following statements is FALSE regarding the diagnosis of ITP?", "full_answer": "In the presence of isolated thrombocytopenia we must not lose sight of the physical examination; thus, the presence of lymphadenopathy forces us to rule out the presence of lymphoma, for example. 1 is correct. When performing a bone marrow biopsy-aspirate it is seen that the number of megakaryocytes is normal or even increased, the failure is not in the marrow but in the peripheral blood, where platelets are destroyed. So. The false answer is 2.", "type": "HEMATOLOGY", "options": {"1": "The presence of lymphadenopathy or splenomegaly in the physical examination suggests a different diagnosis of ITP.", "2": "Bone marrow analysis shows a decreased number of megakaryocytes without other alterations.", "3": "Complete blood count shows isolated thrombocytopenia with often large platelets, without anemia unless there is significant bleeding or associated autoimmune hemolysis (Evans syndrome).", "4": "The diagnosis of ITP is established by exclusion of other processes causing thrombocytopenia.", "5": "The determination of antiplatelet antibodies is not accurate to establish the diagnosis."}, "correct_option": 2, "explanations": {"1": {"exist": true, "char_ranges": [[0, 195]], "word_ranges": [[0, 31]], "text": "In the presence of isolated thrombocytopenia we must not lose sight of the physical examination; thus, the presence of lymphadenopathy forces us to rule out the presence of lymphoma, for example."}, "2": {"exist": true, "char_ranges": [[210, 422]], "word_ranges": [[34, 69]], "text": "When performing a bone marrow biopsy-aspirate it is seen that the number of megakaryocytes is normal or even increased, the failure is not in the marrow but in the peripheral blood, where platelets are destroyed."}, "3": {"exist": false, "char_ranges": [], "word_ranges": [], "text": ""}, "4": {"exist": false, "char_ranges": [], "word_ranges": [], "text": ""}, "5": {"exist": false, "char_ranges": [], "word_ranges": [], "text": ""}}} {"id": 353, "year": 2016, "question_id_specific": 161, "full_question": "A 27-year-old woman referred to the gynecology office for evaluation referring dyspareunia for about 8 months, along with dyschezia and occasional rectorrhagia coinciding with menstruation for 3-4 months. She also reports dysmenorrhea for years, which she controls well with ibuprofen. She has been trying to get pregnant for 16 months without success. In the gynecological examination she only has pain when pressing on the posterior vaginal fornix. Which test do you consider would allow you to reach a diagnosis of certainty of her pathology?", "full_answer": "Diagnostic laparoscopy. It is a typical case of deep endometriosis, with implants in different parts of the pelvis. The definitive diagnosis will be given by the pathological anatomy, so for this we have to do Laparoscopy and send to PA.", "type": "GYNECOLOGY AND OBSTETRICS", "options": {"1": "Transvaginal ultrasound.", "2": "Diagnostic laparoscopy.", "3": "Magnetic resonance imaging.", "4": "Colonoscopy.", "5": NaN}, "correct_option": 2, "explanations": {"1": {"exist": false, "char_ranges": [], "word_ranges": [], "text": ""}, "2": {"exist": true, "char_ranges": [[24, 237]], "word_ranges": [[2, 40]], "text": "It is a typical case of deep endometriosis, with implants in different parts of the pelvis. The definitive diagnosis will be given by the pathological anatomy, so for this we have to do Laparoscopy and send to PA."}, "3": {"exist": false, "char_ranges": [], "word_ranges": [], "text": ""}, "4": {"exist": false, "char_ranges": [], "word_ranges": [], "text": ""}, "5": {"exist": false, "char_ranges": [], "word_ranges": [], "text": ""}}} {"id": 438, "year": 2018, "question_id_specific": 119, "full_question": "A 38-year-old woman of veterinary profession, in charge of monitoring wild animals and assisting in the delivery of domestic livestock. She starts with a high fever with chills, headache, myalgia and non-productive cough that she interprets as a flu-like process. She presented with chest pain. Chest X-ray showed bilateral pulmonary infiltrates in lower fields. A serologic test was performed with elevated titers of antibodies against phase II antigens. Which of the following statements is TRUE?", "full_answer": "As a possible disease I would consider Brucellosis, typical of cattle (milk, abortion, genital secretions...). It presents with a pseudo-flu-like symptomatology (headache, arthralgias, myalgias, fever...) and other symptoms depending on the route of entry. In our case, when attending animal births, the entry is through the respiratory route, so the pulmonary infiltrates and chest pain are due to the bacteria. The treatment varies according to the affectation and mortality is very low, almost null. Morbidity and involvement in pregnant women are of greater concern. Important: it is very frequent in livestock workers (veterinarians, farmers, slaughterhouses...) if you read something like \"attends births of livestock animals\", \"works on a farm\", \"is a veterinarian\" suspect Brucellosis.", "type": "EPIDEMIOLOGY AND PREVENTIVE MEDICINE", "options": {"1": "This entity is transmitted by ticks.", "2": "Both doxycycline and hydroxychloroquine are effective in treating acute forms of this disease.", "3": "In the acute form, the patient also generally has elevated antibodies to phase I antigens.", "4": "Mortality in acute forms is almost nonexistent.", "5": NaN}, "correct_option": 4, "explanations": {"1": {"exist": false, "char_ranges": [], "word_ranges": [], "text": ""}, "2": {"exist": false, "char_ranges": [], "word_ranges": [], "text": ""}, "3": {"exist": false, "char_ranges": [], "word_ranges": [], "text": ""}, "4": {"exist": true, "char_ranges": [[257, 502]], "word_ranges": [[34, 74]], "text": "In our case, when attending animal births, the entry is through the respiratory route, so the pulmonary infiltrates and chest pain are due to the bacteria. The treatment varies according to the affectation and mortality is very low, almost null."}, "5": {"exist": false, "char_ranges": [], "word_ranges": [], "text": ""}}} {"id": 579, "year": 2022, "question_id_specific": 108, "full_question": "A 49-year-old man is admitted to the ICU for traumatic brain injury after an accident at work. In the physical examination he does not open his eyes, does not emit sounds before being intubated and presents extension of extremities to nociceptive stimulus. An intracranial pressure sensor is placed and a decompressive craniotomy must be performed due to intraparenchymal hemorrhage. Which of the following statements is correct?", "full_answer": "Hypercapnia and acidosis produce vasodilatation causing increased cerebral blood flow. According to the described examination the Glasgow is 4 (O1V1M2). Cerebral perfusion pressure is mean arterial pressure minus intracranial pressure. Cerebral edema in which cellular edema is produced by the introduction of extracellular fluid into the intracellular compartment is cytotoxic.", "type": "NEUROLOGY", "options": {"1": "On arrival she is in a Glasgow scale coma of 7.", "2": "Cerebral perfusion pressure is mean arterial pressure plus intracranial pressure.", "3": "Vasogenic cerebral edema is due to cellular edema, membrane rupture and cell death.", "4": "Cerebral vascular flow increases with hypercapnia and acidosis.", "5": NaN}, "correct_option": 4, "explanations": {"1": {"exist": true, "char_ranges": [[87, 152]], "word_ranges": [[10, 20]], "text": "According to the described examination the Glasgow is 4 (O1V1M2)."}, "2": {"exist": true, "char_ranges": [[153, 235]], "word_ranges": [[20, 30]], "text": "Cerebral perfusion pressure is mean arterial pressure minus intracranial pressure."}, "3": {"exist": true, "char_ranges": [[236, 378]], "word_ranges": [[30, 50]], "text": "Cerebral edema in which cellular edema is produced by the introduction of extracellular fluid into the intracellular compartment is cytotoxic."}, "4": {"exist": true, "char_ranges": [[0, 86]], "word_ranges": [[0, 10]], "text": "Hypercapnia and acidosis produce vasodilatation causing increased cerebral blood flow."}, "5": {"exist": false, "char_ranges": [], "word_ranges": [], "text": ""}}} {"id": 212, "year": 2014, "question_id_specific": 88, "full_question": "A 45-year-old man undergoes a truncal vagotomy and antrectomy with Billroth II reconstruction for chronic peptic ulcer disease with pyloro-duodenal stricture. Six weeks after the surgery she reports that shortly after (less than half an hour) after ingestions she presents nausea, asthenia and sweating, dizziness and abdominal cramps usually accompanied by diarrhea. Which of the following is the most appropriate approach for her initial management?", "full_answer": "Answers 1, 2 and 5 are appropriate treatments for dumping syndrome or postgastrectomy, but the question is focused on initial management, so the most appropriate answer seems to be 2.", "type": "GENERAL SURGERY", "options": {"1": "Apply treatment with a somatostatin inhibitor (octreotide).", "2": "Follow specific dietary measures.", "3": "Trial treatment with a benzodiazepine.", "4": "Search for a probable neuroendocrine tumor (e.g. carcinoid).", "5": "Indicate surgical treatment to perform an antiperistaltic Roux-en-Y gastrojejunostomy."}, "correct_option": 2, "explanations": {"1": {"exist": true, "char_ranges": [[0, 183]], "word_ranges": [[0, 30]], "text": "Answers 1, 2 and 5 are appropriate treatments for dumping syndrome or postgastrectomy, but the question is focused on initial management, so the most appropriate answer seems to be 2."}, "2": {"exist": true, "char_ranges": [[0, 183]], "word_ranges": [[0, 30]], "text": "Answers 1, 2 and 5 are appropriate treatments for dumping syndrome or postgastrectomy, but the question is focused on initial management, so the most appropriate answer seems to be 2."}, "3": {"exist": false, "char_ranges": [], "word_ranges": [], "text": ""}, "4": {"exist": false, "char_ranges": [], "word_ranges": [], "text": ""}, "5": {"exist": true, "char_ranges": [[0, 183]], "word_ranges": [[0, 30]], "text": "Answers 1, 2 and 5 are appropriate treatments for dumping syndrome or postgastrectomy, but the question is focused on initial management, so the most appropriate answer seems to be 2."}}} {"id": 507, "year": 2020, "question_id_specific": 89, "full_question": "A 46-year-old man with bipolar disorder is brought to the emergency department after an over-ingestion of lithium carbonate. Examination reveals severe tremor, ataxia, dysarthria, myoclonus and fasciculations. Lithemia is 4.1 mEq/L (toxicity > 1.6 mEq/L). Which of the following therapeutic options would be most indicated?", "full_answer": "Be careful with lithium. Intoxications have been seen with normal lithium. It is not rare -especially due to interactions-. A very effective drug if everyone is forewarned.", "type": "PSYCHIATRY", "options": {"1": "Aminophylline associated with a cathartic.", "2": "Activated charcoal.", "3": "Hemodialysis.", "4": "Forced diuresis.", "5": NaN}, "correct_option": 3, "explanations": {"1": {"exist": false, "char_ranges": [], "word_ranges": [], "text": ""}, "2": {"exist": false, "char_ranges": [], "word_ranges": [], "text": ""}, "3": {"exist": false, "char_ranges": [], "word_ranges": [], "text": ""}, "4": {"exist": false, "char_ranges": [], "word_ranges": [], "text": ""}, "5": {"exist": false, "char_ranges": [], "word_ranges": [], "text": ""}}} {"id": 552, "year": 2022, "question_id_specific": 89, "full_question": "A 4-year-old girl presenting with a high fever of 6 days' evolution. On clinical examination she presents an erythematous maculopapular rash on the trunk and genital area, with a tendency to confluence, without becoming scarlatiniform; conjunctival injection without secretions and red lips with raspberry tongue. She also presents erythema with edema in hands and feet and a unilateral cervical adenopathy of 2 cm in diameter. The most likely clinical diagnosis of suspicion is:", "full_answer": "There is no doubt, they are not meeting the diagnostic criteria of Kawasaki disease (fever of several days of evolution, exanthema, conjunctival injection, red lips, raspberry tongue, edema of acral parts and adenopathy).", "type": "DERMATOLOGY", "options": {"1": "Kawasaki disease.", "2": "Measles.", "3": "Rubella.", "4": "Scarlet fever.", "5": NaN}, "correct_option": 1, "explanations": {"1": {"exist": true, "char_ranges": [[0, 221]], "word_ranges": [[0, 33]], "text": "There is no doubt, they are not meeting the diagnostic criteria of Kawasaki disease (fever of several days of evolution, exanthema, conjunctival injection, red lips, raspberry tongue, edema of acral parts and adenopathy)."}, "2": {"exist": false, "char_ranges": [], "word_ranges": [], "text": ""}, "3": {"exist": false, "char_ranges": [], "word_ranges": [], "text": ""}, "4": {"exist": false, "char_ranges": [], "word_ranges": [], "text": ""}, "5": {"exist": false, "char_ranges": [], "word_ranges": [], "text": ""}}} {"id": 598, "year": 2022, "question_id_specific": 181, "full_question": "A 67-year-old man presents with 3 months of asthenia and febrile fever, with nasal obstruction and mucus emission with some clots in the last month. In the last few days she noticed pain in the right eye and asymmetry with respect to the contralateral eye. Physical examination reveals proptosis of the right eyeball and inspection of the nostrils reveals an erythematous mucosa with serohematic crusts. The rest of the examination was normal. Blood tests (hemogram, renal and hepatic function) are normal, except for an ESR of 65 mm/h; urinalysis shows microhematuria and proteinuria of 520 mg/24h. What is the most probable initial diagnosis?", "full_answer": "In this patient, he presents with a granulomatosis with polyangiitis formerly known as Wegener's granulomatosis. In the clinical case we are told about the otorhinolaryngologic involvement that is present in 92% of patients. At the pulmonary level, we were told that mucus with a clot was present in 85% of the patients. Also at the end of the case, they comment us the urinalysis with a glomerulonephritis that is present in 77% of the patients.", "type": "RHEUMATOLOGY", "options": {"1": "Eosinophilic granulomatosis with polyangiitis.", "2": "Microscopic polyangiitis.", "3": "Granulomatosis with polyangiitis.", "4": "Polyarteritis nodosa.", "5": NaN}, "correct_option": 3, "explanations": {"1": {"exist": false, "char_ranges": [], "word_ranges": [], "text": ""}, "2": {"exist": false, "char_ranges": [], "word_ranges": [], "text": ""}, "3": {"exist": true, "char_ranges": [[0, 446]], "word_ranges": [[0, 75]], "text": "In this patient, he presents with a granulomatosis with polyangiitis formerly known as Wegener's granulomatosis. In the clinical case we are told about the otorhinolaryngologic involvement that is present in 92% of patients. At the pulmonary level, we were told that mucus with a clot was present in 85% of the patients. Also at the end of the case, they comment us the urinalysis with a glomerulonephritis that is present in 77% of the patients."}, "4": {"exist": false, "char_ranges": [], "word_ranges": [], "text": ""}, "5": {"exist": false, "char_ranges": [], "word_ranges": [], "text": ""}}} {"id": 133, "year": 2012, "question_id_specific": 168, "full_question": "75-year-old woman diagnosed 3 years ago with soft drusen in the fundus. She reports presenting, since 2 weeks ago, metamorphopsia and significant visual loss in her right eye that prevents her from reading. Indicate the most probable diagnosis:", "full_answer": "Of the five options, four could be compatible with the referred symptoms. We could initially rule out central serous chorioretinopathy because by definition it occurs in young people (up to 55 years of age, more or less). The key to differentiate between the other four is that he had soft drusen in the fundus. That while not a \"diagnosis\" (the question could have been better worded) is a finding related to age-related macular degeneration, or age-related macular degeneration. The dry or atrophic form presents with slowly progressive visual loss. But the wet or exudative form produces acute or rapidly progressive severe visual loss (usually due to subretinal hemorrhage accompanied by metamorphopsia (image distortion due to retinal lifting). An epiretinal membrane usually has a slower course. In a macular hole, visual loss is more relevant (metamorphopsia may also be reported, but it is less likely in the acute stage). In thrombosis of the central retinal vein, visual loss is more important and does not usually explain metamorphopsia (but may have it, due to macular edema).", "type": "OPHTHALMOLOGY", "options": {"1": "Macular epiretinal membrane.", "2": "Macular hole.", "3": "Thrombosis of the central retinal vein.", "4": "Senile macular degeneration.", "5": "Central serous chorioretinopathy."}, "correct_option": 4, "explanations": {"1": {"exist": true, "char_ranges": [[750, 801]], "word_ranges": [[117, 125]], "text": "An epiretinal membrane usually has a slower course."}, "2": {"exist": true, "char_ranges": [[802, 930]], "word_ranges": [[125, 148]], "text": "In a macular hole, visual loss is more relevant (metamorphopsia may also be reported, but it is less likely in the acute stage)."}, "3": {"exist": true, "char_ranges": [[931, 1088]], "word_ranges": [[148, 174]], "text": "In thrombosis of the central retinal vein, visual loss is more important and does not usually explain metamorphopsia (but may have it, due to macular edema)."}, "4": {"exist": true, "char_ranges": [[222, 480]], "word_ranges": [[37, 78]], "text": "The key to differentiate between the other four is that he had soft drusen in the fundus. That while not a \"diagnosis\" (the question could have been better worded) is a finding related to age-related macular degeneration, or age-related macular degeneration."}, "5": {"exist": true, "char_ranges": [[74, 221]], "word_ranges": [[12, 37]], "text": "We could initially rule out central serous chorioretinopathy because by definition it occurs in young people (up to 55 years of age, more or less)."}}} {"id": 186, "year": 2013, "question_id_specific": 67, "full_question": "A 56-year-old female patient consulted for dorso-lumbar spine pain and progressive difficulty in performing usual tasks. In the last 5 years she gained weight, she has ecchymosis easily and arterial hypertension was detected. Physical examination: Obesity of central predominance, rounded facies, increased supraclavicular fat, decreased proximal muscle strength and some reddish striae in the abdomen. She has a blood glucose of 136 mg/dL and the radiological study showed osteoporosis and vertebral crushing. What do you think is the most coherent interpretation and attitude?", "full_answer": "This question is a bit convoluted for the diagnosis of Cushing's syndrome but easy if the steps are clear. We all agree that the patient seems to have Cushing's syndrome (we rule out 1). For the diagnosis we have to measure urinary free cortisol, do a dexamethasone suppression test or measure nocturnal salivary cortisol. If hypercortisolism is confirmed, the origin should be sought; the ACTH measurement indicates the origin: suppressed in hypercortisolism of adrenal origin or prolonged use of corticoids and elevated or normal if the origin is pituitary or by ectopic ACTH secretion. (Therefore, 4 and 5 are false). If ACTH is low, a CT scan of the abdomen should be performed to locate the origin. If ACTH is normal or high, a pituitary MRI should be performed (pituitary adenomas responsible for Cushing's syndrome are very small and CT of the pituitary is less sensitive than MRI). It is very important to perform the order well to arrive at a proper diagnosis: 1st: confirm hypercortisolism or s. Cushing's syndrome, 2nd: measure ACTH to orient etiology. 3rd: imaging test according to ACTH levels. Therefore, answer 2 is false (abdominal CT is not a test to rule out s. Cushing's); an imaging test should only be done once a clear diagnosis of hypercortisolism has been made. Therefore, the true one is 3.", "type": "ENDOCRINOLOGY", "options": {"1": "Postmenopausal osteoporosis, type 2 diabetes mellitus and essential hypertension, with decreased strength due to diabetic polyneuropathy.", "2": "It is necessary to rule out Cushing's disease by dexamethasone suppression test and perform a cranial CT scan.", "3": "Suggest Cushing's. Determine urinary free cortisol and basal ACTH, which serves to orient its etiology and select the most appropriate imaging technique.", "4": "It looks like Cushing's. If basal ACTH is high, it may be due to corticosteroid use or an adrenal tumor, and an MRI should be performed.", "5": "Probably has Cushing's. If the basal ACTH is low, he probably has a pituitary micro-adenoma, and a cranial CT scan should be performed."}, "correct_option": 3, "explanations": {"1": {"exist": true, "char_ranges": [[107, 186]], "word_ranges": [[19, 34]], "text": "We all agree that the patient seems to have Cushing's syndrome (we rule out 1)."}, "2": {"exist": true, "char_ranges": [[1119, 1191]], "word_ranges": [[184, 199]], "text": "answer 2 is false (abdominal CT is not a test to rule out s. Cushing's);"}, "3": {"exist": true, "char_ranges": [[1192, 1285]], "word_ranges": [[199, 215]], "text": "an imaging test should only be done once a clear diagnosis of hypercortisolism has been made."}, "4": {"exist": true, "char_ranges": [[323, 620]], "word_ranges": [[54, 100]], "text": "If hypercortisolism is confirmed, the origin should be sought; the ACTH measurement indicates the origin: suppressed in hypercortisolism of adrenal origin or prolonged use of corticoids and elevated or normal if the origin is pituitary or by ectopic ACTH secretion. (Therefore, 4 and 5 are false)."}, "5": {"exist": true, "char_ranges": [[323, 620]], "word_ranges": [[54, 100]], "text": "If hypercortisolism is confirmed, the origin should be sought; the ACTH measurement indicates the origin: suppressed in hypercortisolism of adrenal origin or prolonged use of corticoids and elevated or normal if the origin is pituitary or by ectopic ACTH secretion. (Therefore, 4 and 5 are false)."}}} {"id": 7, "year": 2011, "question_id_specific": 208, "full_question": "During a colonoscopy, a 5-cm tumor located in the right colon is detected in a 48-year-old man. No other lesions were found. His maternal grandmother also suffered from colon cancer. The biopsies are superficial and show a poorly differentiated tumor with abundant inflammatory cells in the stroma that is diagnosed as a medullary type carcinoma.", "full_answer": "Complicated question about an unusual adenocarcinoma. By rule 1 would be easy to eliminate, the treatment of colon cancer is surgical (with neoadjuvant in rectal cancer). If biopsies confirm adenocarcinoma, there is no need to repeat them, since it is necessary to have the complete specimen. Most colon cancer develops on adenomas, so we could eliminate this one. Among the other 2, it is important to know that 5 is correct since they almost always present microsatellite instability even without fulfilling the Amsterdam criteria and have a better prognosis than other poorly differentiated colon cancer.", "type": "DIGESTIVE", "options": {"1": "Chemotherapy is the treatment of choice.", "2": "Since the biopsy is superficial, it should be repeated before proceeding with treatment.", "3": "The prognosis of the tumor depends mainly on its high degree of anaplasia.", "4": "It is unlikely that this tumor has developed over a previous adenoma.", "5": "Microsatellite instability and DNA error repair genes should be studied."}, "correct_option": 5, "explanations": {"1": {"exist": true, "char_ranges": [[54, 169]], "word_ranges": [[6, 26]], "text": "By rule 1 would be easy to eliminate, the treatment of colon cancer is surgical (with neoadjuvant in rectal cancer)."}, "2": {"exist": true, "char_ranges": [[171, 291]], "word_ranges": [[26, 46]], "text": "If biopsies confirm adenocarcinoma, there is no need to repeat them, since it is necessary to have the complete specimen."}, "3": {"exist": true, "char_ranges": [[293, 364]], "word_ranges": [[46, 58]], "text": "Most colon cancer develops on adenomas, so we could eliminate this one."}, "4": {"exist": true, "char_ranges": [[365, 607]], "word_ranges": [[58, 95]], "text": "Among the other 2, it is important to know that 5 is correct since they almost always present microsatellite instability even without fulfilling the Amsterdam criteria and have a better prognosis than other poorly differentiated colon cancer."}, "5": {"exist": true, "char_ranges": [[365, 607]], "word_ranges": [[58, 95]], "text": "Among the other 2, it is important to know that 5 is correct since they almost always present microsatellite instability even without fulfilling the Amsterdam criteria and have a better prognosis than other poorly differentiated colon cancer."}}} {"id": 152, "year": 2012, "question_id_specific": 72, "full_question": "A hyperuricemic patient who usually takes 100 mg of allopurinol daily comes to the ED with acute pain and inflammatory signs in the right knee. Arthrocentesis is performed and polarized light microscopy shows intracellular crystals with negative birefringence. Which of the following therapeutic approaches is the most appropriate in this case?", "full_answer": "In the presence of acute gouty arthritis (the presence of intracellular crystals with negative birefringence confirms this) in a hyperuricemic patient previously treated with allopurinol, an NSAID should be added until the crisis subsides. No changes should be made in the hypouricemic treatment during this period because the uric acid metabolic chain is altered and the situation may worsen.", "type": "RHEUMATOLOGY", "options": {"1": "Discontinue allopurinol and start colchicine treatment.", "2": "Discontinue allopurinol and start NSAIDs.", "3": "Add an NSAID until the crisis remits.", "4": "Increase the dose of allopurinol to 300 mg/day.", "5": "Substitute allopurinol for uricosuric acid."}, "correct_option": 3, "explanations": {"1": {"exist": true, "char_ranges": [[240, 393]], "word_ranges": [[34, 59]], "text": "No changes should be made in the hypouricemic treatment during this period because the uric acid metabolic chain is altered and the situation may worsen."}, "2": {"exist": true, "char_ranges": [[240, 393]], "word_ranges": [[34, 59]], "text": "No changes should be made in the hypouricemic treatment during this period because the uric acid metabolic chain is altered and the situation may worsen."}, "3": {"exist": true, "char_ranges": [[0, 239]], "word_ranges": [[0, 34]], "text": "In the presence of acute gouty arthritis (the presence of intracellular crystals with negative birefringence confirms this) in a hyperuricemic patient previously treated with allopurinol, an NSAID should be added until the crisis subsides."}, "4": {"exist": true, "char_ranges": [[240, 393]], "word_ranges": [[34, 59]], "text": "No changes should be made in the hypouricemic treatment during this period because the uric acid metabolic chain is altered and the situation may worsen."}, "5": {"exist": true, "char_ranges": [[240, 393]], "word_ranges": [[34, 59]], "text": "No changes should be made in the hypouricemic treatment during this period because the uric acid metabolic chain is altered and the situation may worsen."}}} {"id": 243, "year": 2014, "question_id_specific": 112, "full_question": "A 20-year-old girl comes to the clinic with an acute fever, cervical lymphadenopathy and skin rash. According to the patient, 3 weeks ago she had a sexual relationship that could be a risk for contracting the HIV virus. Indicate which of the following answers is true:", "full_answer": "A simple question if we are clear about the diagnostic process of HIV. At first glance, we could directly rule out answer 2 since we know that mononucleosis can be an expression of the first phase of HIV infection, especially when we have a history of a risk contact. Answers 3 and 4 are also easily ruled out; ELISA is a very sensitive but not very specific technique and confirmation with a Western-blot test is ALWAYS necessary. Answers 1 and 5 contradict each other if you notice, so one of the two is correct. In this case, the false answer is 1, it is a very categorical answer (\"rule out\") that does not quite fit the reality; As we have said, ELISA is a very sensitive technique, so a negative result could leave us quite calm, however, we all know that there is no medical test that rules out anything with total certainty (unless its sensitivity is 100%, which is not the case) and, on the other hand, we could say that in this case we have a high clinical suspicion (the picture is suggestive, and the antecedents are there). In addition, it is only 3 weeks since the contact, so our patient is probably in the \"window period\" during which serological techniques may be unprofitable. In conclusion, the answer that best fits us would be 5, in the window period a viral load may be indicated for diagnosis (positive if there are more than 10,000 copies), although this test is not done routinely.", "type": "INFECTIOUS DISEASES", "options": {"1": "A negative HIV-1/HIV-2 serology performed by ELISA technique rules out the possibility that the patient has been infected with the HIV virus.", "2": "The clinical process that the patient suffers from is not consistent with acute HIV infection.", "3": "If the patient's ELISA test was positive, nothing further would be necessary for the diagnosis of HIV infection.", "4": "The ELISA technique has a high sensitivity for the diagnosis of HIV infection, but its specificity is even higher.", "5": "If the ELISA test to diagnose HIV in the patient was negative, we could determine by PCR technique the viral load in blood."}, "correct_option": 5, "explanations": {"1": {"exist": true, "char_ranges": [[651, 887]], "word_ranges": [[121, 163]], "text": "ELISA is a very sensitive technique, so a negative result could leave us quite calm, however, we all know that there is no medical test that rules out anything with total certainty (unless its sensitivity is 100%, which is not the case)"}, "2": {"exist": true, "char_ranges": [[88, 267]], "word_ranges": [[16, 49]], "text": "we could directly rule out answer 2 since we know that mononucleosis can be an expression of the first phase of HIV infection, especially when we have a history of a risk contact."}, "3": {"exist": true, "char_ranges": [[311, 431]], "word_ranges": [[58, 77]], "text": "ELISA is a very sensitive but not very specific technique and confirmation with a Western-blot test is ALWAYS necessary."}, "4": {"exist": true, "char_ranges": [[311, 431]], "word_ranges": [[58, 77]], "text": "ELISA is a very sensitive but not very specific technique and confirmation with a Western-blot test is ALWAYS necessary."}, "5": {"exist": true, "char_ranges": [[1210, 1406]], "word_ranges": [[218, 254]], "text": "the answer that best fits us would be 5, in the window period a viral load may be indicated for diagnosis (positive if there are more than 10,000 copies), although this test is not done routinely."}}} {"id": 458, "year": 2018, "question_id_specific": 126, "full_question": "A 47-year-old man, with a history of a right parotid pleomorphic adenoma, treated with surgery (extrafacial parotidectomy) 6 months ago, who comes to our office for presenting pain with sweating and reddening of the skin in the preauricular region during mastication. What treatment would be the treatment of choice?", "full_answer": "You are asking about auriculotemporal syndrome or Frey's syndrome. It is the clinical expression of a vegetative neuropathy caused by mechanical or irritative lesion of the vegetative fibers of the auriculotemporal nerve in its infratemporal pathway. In our environment the most frequent cause is surgery of the split gland with a percentage of occurrence between 10 and 80%. Conservative treatment with botulinum toxin offers good results.", "type": "OTOLARYNGOLOGY AND MAXILLOFACIAL SURGERY", "options": {"1": "Extended total parotidectomy on suspicion of tumor recurrence.", "2": "Pregabalin.", "3": "Intradermal botulinum toxin injection.", "4": "Broad-spectrum antibiotherapy.", "5": NaN}, "correct_option": 3, "explanations": {"1": {"exist": false, "char_ranges": [], "word_ranges": [], "text": ""}, "2": {"exist": false, "char_ranges": [], "word_ranges": [], "text": ""}, "3": {"exist": true, "char_ranges": [[251, 440]], "word_ranges": [[36, 66]], "text": "In our environment the most frequent cause is surgery of the split gland with a percentage of occurrence between 10 and 80%. Conservative treatment with botulinum toxin offers good results."}, "4": {"exist": false, "char_ranges": [], "word_ranges": [], "text": ""}, "5": {"exist": false, "char_ranges": [], "word_ranges": [], "text": ""}}} {"id": 123, "year": 2012, "question_id_specific": 223, "full_question": "Pregnant woman, 27 years old, 30 weeks of gestation. She comes to the emergency room because she noticed pain in the left lumbar region and dysuria since yesterday. She has no febrile sensation. She refers repeated urinary tract infections (UTI). Urinalysis shows Hb 3+, leukocytes 3+, nitrites 2+, sediment: 15-20 leukocytes per field and 5-10 red blood cells per field. Which of the following microorganisms is the most frequent culprit in pregnant women?", "full_answer": "The correct answer is the first one. The most frequent cause of urinary tract infections is Escherichia coli and in the case of pregnant women as well.", "type": "MICROBIOLOGY", "options": {"1": "Escherichia coli.", "2": "Enterococcus faecalis.", "3": "Streptococcus agalactiae.", "4": "Proteus mirabilis.", "5": "Satphylococcus saprophyticus."}, "correct_option": 1, "explanations": {"1": {"exist": true, "char_ranges": [[37, 151]], "word_ranges": [[7, 27]], "text": "The most frequent cause of urinary tract infections is Escherichia coli and in the case of pregnant women as well."}, "2": {"exist": false, "char_ranges": [], "word_ranges": [], "text": ""}, "3": {"exist": false, "char_ranges": [], "word_ranges": [], "text": ""}, "4": {"exist": false, "char_ranges": [], "word_ranges": [], "text": ""}, "5": {"exist": false, "char_ranges": [], "word_ranges": [], "text": ""}}} {"id": 522, "year": 2021, "question_id_specific": 165, "full_question": "A 66-year-old woman diagnosed with type 2 diabetes mellitus since three months ago. She has a BMI of 31 kg/m2 and presents poor glycemic control despite a program of non-pharmacological measures (healthy diet, exercise). Which of the following hypoglycemic drugs is associated with weight gain and should we avoid in this patient?:", "full_answer": "A somewhat more complicated question than the previous one. Regarding weight, metformin to a lesser extent, SGLT-2 inhibitors and, above all, GLP-1 agonists have been associated with a significant decrease in weight in patients with DM-2 (answers 1,3 and 4 incorrect). On the other hand, pioglitazone, as reflected in its data sheet, can produce dose-dependent weight gain, mainly due to fat accumulation and added, in some cases, to water retention.", "type": "ENDOCRINOLOGY", "options": {"1": "Metformin (biguanide).", "2": "Pioglitazone (thiazolidinedione).", "3": "Canagliflozin (sodium-glucose cotransporter 2 inhibitor- iSGLT2).", "4": "Liraglutide (GLP-1 receptor agonist).", "5": NaN}, "correct_option": 2, "explanations": {"1": {"exist": true, "char_ranges": [[60, 268]], "word_ranges": [[9, 41]], "text": "Regarding weight, metformin to a lesser extent, SGLT-2 inhibitors and, above all, GLP-1 agonists have been associated with a significant decrease in weight in patients with DM-2 (answers 1,3 and 4 incorrect)."}, "2": {"exist": true, "char_ranges": [[269, 450]], "word_ranges": [[41, 70]], "text": "On the other hand, pioglitazone, as reflected in its data sheet, can produce dose-dependent weight gain, mainly due to fat accumulation and added, in some cases, to water retention."}, "3": {"exist": true, "char_ranges": [[60, 268]], "word_ranges": [[9, 41]], "text": "Regarding weight, metformin to a lesser extent, SGLT-2 inhibitors and, above all, GLP-1 agonists have been associated with a significant decrease in weight in patients with DM-2 (answers 1,3 and 4 incorrect)."}, "4": {"exist": true, "char_ranges": [[60, 268]], "word_ranges": [[9, 41]], "text": "Regarding weight, metformin to a lesser extent, SGLT-2 inhibitors and, above all, GLP-1 agonists have been associated with a significant decrease in weight in patients with DM-2 (answers 1,3 and 4 incorrect)."}, "5": {"exist": false, "char_ranges": [], "word_ranges": [], "text": ""}}} {"id": 209, "year": 2014, "question_id_specific": 185, "full_question": "A 32-year-old woman requests preconception counseling. The patient reports that she underwent cervical conization for a high-grade intraepithelial lesion (H-SIL) and subsequently had three miscarriages between 20 and 22 weeks gestation. She has no living children. On all three occasions she came to the emergency department with a feeling of weight in the hypogastrium, where she was found to be 8 cm dilated and with prominent amniotic membranes. She had never felt contractions before, what advice would you give her for the next pregnancy?", "full_answer": "This is a typical case of cervical incompetence (and this patient has conization as a risk factor). This pathology consists of dilatation of the cervix in the absence of contractions, requiring cerclage to prevent miscarriage or immature delivery. The cerclage is ideally performed electively, programmed, and not when the patient arrives at the ER \"in extemis\" (which are also done, but with a lower success rate... the ideal is to do them cold).", "type": "GYNECOLOGY AND OBSTETRICS", "options": {"1": "I would prescribe oral atosiban prophylaxis throughout the pregnancy.", "2": "I would offer lung maturation with corticosteroids from 19-20 weeks of gestation.", "3": "I would recommend a cervical cerclage at 14 weeks gestation.", "4": "I would advise her not to attempt any more pregnancies because of the high risk of recurrence.", "5": "I would recommend resorting to assisted reproductive techniques."}, "correct_option": 3, "explanations": {"1": {"exist": false, "char_ranges": [], "word_ranges": [], "text": ""}, "2": {"exist": false, "char_ranges": [], "word_ranges": [], "text": ""}, "3": {"exist": true, "char_ranges": [[0, 247]], "word_ranges": [[0, 38]], "text": "This is a typical case of cervical incompetence (and this patient has conization as a risk factor). This pathology consists of dilatation of the cervix in the absence of contractions, requiring cerclage to prevent miscarriage or immature delivery."}, "4": {"exist": false, "char_ranges": [], "word_ranges": [], "text": ""}, "5": {"exist": false, "char_ranges": [], "word_ranges": [], "text": ""}}} {"id": 282, "year": 2016, "question_id_specific": 55, "full_question": "A 70-year-old female patient is admitted to the ICU after suffering anterior AMI treated by coronary angioplasty and stent placement in the anterior descending artery. Four days later she suddenly presented hypotension that required vigorous volume support, initiation of vasoactive drugs, orotracheal intubation and connection to mechanical ventilation. Physical examination revealed a murmur not previously present. Suspicion of a mechanical complication of the infarction led to transthoracic echocardiography showing pericardial effusion. Mark the CORRECT answer:", "full_answer": "In the case of free wall rupture, there is no palpable frémito.", "type": "CARDIOLOGY AND VASCULAR SURGERY", "options": {"1": "Mortality with medical treatment is 20%.", "2": "In case of free wall rupture there is an oximetric jump in the right ventricle in the Swan-Ganz catheterization.", "3": "In case of free wall rupture, there is no palpable frémito.", "4": "Mechanical complications usually appear on the first post-infarction day.", "5": NaN}, "correct_option": 3, "explanations": {"1": {"exist": false, "char_ranges": [], "word_ranges": [], "text": ""}, "2": {"exist": false, "char_ranges": [], "word_ranges": [], "text": ""}, "3": {"exist": true, "char_ranges": [[0, 63]], "word_ranges": [[0, 12]], "text": "In the case of free wall rupture, there is no palpable frémito."}, "4": {"exist": false, "char_ranges": [], "word_ranges": [], "text": ""}, "5": {"exist": false, "char_ranges": [], "word_ranges": [], "text": ""}}} {"id": 464, "year": 2019, "question_id_specific": 155, "full_question": "A 58-year-old man, three weeks after a severe ankle sprain presents, rapidly progressive, with dyspnea at rest, dizziness and syncope. On arrival at the hospital he has hypotension (systolic BP 80 mmHg, diastolic 40 mmHg) and poor perfusion. He is intubated and connected to mechanical ventilation and noradrenaline is started. Echocardiogram shows signs of pulmonary hypertension. Angio-CT shows multiple repletion defects occupying both main pulmonary arteries. Which of the following treatments would be associated with the most rapid hemodynamic improvement in this case?", "full_answer": "The patient presented pulmonary thromboembolism which, in addition to being bilateral, produced severe hemodynamic involvement, shock and required mechanical ventilation. We are not told that he has any contraindication to fibrinolysis, so this would be the most appropriate option due to its rapid administration and action. Sodium heparin is reserved for cases in which the patient is stable. Thromboendarterectomy could be performed urgently in selected centers (not available in all) in patients in whom systemic fibrinolysis is contraindicated, or in centers where the experience with this technique is proven and it can be performed quickly. The inferior vena cava filter is a treatment indicated in the acute phase in stable patients, when anticoagulation is contraindicated; or a posteriori, as prophylaxis, in this group of patients. The indications for fibrinolysis in the last SEPAR consensus document included patients at intermediate risk (they could benefit from it) and high risk. They classify intermediate risk as PESIs ≥ 1 or PESI III-IV. In this subgroup, patients with right ventricular dysfunction, troponin or BNP above the cutoff and deep vein thrombosis appear to benefit most from fibrinolysis. In the subgroup of high-risk patients (defined by hypotension or cardiogenic shock criteria), the use of systemic fibrinolysis is much clearer, as in the case presented in the question.", "type": "CRITICAL CARE AND EMERGENCIES", "options": {"1": "Intravenous perfused sodium heparin.", "2": "Thromboendartectomy.", "3": "Systemic fibrinolysis with rt-PA (alteplase) 100 mg intravenous.", "4": "Inferior vena cava filter.", "5": NaN}, "correct_option": 3, "explanations": {"1": {"exist": true, "char_ranges": [[326, 394]], "word_ranges": [[46, 58]], "text": "Sodium heparin is reserved for cases in which the patient is stable."}, "2": {"exist": true, "char_ranges": [[395, 647]], "word_ranges": [[58, 95]], "text": "Thromboendarterectomy could be performed urgently in selected centers (not available in all) in patients in whom systemic fibrinolysis is contraindicated, or in centers where the experience with this technique is proven and it can be performed quickly."}, "3": {"exist": true, "char_ranges": [[0, 325]], "word_ranges": [[0, 46]], "text": "The patient presented pulmonary thromboembolism which, in addition to being bilateral, produced severe hemodynamic involvement, shock and required mechanical ventilation. We are not told that he has any contraindication to fibrinolysis, so this would be the most appropriate option due to its rapid administration and action."}, "4": {"exist": true, "char_ranges": [[648, 842]], "word_ranges": [[95, 125]], "text": "The inferior vena cava filter is a treatment indicated in the acute phase in stable patients, when anticoagulation is contraindicated; or a posteriori, as prophylaxis, in this group of patients."}, "5": {"exist": false, "char_ranges": [], "word_ranges": [], "text": ""}}} {"id": 502, "year": 2020, "question_id_specific": 77, "full_question": "6-month-old infant presenting to the emergency department for respiratory distress. Examination: axillary temperature 37.2°C, respiratory rate 40 rpm, heart rate 160 bpm, blood pressure 90/45 mmHg, SatO2 95% on room air. He shows moderate respiratory distress with intercostal and subcostal retraction. Pulmonary auscultation: scattered expiratory rhonchi, elongated expiration and slight decrease in air entry in both lung fields. Cardiac auscultation: no murmurs. It is decided to keep the patient under observation in the hospital for a few hours. What do you consider the most appropriate attitude at this time with regard to the complementary tests?", "full_answer": "The patient probably presents with bronchiolitis. At this stage, no additional tests should be performed unless there is a clinical worsening.", "type": "PEDIATRICS", "options": {"1": "Request venous blood gas, leukocyte count and acute phase reactants.", "2": "Request chest X-ray.", "3": "Request arterial blood gases and acute phase reactants.", "4": "Do not request complementary tests.", "5": NaN}, "correct_option": 4, "explanations": {"1": {"exist": false, "char_ranges": [], "word_ranges": [], "text": ""}, "2": {"exist": false, "char_ranges": [], "word_ranges": [], "text": ""}, "3": {"exist": false, "char_ranges": [], "word_ranges": [], "text": ""}, "4": {"exist": true, "char_ranges": [[0, 142]], "word_ranges": [[0, 21]], "text": "The patient probably presents with bronchiolitis. At this stage, no additional tests should be performed unless there is a clinical worsening."}, "5": {"exist": false, "char_ranges": [], "word_ranges": [], "text": ""}}} {"id": 71, "year": 2012, "question_id_specific": 50, "full_question": "A patient with a history of fever and chest pain comes to the hospital with dyspnea and tachypnea. On physical examination, the blood pressure cyphrads are low, jugular venous pressure is elevated with a deep descending sinus X, and he has a pulsus paradoxus. What pathology should be suspected?", "full_answer": "Low BP with high jugular pressure should always raise the suspicion of cardiac tamponade. Fever and chest pain with dyspnea and tachypnea should raise suspicion of pericardial effusion with hemodynamic compromise. Paradoxical pulse is a typical finding of cardiac tamponade. Therefore, the correct answer is 5.", "type": "ANESTHESIOLOGY, CRITICAL CARE AND EMERGENCIES", "options": {"1": "Ischemic heart disease.", "2": "Dilated cardiomyopathy.", "3": "Severe aortic valve stenosis.", "4": "Constrictive pericarditis.", "5": "Pericardial effusion with cardiac tamponade."}, "correct_option": 5, "explanations": {"1": {"exist": false, "char_ranges": [], "word_ranges": [], "text": ""}, "2": {"exist": false, "char_ranges": [], "word_ranges": [], "text": ""}, "3": {"exist": false, "char_ranges": [], "word_ranges": [], "text": ""}, "4": {"exist": false, "char_ranges": [], "word_ranges": [], "text": ""}, "5": {"exist": true, "char_ranges": [[0, 274]], "word_ranges": [[0, 40]], "text": "Low BP with high jugular pressure should always raise the suspicion of cardiac tamponade. Fever and chest pain with dyspnea and tachypnea should raise suspicion of pericardial effusion with hemodynamic compromise. Paradoxical pulse is a typical finding of cardiac tamponade."}}} {"id": 486, "year": 2020, "question_id_specific": 45, "full_question": "15-year-old female presenting with delayed menarche and short stature. She does not have intellectual disability. Which of the following genetic tests would be routinely used for the diagnosis of this patient:", "full_answer": "This question may be a little more difficult, as the clinical case presented to us is very vague. An adolescent with pubertal delay and short stature; nothing more. With this information, she could have several genetic pathologies, but this is the MIR and we are told that the patient does not have an intellectual disability. In the MIR, an adolescent with pubertal delay and short stature should be a Turner until proven otherwise (there were two questions in 2017). Therefore, if that is our first diagnostic suspicion, the HABITUAL test (as the statement says) for diagnosis should be a conventional karyotype (formula: 45, X0). A Turner can also be diagnosed with a FISH (there would only be one signal for the X instead of the two that females usually have) or with an array. However, both array and NGS are usually reserved for patients with non-obvious clinical pictures, intellectual disability, or suspicion of a genetic syndrome that requires these specific tests (for example, an array to diagnose a 22q11 deletion syndrome or an NGS panel to diagnose a Noonan syndrome).", "type": "GENETICS", "options": {"1": "Massive sequencing (NGS).", "2": "FISH.", "3": "DNA and/or RNA microarrays.", "4": "Karyotype.", "5": NaN}, "correct_option": 4, "explanations": {"1": {"exist": true, "char_ranges": [[791, 974]], "word_ranges": [[135, 161]], "text": "both array and NGS are usually reserved for patients with non-obvious clinical pictures, intellectual disability, or suspicion of a genetic syndrome that requires these specific tests"}, "2": {"exist": true, "char_ranges": [[633, 763]], "word_ranges": [[104, 130]], "text": "A Turner can also be diagnosed with a FISH (there would only be one signal for the X instead of the two that females usually have)"}, "3": {"exist": true, "char_ranges": [[791, 974]], "word_ranges": [[135, 161]], "text": "both array and NGS are usually reserved for patients with non-obvious clinical pictures, intellectual disability, or suspicion of a genetic syndrome that requires these specific tests"}, "4": {"exist": true, "char_ranges": [[339, 632]], "word_ranges": [[58, 104]], "text": "an adolescent with pubertal delay and short stature should be a Turner until proven otherwise (there were two questions in 2017). Therefore, if that is our first diagnostic suspicion, the HABITUAL test (as the statement says) for diagnosis should be a conventional karyotype (formula: 45, X0)."}, "5": {"exist": false, "char_ranges": [], "word_ranges": [], "text": ""}}} {"id": 216, "year": 2014, "question_id_specific": 121, "full_question": "A 38-year-old man consults for dyspnea and hemoptysis. Blood tests show creatinine 7 mg/dL, urea 250 mg/dL and high titer positive anti-GBM (anti-glomerular basement membrane antibodies). Renal biopsy shows crescents in 75% of the glomeruli and immunofluorescence shows a linear Ig deposition pattern. Which of the following is the correct answer?", "full_answer": "The diagnostic suspicion for the data given (especially the anti-GBM antibodies and the presence of renopulmonary syndrome) is that of Goodpasture's syndrome. The rest of the data (seminules in the biopsy, linear IgG deposition) supports the diagnosis. Knowing this, the options are easily discarded: option 1 speaks to us of an IgA nephropathy (it is not the case because the deposits are of IgG); option 3 is also discarded, because it is not a primary GMN, like the membranous one, but a secondary glomerulopathy; option 4 is also false: the initial treatment is with corticosteroids and cyclophosphamide associated to plasmapheresis; and option 5 is also false, since the damage is not due to circulating immunocomplexes, but to antibodies deposited in the glomerular basement membrane. This leaves option 2 as true: as it has been previously said, the treatment would be performed combining corticosteroids, cyclophosphamide and plasmapheresis.", "type": "NEPHROLOGY", "options": {"1": "It is an IgA nephropathy with acute renal failure.", "2": "Plasmapheresis would be indicated.", "3": "It is a membranous glomerulonephritis.", "4": "Mycophenolate mofetil is the initial treatment of choice.", "5": "Glomerular involvement is caused by the presence of circulating immunocomplexes."}, "correct_option": 2, "explanations": {"1": {"exist": true, "char_ranges": [[301, 398]], "word_ranges": [[44, 64]], "text": "option 1 speaks to us of an IgA nephropathy (it is not the case because the deposits are of IgG);"}, "2": {"exist": true, "char_ranges": [[0, 158]], "word_ranges": [[0, 22]], "text": "The diagnostic suspicion for the data given (especially the anti-GBM antibodies and the presence of renopulmonary syndrome) is that of Goodpasture's syndrome."}, "3": {"exist": true, "char_ranges": [[399, 516]], "word_ranges": [[64, 84]], "text": "option 3 is also discarded, because it is not a primary GMN, like the membranous one, but a secondary glomerulopathy;"}, "4": {"exist": true, "char_ranges": [[517, 637]], "word_ranges": [[84, 100]], "text": "option 4 is also false: the initial treatment is with corticosteroids and cyclophosphamide associated to plasmapheresis;"}, "5": {"exist": true, "char_ranges": [[642, 790]], "word_ranges": [[101, 124]], "text": "option 5 is also false, since the damage is not due to circulating immunocomplexes, but to antibodies deposited in the glomerular basement membrane."}}} {"id": 384, "year": 2016, "question_id_specific": 151, "full_question": "A woman comes to the office with her 3 year old daughter because she has detected a slight mammary development since 3 months without taking any medication or any relevant history. Indeed, the physical examination shows a Tanner stage IV, with no growth of pubic or axillary hair. The external genitalia are normal. Ultrasonography reveals a small uterus and radiology reveals a bone age of 3 years. What attitude should be adopted?", "full_answer": "It seems that they want to present us with precocious puberty (or premature telarche) but they do not provide any analytical data and the ultrasound data are ambiguous (we should assume that by a small uterus they are referring to a prepubertal uterus, but they do not provide any data on ovarian size). We are presented with the case of a three-year-old girl with advanced mammary development, in principle without any associated cause (in principle she does not take drugs that can increase the level of estrogen in the blood, she does not seem to use body creams or eat a lot of chicken meat). If we follow the diagnostic scheme for a premature telarche or suspicion of precocious puberty, we request bone age and abdominal ultrasound (the EO is not advanced as in precocious puberty, and we assume that with a small uterus they mean a prepubertal uterus); according to the complementary examinations that we are given, it does not seem to be precocious puberty, except for the clinical (Tanner IV). Strictly speaking, without analytical hormonal data, it seems that we could mark option 1, being necessary to follow the girl closely. If we take all the above data for granted, we could rule out option 4, which would be the treatment of a central precocious puberty. Regarding the option of mammography, breast ultrasound is used in pediatrics, and in this case it would be indicated if we were told that there is breast asymmetry (we discard option 3). Regarding breast biopsy, it would only be indicated if there are warning signs.", "type": "PEDIATRICS", "options": {"1": "Follow-up every 3-4 months, as this is a temporary condition that often resolves on its own.", "2": "Breast biopsy.", "3": "Mammography.", "4": "Administration of GnRh analogues.", "5": NaN}, "correct_option": 1, "explanations": {"1": {"exist": true, "char_ranges": [[597, 965]], "word_ranges": [[105, 166]], "text": "If we follow the diagnostic scheme for a premature telarche or suspicion of precocious puberty, we request bone age and abdominal ultrasound (the EO is not advanced as in precocious puberty, and we assume that with a small uterus they mean a prepubertal uterus); according to the complementary examinations that we are given, it does not seem to be precocious puberty,"}, "2": {"exist": true, "char_ranges": [[1458, 1537]], "word_ranges": [[250, 263]], "text": "Regarding breast biopsy, it would only be indicated if there are warning signs."}, "3": {"exist": true, "char_ranges": [[1271, 1457]], "word_ranges": [[218, 250]], "text": "Regarding the option of mammography, breast ultrasound is used in pediatrics, and in this case it would be indicated if we were told that there is breast asymmetry (we discard option 3)."}, "4": {"exist": true, "char_ranges": [[597, 965]], "word_ranges": [[105, 166]], "text": "If we follow the diagnostic scheme for a premature telarche or suspicion of precocious puberty, we request bone age and abdominal ultrasound (the EO is not advanced as in precocious puberty, and we assume that with a small uterus they mean a prepubertal uterus); according to the complementary examinations that we are given, it does not seem to be precocious puberty,"}, "5": {"exist": false, "char_ranges": [], "word_ranges": [], "text": ""}}} {"id": 238, "year": 2014, "question_id_specific": 145, "full_question": "A 40-year-old woman consults for approximately 20 episodes per day of intense left periocular pain lasting 15 minutes, accompanied by intense tearing and rhinorrhea. Her examination and MRI are normal. His treatment of choice would be:", "full_answer": "In this question we are presented with a case to deduce a diagnosis and then indicate which would be the treatment of choice. Both the characteristics of the case and the answers suggest that it is a trigemino-autonomic headache (intense, periocular pain with lacrimation and rhinorrhea). The idea is to make a differential diagnosis mainly between a cluster headache (whose treatments include answers 2, 3, 4 and 5), a paroxysmal hemicrania (whose treatment of choice is the indomethacin of answer 1) and a SUNCT (unilateral neuralgiform headache with conjunctival injection and lacrimation). Cluster headache predominates in males whose duration can vary between 15-180 minutes, between once every 2 days, up to 8 times a day. Paroxysmal hemicrania predominates in women, with episodes of pain similar to cluster headache, but with a shorter duration (2-30 min), and a higher frequency (5-30 episodes per day). As for SUNCT, the crises are much shorter, lasting seconds (5-240 seconds) and are usually refractory to treatment. Therefore the diagnosis would be of a paroxysmal hemicrania and its treatment of choice is indomethacin (which also the answer is a diagnostic criterion). Therefore correct answer 1 (Indomethacin). All data provided are based on the Diagnostic Criteria of the International Headache Society.", "type": "NEUROLOGY", "options": {"1": "Indomethacin.", "2": "Lamotrigine.", "3": "Verapamil.", "4": "Prednisone.", "5": "Lithium carbonate."}, "correct_option": 1, "explanations": {"1": {"exist": true, "char_ranges": [[729, 1183]], "word_ranges": [[115, 185]], "text": "Paroxysmal hemicrania predominates in women, with episodes of pain similar to cluster headache, but with a shorter duration (2-30 min), and a higher frequency (5-30 episodes per day). As for SUNCT, the crises are much shorter, lasting seconds (5-240 seconds) and are usually refractory to treatment. Therefore the diagnosis would be of a paroxysmal hemicrania and its treatment of choice is indomethacin (which also the answer is a diagnostic criterion)."}, "2": {"exist": true, "char_ranges": [[594, 728]], "word_ranges": [[92, 115]], "text": "Cluster headache predominates in males whose duration can vary between 15-180 minutes, between once every 2 days, up to 8 times a day."}, "3": {"exist": true, "char_ranges": [[594, 728]], "word_ranges": [[92, 115]], "text": "Cluster headache predominates in males whose duration can vary between 15-180 minutes, between once every 2 days, up to 8 times a day."}, "4": {"exist": true, "char_ranges": [[594, 728]], "word_ranges": [[92, 115]], "text": "Cluster headache predominates in males whose duration can vary between 15-180 minutes, between once every 2 days, up to 8 times a day."}, "5": {"exist": true, "char_ranges": [[594, 728]], "word_ranges": [[92, 115]], "text": "Cluster headache predominates in males whose duration can vary between 15-180 minutes, between once every 2 days, up to 8 times a day."}}} {"id": 47, "year": 2011, "question_id_specific": 157, "full_question": "A 6-year-old boy comes to the clinic accompanied by the monitor of a day care center in our neighborhood because of a painful lump 3 cm in diameter on palpation in the right occipital area of the scalp. He suffers from alopecia in this area and 3 adenomegalies of quite hard consistency in the right posterior cervical region. What would be the most appropriate treatment?", "full_answer": "The correct answer is 3. If I am not mistaken, you are describing a Celso's kerion for which the treatment of choice is oral griseofulvin.", "type": "PEDIATRICS", "options": {"1": "Incision and drainage.", "2": "Topical Mupirocin.", "3": "Griseofulvin orally.", "4": "Intravenous cefazolin.", "5": "Topical Ketoconazole."}, "correct_option": 3, "explanations": {"1": {"exist": false, "char_ranges": [], "word_ranges": [], "text": ""}, "2": {"exist": false, "char_ranges": [], "word_ranges": [], "text": ""}, "3": {"exist": true, "char_ranges": [[25, 138]], "word_ranges": [[5, 25]], "text": "If I am not mistaken, you are describing a Celso's kerion for which the treatment of choice is oral griseofulvin."}, "4": {"exist": false, "char_ranges": [], "word_ranges": [], "text": ""}, "5": {"exist": false, "char_ranges": [], "word_ranges": [], "text": ""}}} {"id": 381, "year": 2016, "question_id_specific": 139, "full_question": "A 70-year-old woman with a history of anorexia, weight loss, discomfort in the muscles and proximal joints and pain in the temporomandibular region who comes to the emergency department for unilateral loss of vision (hand movement), sudden and painless onset (afferent pupillary defect).what test would you request first for diagnostic purposes?", "full_answer": "We are presented with a case of monocular amaurosis with a vascular profile, also in an older woman with a history of weight loss and what seems to be symptoms of mandibular claudication and polymyalgia rheumatica, so the first cause to think about is giant cell arteritis as the cause of NOIA, so the correct option would be 2. Option 4 would be considered if we were told of an AINO but with non-arteritic characteristics (without all the accompanying symptomatology).", "type": "NEUROLOGY", "options": {"1": "Lumbar puncture.", "2": "C Reactive Protein.", "3": "Magnetic resonance angiography.", "4": "Carotid ultrasound.", "5": NaN}, "correct_option": 2, "explanations": {"1": {"exist": false, "char_ranges": [], "word_ranges": [], "text": ""}, "2": {"exist": true, "char_ranges": [[218, 328]], "word_ranges": [[37, 59]], "text": "the first cause to think about is giant cell arteritis as the cause of NOIA, so the correct option would be 2."}, "3": {"exist": false, "char_ranges": [], "word_ranges": [], "text": ""}, "4": {"exist": true, "char_ranges": [[329, 470]], "word_ranges": [[59, 80]], "text": "Option 4 would be considered if we were told of an AINO but with non-arteritic characteristics (without all the accompanying symptomatology)."}, "5": {"exist": false, "char_ranges": [], "word_ranges": [], "text": ""}}} {"id": 32, "year": 2011, "question_id_specific": 64, "full_question": "In a patient diagnosed with epilepsy who presents with episodes of unresponsiveness to external stimuli, irregular movements of all four limbs, closed eyes, crying and pelvic movements, lasting five to twenty seconds and unresponsive to treatment with antiepileptic drugs, which complementary study is most likely to clarify the diagnosis?", "full_answer": "Correct answer 1: The picture described is very suggestive of pseudocrisis with asynchronous limb movements, pelvic movements, crying and poor response to antiepileptic drugs.", "type": "NEUROLOGY AND NEUROSURGERY", "options": {"1": "Video-EEG monitoring for diagnosis of pseudocrisis (psychogenic seizures).", "2": "Holter ECG for diagnosis of arrhythmic heart disease.", "3": "Routine EEG to diagnose the type of epilepsy (generalized or foc).", "4": "Brain MRI to detect epileptogenic lesions (cortical dysplasia, tumor, medial temporal sclerosis).", "5": "Determine capillary blood glucose for diagnosis of hypoglycemia."}, "correct_option": 1, "explanations": {"1": {"exist": true, "char_ranges": [[18, 175]], "word_ranges": [[3, 24]], "text": "The picture described is very suggestive of pseudocrisis with asynchronous limb movements, pelvic movements, crying and poor response to antiepileptic drugs."}, "2": {"exist": false, "char_ranges": [], "word_ranges": [], "text": ""}, "3": {"exist": false, "char_ranges": [], "word_ranges": [], "text": ""}, "4": {"exist": false, "char_ranges": [], "word_ranges": [], "text": ""}, "5": {"exist": false, "char_ranges": [], "word_ranges": [], "text": ""}}} {"id": 106, "year": 2012, "question_id_specific": 135, "full_question": "2-year-old boy. His personal history includes 3 episodes of acute otitis media, 1 meningococcal meningitis and 2 pneumonias (one middle lobe and one left upper lobe). She has been admitted on 3 occasions for thrombopenic purpura (on three occasions antiplatelet antibodies were negative and bone marrow showed normal megakaryocytes). Several males of the maternal family had died in childhood due to infectious processes. Physical examination showed lesions typical of atopic dermatitis. The immunological study showed a slight decrease in T-lymphocyte subpopulations; elevated IgA and IgE; decreased IgM and IgG at the lower limit of normal. What is the most likely diagnosis?", "full_answer": "To correctly answer this question it should be emphasized that among the symptomatology that is exposed we find: - 2-year-old child. - ENT infections. - Pulmonary infections. - Hospital admissions. - PTI. - Family history of infections with deaths due to infections in males (maternal family). - Atopic Dermatitis. All these data, in addition to the analytical data, are describing an immunodeficiency that by the family-maternal history, seems to be X-linked, as several males have died of a similar clinical condition (the father of the patient contributed the Y chromosome, the mother the X). Among the X-linked immunodeficiencies is Wiskott-Aldrich syndrome, an entity described with an initial triad of symptoms consisting of bleeding (typical BUT absent in the case: heavy bleeding after circumcision, bloody diarrhea), recurrent infections and eczema. In addition to thrombocytopenia, they are at increased risk for autoimmune phenomena and lymphoid neoplasms. The Hyper-IgE option lacks very characteristic clinical data such as bone alterations and skin lesions, which are not atopic dermatitis, since they follow a different pattern (papulopustular rash on the face and scalp). The option of transient hypogammaglobulinemia of infancy and the common severe and variable combined immunodeficiency fails, among other features, the determination of immunoglobulins G and M, which are at the lower limit, but within normality.", "type": "GENETICS AND IMMUNOLOGY", "options": {"1": "Wiskott-Aldrich syndrome.", "2": "Hyper IgE syndrome.", "3": "Transient hypogammaglobulinemia of childhood.", "4": "X-linked severe combined immunodeficiency.", "5": "Common variable immunodeficiency."}, "correct_option": 4, "explanations": {"1": {"exist": true, "char_ranges": [[596, 858]], "word_ranges": [[94, 129]], "text": "Among the X-linked immunodeficiencies is Wiskott-Aldrich syndrome, an entity described with an initial triad of symptoms consisting of bleeding (typical BUT absent in the case: heavy bleeding after circumcision, bloody diarrhea), recurrent infections and eczema."}, "2": {"exist": true, "char_ranges": [[968, 1187]], "word_ranges": [[144, 177]], "text": "The Hyper-IgE option lacks very characteristic clinical data such as bone alterations and skin lesions, which are not atopic dermatitis, since they follow a different pattern (papulopustular rash on the face and scalp)."}, "3": {"exist": true, "char_ranges": [[1188, 1432]], "word_ranges": [[177, 212]], "text": "The option of transient hypogammaglobulinemia of infancy and the common severe and variable combined immunodeficiency fails, among other features, the determination of immunoglobulins G and M, which are at the lower limit, but within normality."}, "4": {"exist": true, "char_ranges": [[367, 595]], "word_ranges": [[58, 94]], "text": "are describing an immunodeficiency that by the family-maternal history, seems to be X-linked, as several males have died of a similar clinical condition (the father of the patient contributed the Y chromosome, the mother the X)."}, "5": {"exist": false, "char_ranges": [], "word_ranges": [], "text": ""}}} {"id": 62, "year": 2011, "question_id_specific": 119, "full_question": "We are consulted to assess an 83-year-old woman admitted to the Trauma service for a hip fracture 6 hours ago. She has AP of hypertension, LBP, moderate dementia and lives in a nursing home. Her usual treatment is thiazide, atorvastatin, donepezil, Calcium and vitamin D. EF: Confused patient, pulse 90 bpm, respiratory rate 20 rpm, T art 170/88, jugular venous pressure normal. The CBC and chest X-ray are normal and the ECG shows sinus rhythm without ischemic alterations. Which of the following is the most correct therapeutic approach?", "full_answer": "The confusional picture is determined by the dementia that the patient already suffers, so no matter how much we delay surgery, we are not going to achieve anything. The elevation of blood pressure is due, in principle, to pain (so the first option is an analgesic) and then to the stressful situation that leads an already hypertensive patient to increase her blood pressure, so labetalol could help us. The last answer is not correct because it depends on the type of fracture, which is not specified in the statement: a pertrochanteric fracture would require closed osteosynthesis and a subcapital fracture would require a prosthesis.", "type": "ANESTHESIOLOGY AND CRITICAL CARE", "options": {"1": "Delay surgery until the confusional picture has disappeared.", "2": "Delay surgery and perform an echocardiogram.", "3": "Delay surgery until good blood pressure control.", "4": "Start a beta-blocker and initiate surgery.", "5": "Perform closed osteosynthesis, avoiding in any case the implantation of prosthesis."}, "correct_option": 4, "explanations": {"1": {"exist": true, "char_ranges": [[0, 165]], "word_ranges": [[0, 28]], "text": "The confusional picture is determined by the dementia that the patient already suffers, so no matter how much we delay surgery, we are not going to achieve anything."}, "2": {"exist": false, "char_ranges": [], "word_ranges": [], "text": ""}, "3": {"exist": true, "char_ranges": [[166, 404]], "word_ranges": [[28, 68]], "text": "The elevation of blood pressure is due, in principle, to pain (so the first option is an analgesic) and then to the stressful situation that leads an already hypertensive patient to increase her blood pressure, so labetalol could help us."}, "4": {"exist": false, "char_ranges": [], "word_ranges": [], "text": ""}, "5": {"exist": true, "char_ranges": [[405, 637]], "word_ranges": [[68, 104]], "text": "The last answer is not correct because it depends on the type of fracture, which is not specified in the statement: a pertrochanteric fracture would require closed osteosynthesis and a subcapital fracture would require a prosthesis."}}} {"id": 592, "year": 2022, "question_id_specific": 76, "full_question": "A 40-year-old woman consults because she has noticed a lump in the superoexternal quadrant of the right breast for the past month. She provides a mammography report describing a BIRADS 3 lesion. What is the best course of action?", "full_answer": "BI-RADS Breast Imaging Reporting and Data System. BI-RADS 3 is defined by answer 3.", "type": "ONCOLOGY", "options": {"1": "Reassure him, since an imaging test has already been done and malignancy has been ruled out.", "2": "This classification probably implies surgery since the probability of cancer is greater than 10%. He explains it to you and refers you preferentially to the Breast Unit.", "3": "This is a probably benign finding, since there is less than a 2% chance of cancer. He explains that it requires follow-up every 6-12 months until 24 months or a biopsy.", "4": "The findings are of low suspicion of cancer (between 2 and 10 %) but a biopsy is necessary.", "5": NaN}, "correct_option": 3, "explanations": {"1": {"exist": false, "char_ranges": [], "word_ranges": [], "text": ""}, "2": {"exist": false, "char_ranges": [], "word_ranges": [], "text": ""}, "3": {"exist": true, "char_ranges": [[0, 83]], "word_ranges": [[0, 14]], "text": "BI-RADS Breast Imaging Reporting and Data System. BI-RADS 3 is defined by answer 3."}, "4": {"exist": false, "char_ranges": [], "word_ranges": [], "text": ""}, "5": {"exist": false, "char_ranges": [], "word_ranges": [], "text": ""}}} {"id": 26, "year": 2011, "question_id_specific": 117, "full_question": "An immunodeficient patient who presents a pneumonia with meniscus halo sign or crescentic contour on chest X-ray/CT suggests infection by a microorganism:", "full_answer": "The halo sign is characteristic of pulmonary aspergillosis, especially in an immuodepressed patient. But it is not pathognomonic; it has also been associated with TB, some neoplasms and Wegener's granulomatosis.", "type": "INFECTIOUS", "options": {"1": "Staphylococcus aureus.", "2": "Streptococcus pneumoniae.", "3": "Candida albicans.", "4": "Pseudomonas aeruginosa.", "5": "Aspergillus fumigatus."}, "correct_option": 5, "explanations": {"1": {"exist": false, "char_ranges": [], "word_ranges": [], "text": ""}, "2": {"exist": false, "char_ranges": [], "word_ranges": [], "text": ""}, "3": {"exist": false, "char_ranges": [], "word_ranges": [], "text": ""}, "4": {"exist": false, "char_ranges": [], "word_ranges": [], "text": ""}, "5": {"exist": true, "char_ranges": [[0, 100]], "word_ranges": [[0, 13]], "text": "The halo sign is characteristic of pulmonary aspergillosis, especially in an immuodepressed patient."}}} {"id": 435, "year": 2018, "question_id_specific": 121, "full_question": "A 45-year-old man consults for a productive cough, pleuritic pain in the right flank and fever of 48 h of evolution. He has a baseline O2 saturation of 88% and rales in the right base. Chest X-ray shows a right basal consolidation. She has a history of HIV infection well controlled with antiretroviral drugs (CD4 lymphocytes 550 ce/uL and undetectable HIV viral load). Which of the following empirical antimicrobial treatments do you consider most appropriate?", "full_answer": "The first option seems to be the correct one, due to the low probability of P. jirovecii infection in a patient with more than 500 CD4 lymphocytes and undetectable viral load. It is most likely to be a pneumococcal infection that we cover with Ceftriaxone and with azithromycin we cover the so-called \"atypical\" ones. Meropenem is an antibiotic with too broad a spectrum, which would be an option in in-hospital pneumonia caused by P. aeruginosa, a bacterial agent that can also cause pneumonia in HIV patients, but this is not common.", "type": "INFECTIOUS DISEASES AND MICROBIOLOGY", "options": {"1": "Cefiriaxone 2 g and azithromycin 500 mg every 24 hours.", "2": "Cefiriaxone 2 g, azithrornicin 500 mg every 24 h and trimethoprim-sulfamethoxazole 5 mg/kg/8 h (based on trimethoprim doses).", "3": "Methyl-prednisolone 40 mg/day, cefiriaxone 2 g IV 124 h and trimethoprim-sulfamethoxazole 5 mg/kg/8 h (based on trimethoprim doses).", "4": "Meropenem I g/8 h and vancomycin I g/l2 h.", "5": NaN}, "correct_option": 1, "explanations": {"1": {"exist": true, "char_ranges": [[176, 317]], "word_ranges": [[31, 54]], "text": "It is most likely to be a pneumococcal infection that we cover with Ceftriaxone and with azithromycin we cover the so-called \"atypical\" ones."}, "2": {"exist": false, "char_ranges": [], "word_ranges": [], "text": ""}, "3": {"exist": false, "char_ranges": [], "word_ranges": [], "text": ""}, "4": {"exist": true, "char_ranges": [[318, 535]], "word_ranges": [[54, 91]], "text": "Meropenem is an antibiotic with too broad a spectrum, which would be an option in in-hospital pneumonia caused by P. aeruginosa, a bacterial agent that can also cause pneumonia in HIV patients, but this is not common."}, "5": {"exist": false, "char_ranges": [], "word_ranges": [], "text": ""}}} {"id": 474, "year": 2020, "question_id_specific": 142, "full_question": "A 70-year-old woman, diabetic and hypertensive, who suffers a fall at home, presenting a 9 cm wound communicating with a fracture site of the right tibia. Radiographically, a short oblique fracture of the mid-distal third of the tibia was observed. An emergency operation was performed by cleaning (Friederich) and placement of an endomedullary steel-plated nail. At 11 months he presents with atrophic pseudarthrosis of the tibia with suppuration in the wound area. What will be his best immediate therapeutic option?", "full_answer": "The first thing is to treat the infection and for this we must remove all the osteosynthesis material, debride and give antibiotherapy. Fixation in this case is better external to avoid all the material around the affected area, so 4 is correct and 1 and 2 is not. The 3 would be considered after expiration of the infectious picture. TREATMENT - Suppressive antibiotic treatment: indicated in patients with Ciemy type C, it consists of prolonged oral antibiotic therapy for at least 6 months to \"cool\" the exacerbation of the clinical picture. - Curative treatment: includes a first surgery with aggressive tumor-like debridement of all affected tissues, profuse irrigation, stabilization with external fixator if stability is compromised, and eventual filling of the cavities with antibiotic-releasing substances. After prolonged periods of antibiotic therapy selected according to the antibiograms of the cultures, and once it is certain that the infection has been cured, special techniques for the reconstruction of the bone defect and to achieve adequate coverage of soft tissues should be considered. We are dealing with an infection associated with an implant (intramedullary nail) and pseudoarthrosis of the tibia fracture (absence of union after 11 months). The infection is understood by the clinical manifestations (suppuration, nonunion) and the history of diabetes and open fracture. Management overlaps with the management of chronic osteomyelitis. Isolated antibiotherapy (option 2) is indicated only in patients with severe comorbidity in whom surgical treatment would be more aggressive than continuing the disease. Nail dynamization associated with broad-spectrum antibiotherapy (option 1) is not indicated either, because (a) it has not demonstrated benefit in established pseudarthrosis and (b) we have the same issue as option 1, we do not eliminate biofilm. Option 3, discussed, would be considered only if there was no associated infection.", "type": "ORTHOPEDIC SURGERY AND TRAUMATOLOGY", "options": {"1": "Triple antibiotherapy (gram-positive, gram-negative and anaerobic) and cleaning of the surgical wound, removing the distal locks to promote bone consolidation.", "2": "Expectant attitude and antibiotic treatment with quinolones.", "3": "Autologous graft and growth factors (BMP 2 and 7) to stimulate the bone consolidation process, which is slowing.", "4": "Removal of the nail, debridement, placement of external fixator and antibiotherapy adjusted to culture results.", "5": NaN}, "correct_option": 4, "explanations": {"1": {"exist": true, "char_ranges": [[136, 264]], "word_ranges": [[22, 48]], "text": "Fixation in this case is better external to avoid all the material around the affected area, so 4 is correct and 1 and 2 is not."}, "2": {"exist": true, "char_ranges": [[136, 264]], "word_ranges": [[22, 48]], "text": "Fixation in this case is better external to avoid all the material around the affected area, so 4 is correct and 1 and 2 is not."}, "3": {"exist": true, "char_ranges": [[265, 334]], "word_ranges": [[48, 59]], "text": "The 3 would be considered after expiration of the infectious picture."}, "4": {"exist": true, "char_ranges": [[136, 264]], "word_ranges": [[22, 48]], "text": "Fixation in this case is better external to avoid all the material around the affected area, so 4 is correct and 1 and 2 is not."}, "5": {"exist": false, "char_ranges": [], "word_ranges": [], "text": ""}}} {"id": 17, "year": 2011, "question_id_specific": 137, "full_question": "A 14-year-old female patient in good general condition presents since 4 days ago a very pruritic generalized cutaneous eruption formed by erythematous-edematous plaques between 2 and 15 cm in diameter without desquamation with a tendency to acquire an annular morphology that individually disappear in less than 24 hours. The mucous membranes are respected. Your first diagnostic impression would be:", "full_answer": "Acute urticaria: characterized by erythematous-edematous, evanescent, pruritic, evanescent lesions, lasting less than 24 hours, without desquamation. The general condition is usually preserved. In children the annular pattern is more frequent. In both rubella and toxicoderma, the general condition is not preserved. In staphylococcal shock, the general condition is affected and the lesions are blistering. Scabies lesions are preferably interdigital, in the form of papulocoses that can follow linear trajectories.", "type": "DERMATOLOGY", "options": {"1": "Urticaria.", "2": "Rubella.", "3": "Toxicoderma.", "4": "Staphylococcal toxic shock.", "5": "Scabies."}, "correct_option": 1, "explanations": {"1": {"exist": true, "char_ranges": [[0, 315]], "word_ranges": [[0, 41]], "text": "Acute urticaria: characterized by erythematous-edematous, evanescent, pruritic, evanescent lesions, lasting less than 24 hours, without desquamation. The general condition is usually preserved. In children the annular pattern is more frequent. In both rubella and toxicoderma, the general condition is not preserved."}, "2": {"exist": false, "char_ranges": [], "word_ranges": [], "text": ""}, "3": {"exist": false, "char_ranges": [], "word_ranges": [], "text": ""}, "4": {"exist": false, "char_ranges": [], "word_ranges": [], "text": ""}, "5": {"exist": false, "char_ranges": [], "word_ranges": [], "text": ""}}} {"id": 425, "year": 2018, "question_id_specific": 92, "full_question": "A 34-year-old woman is admitted for polyuria and polydipsia. In the first 24 hours of admission a diuresis of 8.2 liters is found and a blood test shows a glycemia of 96 mg/dL, natremia of 148 mEq/L and plasma osmolality of 309 mOsm/kg with urinary osmolality of 89 mOsmlkg. What diagnostic test should be performed next?", "full_answer": "We are faced with polyuria. Initially we rule out diabetes mellitus (our patient has a normal blood glucose of 96mg/dl). The analytical data suggest diabetes insipidus (high plasma osm with low urinary osm). Now we must differentiate between central diabetes insipidus (lack of ADH) or nephrogenic diabetes insipidus (ADH does not exert its action at the renal level). This is achieved by the vasopressin test (intravenous administration of ADH and remeasurement of urinary osmolarity). Therefore, the correct answer is option 3.", "type": "ENDOCRINOLOGY", "options": {"1": "Hypertonic saline infusion test for serial determination of antidiuretic hormone.", "2": "Dehydration test (Miller test).", "3": "Administration of desmopressin with serial monitoring of urine osmolality.", "4": "Determination of antidiuretic hormone in plasma.", "5": NaN}, "correct_option": 3, "explanations": {"1": {"exist": false, "char_ranges": [], "word_ranges": [], "text": ""}, "2": {"exist": false, "char_ranges": [], "word_ranges": [], "text": ""}, "3": {"exist": true, "char_ranges": [[121, 529]], "word_ranges": [[20, 81]], "text": "The analytical data suggest diabetes insipidus (high plasma osm with low urinary osm). Now we must differentiate between central diabetes insipidus (lack of ADH) or nephrogenic diabetes insipidus (ADH does not exert its action at the renal level). This is achieved by the vasopressin test (intravenous administration of ADH and remeasurement of urinary osmolarity). Therefore, the correct answer is option 3."}, "4": {"exist": false, "char_ranges": [], "word_ranges": [], "text": ""}, "5": {"exist": false, "char_ranges": [], "word_ranges": [], "text": ""}}} {"id": 8, "year": 2011, "question_id_specific": 232, "full_question": "A 52-year-old woman consulted because she had noticed during the previous week a yellowish discoloration of the conjunctivae. She does not refer to risky sexual behaviors or epidemiological history of risk of viral hepatitis. She does not consume alcohol or hepatotoxic drugs. She reports a one-year history of generalized pruritus, asthenia, dry mouth and absence of lacrimation with no known cause. Rest of the anamnesis without pathological data. Physical examination showed scratching lesions, conjunctival jaundice and non-painful hepatomegaly. The patient brings a blood test carried out in his company with the following pathological results: Bilirubin 3 mg/dl, FA 400 UI/ VSG 40mm 1 hour. Indicate which would be the best recommendation to establish the etiological diagnosis of the patient's condition:", "full_answer": "Book picture of Primary Biliary Cirrhosis, one of those that do not occur in real life. The diagnosis would be practically made with the AMA (1). With 2 we would rule out hemochromatosis, with 3 we would rule out Wilson's disease. With 4 we could rule out rare diseases such as biliary tract malformations or Caroli's disease and with 5 we could rule out viral hepatitis.", "type": "DIGESTIVE", "options": {"1": "Anti-mitochondrial antibodies.", "2": "Study of Fe metabolism.", "3": "Study of copper metabolism.", "4": "Hepatic MRI.", "5": "Serology for B and C viruses."}, "correct_option": 1, "explanations": {"1": {"exist": true, "char_ranges": [[88, 144]], "word_ranges": [[16, 26]], "text": "The diagnosis would be practically made with the AMA (1)."}, "2": {"exist": true, "char_ranges": [[146, 186]], "word_ranges": [[26, 33]], "text": "With 2 we would rule out hemochromatosis,"}, "3": {"exist": true, "char_ranges": [[188, 229]], "word_ranges": [[33, 41]], "text": "with 3 we would rule out Wilson's disease."}, "4": {"exist": true, "char_ranges": [[231, 325]], "word_ranges": [[41, 57]], "text": "With 4 we could rule out rare diseases such as biliary tract malformations or Caroli's disease"}, "5": {"exist": true, "char_ranges": [[330, 371]], "word_ranges": [[58, 66]], "text": "with 5 we could rule out viral hepatitis."}}} {"id": 621, "year": 2022, "question_id_specific": 60, "full_question": "20-year-old patient who comes to the emergency department after suffering a bicycle accident with facial trauma. A cranial CT scan was performed showing a fracture of the middle third of the face involving the orbito-malar region. One of the most frequent complications of this type of fracture is:", "full_answer": "Fractures of the midface in the orbito-malar region may involve the floor and, to a lesser extent, the lateral wall of the orbit. Assuming therefore that we are asked which is one of the most frequent complications of the floor of the orbit, two complications should always be highlighted that may indicate surgical treatment, even urgent: diplopia, due to dislocation of the inferior rectus muscle to the underlying maxillary sinus (and even its entrapment); and enophthalmos, which may cause other associated complications in the medium and long term, such as superior palpebral pseudo-ptosis due to loss of orbital volume (option 4 correct). Temporomandibular ankylosis is not considered because, although relatively close to the orbital cavity, it is not part of the orbito-malar complex (option 1 discarded). Involvement of the maxilla can lead to dental malocclusion, but usually occurs in fractures located lower than the orbital cavity (option 2 discarded). Naso-ethmoidal fractures are included in midface fractures, but the bones of the nose are located more anteriorly to the medial orbital rim, and therefore outside the orbit (option 3 discarded).", "type": "OPHTHALMOLOGY (ECTOPIC)", "options": {"1": "Temporomandibular ankylosis.", "2": "Dental malocclusion.", "3": "Naso-ethmoidal pseudoarthrosis.", "4": "Enophthalmos.", "5": NaN}, "correct_option": 4, "explanations": {"1": {"exist": true, "char_ranges": [[645, 813]], "word_ranges": [[102, 126]], "text": "Temporomandibular ankylosis is not considered because, although relatively close to the orbital cavity, it is not part of the orbito-malar complex (option 1 discarded)."}, "2": {"exist": true, "char_ranges": [[814, 965]], "word_ranges": [[126, 149]], "text": "Involvement of the maxilla can lead to dental malocclusion, but usually occurs in fractures located lower than the orbital cavity (option 2 discarded)."}, "3": {"exist": true, "char_ranges": [[966, 1160]], "word_ranges": [[149, 179]], "text": "Naso-ethmoidal fractures are included in midface fractures, but the bones of the nose are located more anteriorly to the medial orbital rim, and therefore outside the orbit (option 3 discarded)."}, "4": {"exist": true, "char_ranges": [[130, 644]], "word_ranges": [[23, 102]], "text": "Assuming therefore that we are asked which is one of the most frequent complications of the floor of the orbit, two complications should always be highlighted that may indicate surgical treatment, even urgent: diplopia, due to dislocation of the inferior rectus muscle to the underlying maxillary sinus (and even its entrapment); and enophthalmos, which may cause other associated complications in the medium and long term, such as superior palpebral pseudo-ptosis due to loss of orbital volume (option 4 correct)."}, "5": {"exist": false, "char_ranges": [], "word_ranges": [], "text": ""}}} {"id": 470, "year": 2020, "question_id_specific": 138, "full_question": "A 73-year-old woman with a history of obesity, type 2 diabetes mellitus, hypertension and dyslipidemia. She consults for unbearable pain in the right knee of 5 days of evolution, without previous trauma. Examination: globular knee, moderate varus, extension and flexion limited by pain, diffuse medial pain. X-ray shows osteophytes and mild impingement of the medial interlining. What would be his initial management?", "full_answer": "We are being described an acute pain crisis in a patient with gonarthrosis. In this situation, the first thing to do is to resolve the pain crisis and to propose an appropriate conservative treatment for this osteoarthritis (1 correct). A knee arthroplasty is not considered at the outset, so 2 is false. They are not telling us an infectious clinic to suspect an arthritis that would justify a debridement and washing so 3 is false. In a patient with gonarthrosis there will always be meniscopathy, it is part of the degenerative changes. Baker's cyst would only interest us in a severe pain crisis in the differential diagnosis with a deep thrombosis and it is evaluated with an echo-Doppler, in the picture of gonarthrosis it has no value to detect a Baker's cyst. Tendinitis is diagnosed by examination, not with MRI. Therefore, 4 is false.", "type": "ORTHOPEDIC SURGERY AND TRAUMATOLOGY", "options": {"1": "Explanation of the diagnosis, relative rest, paracetamol 1g/8h plus metamizol 500 mg/ 8 h rescue naproxen.", "2": "Preferential referral to Traumatology outpatients for evaluation of total cemented prosthesis.", "3": "Preferential referral to Traumatology outpatient clinic for arthroscopic debridement.", "4": "Preferred MRI request for evaluation of meniscopathy, Baker's cyst and/or tendinitis.", "5": NaN}, "correct_option": 1, "explanations": {"1": {"exist": true, "char_ranges": [[0, 236]], "word_ranges": [[0, 39]], "text": "We are being described an acute pain crisis in a patient with gonarthrosis. In this situation, the first thing to do is to resolve the pain crisis and to propose an appropriate conservative treatment for this osteoarthritis (1 correct)."}, "2": {"exist": true, "char_ranges": [[237, 304]], "word_ranges": [[39, 52]], "text": "A knee arthroplasty is not considered at the outset, so 2 is false."}, "3": {"exist": true, "char_ranges": [[305, 433]], "word_ranges": [[52, 75]], "text": "They are not telling us an infectious clinic to suspect an arthritis that would justify a debridement and washing so 3 is false."}, "4": {"exist": true, "char_ranges": [[540, 844]], "word_ranges": [[92, 144]], "text": "Baker's cyst would only interest us in a severe pain crisis in the differential diagnosis with a deep thrombosis and it is evaluated with an echo-Doppler, in the picture of gonarthrosis it has no value to detect a Baker's cyst. Tendinitis is diagnosed by examination, not with MRI. Therefore, 4 is false."}, "5": {"exist": false, "char_ranges": [], "word_ranges": [], "text": ""}}} {"id": 298, "year": 2016, "question_id_specific": 103, "full_question": "A 25-year-old man with no past history of interest presents to the emergency department with fever, headache, myalgia, nausea, vomiting, abdominal pain, jaundice and conjunctival injection, 2 weeks after traveling to Thailand to participate in a freshwater regatta. What is the most likely diagnosis?", "full_answer": "The history of contact with fresh water in an endemic area points to leptospirosis, and in the clinical picture we also find jaundice and conjunctival injection which also points to leptospirosis and is not so characteristically related to the other three pathogens.", "type": "INFECTIOUS DISEASES", "options": {"1": "Malaria.", "2": "Schistosomiasis.", "3": "Leptospirosis.", "4": "Rabies.", "5": NaN}, "correct_option": 3, "explanations": {"1": {"exist": true, "char_ranges": [[0, 266]], "word_ranges": [[0, 42]], "text": "The history of contact with fresh water in an endemic area points to leptospirosis, and in the clinical picture we also find jaundice and conjunctival injection which also points to leptospirosis and is not so characteristically related to the other three pathogens."}, "2": {"exist": true, "char_ranges": [[0, 266]], "word_ranges": [[0, 42]], "text": "The history of contact with fresh water in an endemic area points to leptospirosis, and in the clinical picture we also find jaundice and conjunctival injection which also points to leptospirosis and is not so characteristically related to the other three pathogens."}, "3": {"exist": true, "char_ranges": [[0, 266]], "word_ranges": [[0, 42]], "text": "The history of contact with fresh water in an endemic area points to leptospirosis, and in the clinical picture we also find jaundice and conjunctival injection which also points to leptospirosis and is not so characteristically related to the other three pathogens."}, "4": {"exist": true, "char_ranges": [[0, 266]], "word_ranges": [[0, 42]], "text": "The history of contact with fresh water in an endemic area points to leptospirosis, and in the clinical picture we also find jaundice and conjunctival injection which also points to leptospirosis and is not so characteristically related to the other three pathogens."}, "5": {"exist": false, "char_ranges": [], "word_ranges": [], "text": ""}}} {"id": 365, "year": 2016, "question_id_specific": 100, "full_question": "A 64-year-old patient, farmer, former smoker (5 years), COPD and afflicted with rheumatoid arthritis on corticosteroid therapy. He consults the emergency department for presenting intense headache of 2 days of evolution with deviation of the oral commissure. As background, he reports that after a month of influenza, he persists with cough, purulent and occasionally hemoptotic expectoration, febrile fever, anorexia, asthenia and weight loss. On arrival, the patient had a fever of 38.2ºC, multiple skin abscesses on the hands, back and buttocks (some with fistulous tracts) and right central facial paralysis, apical infiltrates with small associated pleural effusion on chest X-ray and leukocytosis with neutrophilia. Among the following suspected diagnoses I would consider MOST likely:", "full_answer": "Nocardia can typically affect immunosuppressed patients, especially those with impaired cell-mediated immunity such as that produced by steroids, and can present with pulmonary involvement, brain abscesses and skin abscesses. Tuberculosis can affect lung and brain but not cutaneous abscesses. Aspergillus also does not affect the skin.", "type": "PNEUMOLOGY AND THORACIC SURGERY", "options": {"1": "Lung neoplasm with brain metastases.", "2": "Disseminated tuberculosis.", "3": "Nocardiosis.", "4": "Aspergillosis.", "5": NaN}, "correct_option": 4, "explanations": {"1": {"exist": false, "char_ranges": [], "word_ranges": [], "text": ""}, "2": {"exist": true, "char_ranges": [[226, 293]], "word_ranges": [[29, 39]], "text": "Tuberculosis can affect lung and brain but not cutaneous abscesses."}, "3": {"exist": true, "char_ranges": [[0, 225]], "word_ranges": [[0, 29]], "text": "Nocardia can typically affect immunosuppressed patients, especially those with impaired cell-mediated immunity such as that produced by steroids, and can present with pulmonary involvement, brain abscesses and skin abscesses."}, "4": {"exist": true, "char_ranges": [[294, 336]], "word_ranges": [[39, 46]], "text": "Aspergillus also does not affect the skin."}, "5": {"exist": false, "char_ranges": [], "word_ranges": [], "text": ""}}} {"id": 392, "year": 2016, "question_id_specific": 223, "full_question": "68-year-old woman, with a history of 2 major depressive episodes in her lifetime, who consults for symptoms of sadness, depressed mood, anhedonia, asthenia and anorexia compatible with a new depressive episode. She was prescribed 10 mg of escitalopram and was evaluated 2 weeks later. In this review the patient reports feeling very well, she wakes up early very hyperactive and with 'a lot of desire to do things', she says she has a lot of energy and is more talkative than usual. She does not report being irritable and is able to sleep for 6 hours continuously. Given this situation, what would you think the patient has?", "full_answer": "In this question, one might suspect the possibility of an affective shift after the introduction of the antidepressant drug, according to the patient, but if we look at the patient's ability to maintain sleep, there is a clear fact that \"rules out\" the hypo/manic shift. Two weeks is not enough time for the drug to have taken full effect on mood, but an initial activation may appear, which patients sometimes do not cope with very well (something that does not seem to be the case). In this patient it is indicated to maintain the treatment and reevaluate in no more than one month.", "type": "PSYCHIATRY", "options": {"1": "Bipolar disorder type I.", "2": "Drug-induced hypomania.", "3": "Normal response to escitalopram.", "4": "Frontal dementia.", "5": NaN}, "correct_option": 3, "explanations": {"1": {"exist": false, "char_ranges": [], "word_ranges": [], "text": ""}, "2": {"exist": true, "char_ranges": [[155, 270]], "word_ranges": [[24, 45]], "text": "if we look at the patient's ability to maintain sleep, there is a clear fact that \"rules out\" the hypo/manic shift."}, "3": {"exist": false, "char_ranges": [], "word_ranges": [], "text": ""}, "4": {"exist": false, "char_ranges": [], "word_ranges": [], "text": ""}, "5": {"exist": false, "char_ranges": [], "word_ranges": [], "text": ""}}} {"id": 285, "year": 2016, "question_id_specific": 59, "full_question": "A 73-year-old woman is admitted with progressive dyspnea until she becomes at rest, orthopnea and weight gain of 4 kg. Physical examination showed blood pressure of 150/84 mm Hg, heart rate 100 beats/minute, increased jugular venous pressure, crepitant in both bases and malleolar edema. Usual treatment: enalapril 5 mg every 12 hours, furosemide 80 mg per day. What is the most appropriate treatment at this time?", "full_answer": "Increase enalapril dose according to tolerance and administer intravenous turosemide.", "type": "CARDIOLOGY AND VASCULAR SURGERY", "options": {"1": "Administer fiirosemide intravenously.", "2": "Increase enalapril dose according to tolerance and administer intravenous furosemide.", "3": "Start a beta-blocker.", "4": "Add treatment with amlodipine.", "5": NaN}, "correct_option": 2, "explanations": {"1": {"exist": false, "char_ranges": [], "word_ranges": [], "text": ""}, "2": {"exist": true, "char_ranges": [[0, 85]], "word_ranges": [[0, 10]], "text": "Increase enalapril dose according to tolerance and administer intravenous turosemide."}, "3": {"exist": false, "char_ranges": [], "word_ranges": [], "text": ""}, "4": {"exist": false, "char_ranges": [], "word_ranges": [], "text": ""}, "5": {"exist": false, "char_ranges": [], "word_ranges": [], "text": ""}}} {"id": 301, "year": 2016, "question_id_specific": 232, "full_question": "A 24-year-old woman consults after noticing inguinal lymphadenopathy. The interrogation does not reveal the presence of any local discomfort or data suggestive of sexually transmitted infection. The examination revealed two lymphadenopathies, one in each groin, 1 cm in diameter, soft, mobile, non-painful. There are no skin lesions on the lower limbs, anus or perineum. Which test do you consider essential?", "full_answer": "In healthy adults there may be palpable inguinal nodes up to 2 centimeters that can be considered normal. A complementary study of these normal lymph nodes is not warranted.", "type": "INFECTIOUS DISEASES", "options": {"1": "A lues serology since it is most likely a Treponema pallidum infection.", "2": "A gynecological examination to rule out ovarian cancer.", "3": "By the clinical characteristics it seems to be normal lymph nodes and complementary explorations should not be done.", "4": "A Paul-Bunnell test should be performed in order to rule out infectious mononucleosis.", "5": NaN}, "correct_option": 3, "explanations": {"1": {"exist": false, "char_ranges": [], "word_ranges": [], "text": ""}, "2": {"exist": false, "char_ranges": [], "word_ranges": [], "text": ""}, "3": {"exist": true, "char_ranges": [[0, 173]], "word_ranges": [[0, 29]], "text": "In healthy adults there may be palpable inguinal nodes up to 2 centimeters that can be considered normal. A complementary study of these normal lymph nodes is not warranted."}, "4": {"exist": false, "char_ranges": [], "word_ranges": [], "text": ""}, "5": {"exist": false, "char_ranges": [], "word_ranges": [], "text": ""}}} {"id": 23, "year": 2011, "question_id_specific": 113, "full_question": "A 71-year-old woman with a history of rheumatoid arthritis on sulfasalazine, prednisone and etanercept. She goes to the emergency room for 72 hours of clinical manifestations compatible with facial herpes zoster affecting the right hemiface, auricular pavilion, respecting the forehead and conjunctival chemosis. What would be the appropriate treatment?", "full_answer": "I think this question is not clearly in the Infectious Diseases syllabus, and may overlap with OFT and DERMA, but as I understand it, in an immunocompromised patient and also with data of ocular involvement, admission for intravenous treatment would be indicated due to the high risk of possible complications.", "type": "INFECTIOUS", "options": {"1": "Symptomatic treatment of pain only.", "2": "Topical treatment with acyclovir.", "3": "Outpatient treatment with acyclovir, valacyclovir or oral famciclovir.", "4": "Hospital admission and treatment with acyclovir or famciclovir iv.", "5": "Parenteral Ig and vaccination."}, "correct_option": 4, "explanations": {"1": {"exist": false, "char_ranges": [], "word_ranges": [], "text": ""}, "2": {"exist": false, "char_ranges": [], "word_ranges": [], "text": ""}, "3": {"exist": false, "char_ranges": [], "word_ranges": [], "text": ""}, "4": {"exist": true, "char_ranges": [[135, 310]], "word_ranges": [[24, 50]], "text": "in an immunocompromised patient and also with data of ocular involvement, admission for intravenous treatment would be indicated due to the high risk of possible complications."}, "5": {"exist": false, "char_ranges": [], "word_ranges": [], "text": ""}}} {"id": 564, "year": 2022, "question_id_specific": 126, "full_question": "A 58-year-old man with a 6-year history of hypertension consults for poor blood pressure control despite treatment with an angiotensin-converting enzyme inhibitor, a diuretic and a calcium antagonist. On consultation she presented with blood pressure of 149/100 mmHg. Laboratory tests: creatinine 1.2 mg/dl, potassium 2.2 mEq/l and compensated metabolic alkalosis; the rest of the biochemical study, blood count, coagulation and urinary sediment were normal. Point out the correct statement:", "full_answer": "We are presented with a patient with resistant arterial hypertension, assuming that he has primary hyperaldosteronism due to hypokalemic metabolic alkalosis. When the diagnosis is biochemically confirmed, the next test to be performed is a CT scan to determine the subtype and rule out the presence of an adrenal carcinoma (option 3 correct). Therefore, it is necessary to wait for the imaging test to confirm the etiology (option 1 incorrect). The most frequent cause is aldosterone-producing adenoma (option 2 incorrect). Spironolactone is the medical treatment of choice (option 4 incorrect).", "type": "NEPHROLOGY", "options": {"1": "The origin of hypertension in this case is excessive secretion of aldosterone caused by autonomic hyperfunction of the adrenal medulla.", "2": "In most cases the anatomical substrate is a bilateral hyperplasia of the adrenal cortex.", "3": "CT scan is part of the diagnostic study in case of biochemical confirmation.", "4": "Spironolactone is contraindicated in the management of this pathology.", "5": NaN}, "correct_option": 3, "explanations": {"1": {"exist": true, "char_ranges": [[343, 444]], "word_ranges": [[53, 70]], "text": "Therefore, it is necessary to wait for the imaging test to confirm the etiology (option 1 incorrect)."}, "2": {"exist": true, "char_ranges": [[445, 523]], "word_ranges": [[70, 80]], "text": "The most frequent cause is aldosterone-producing adenoma (option 2 incorrect)."}, "3": {"exist": true, "char_ranges": [[0, 342]], "word_ranges": [[0, 53]], "text": "We are presented with a patient with resistant arterial hypertension, assuming that he has primary hyperaldosteronism due to hypokalemic metabolic alkalosis. When the diagnosis is biochemically confirmed, the next test to be performed is a CT scan to determine the subtype and rule out the presence of an adrenal carcinoma (option 3 correct)."}, "4": {"exist": true, "char_ranges": [[524, 595]], "word_ranges": [[80, 90]], "text": "Spironolactone is the medical treatment of choice (option 4 incorrect)."}, "5": {"exist": false, "char_ranges": [], "word_ranges": [], "text": ""}}} {"id": 292, "year": 2016, "question_id_specific": 213, "full_question": "Gustavo comes to the emergency room with skin lesions and general malaise of several days of evolution. He has psoriasiform lesions on the trunk with involvement of palms and soles. He also presents asymmetric non-suppurative joint inflammation and bilateral ocular redness as well as erosions on the glans penis. In the subsequent anamnesis Gustavo recognizes a risky sexual contact 20 days before. What is his diagnosis?", "full_answer": "He is describing a textbook Reiter's syndrome: palmo-plantar keratoderma, arthritis and ocular manifestations, together with probably a chlamydial urethritis, perhaps asymptomatic (they do not explain urethral exudate). Of course, the patient may also have HIV infection or even secondary syphilis (in addition to Reiter's).", "type": "DERMATOLOGY, VENEREOLOGY AND PLASTIC SURGERY", "options": {"1": "HIV infection.", "2": "Secondary syphilis.", "3": "Reiter's syndrome.", "4": "Erythema multiforme.", "5": NaN}, "correct_option": 3, "explanations": {"1": {"exist": false, "char_ranges": [], "word_ranges": [], "text": ""}, "2": {"exist": false, "char_ranges": [], "word_ranges": [], "text": ""}, "3": {"exist": true, "char_ranges": [[0, 219]], "word_ranges": [[0, 27]], "text": "He is describing a textbook Reiter's syndrome: palmo-plantar keratoderma, arthritis and ocular manifestations, together with probably a chlamydial urethritis, perhaps asymptomatic (they do not explain urethral exudate)."}, "4": {"exist": false, "char_ranges": [], "word_ranges": [], "text": ""}, "5": {"exist": false, "char_ranges": [], "word_ranges": [], "text": ""}}} {"id": 477, "year": 2020, "question_id_specific": 128, "full_question": "35-year-old man admitted for severe chest trauma with multiple rib fractures. After responding favorably to treatment with analgesics and oxygen, he begins to present severe hypoxemia. Indicate the most probable cause of this deterioration:", "full_answer": "Pulmonary contusion is the most serious lesion and the one with the worst prognosis in thoracic trauma. It is also the injury that causes hypoxemia the earliest. Contrary to what we may think, an unstable thorax due to multiple rib fractures (costal volet) would cause progressive hypoventilation with atelectasis of the pulmonary parenchyma, which would progress on the one hand to hypercapnia and respiratory acidosis, and on the other, to later hypoxemia due to infection associated with atelectasis. Respiratory infection due to aspiration would also occur later, and does not appear in all cases of severe chest trauma, only if there has been a decrease in the level of consciousness (associated TBI, uncontrolled intubation with bronchoaspiration...). Post-traumatic hypovolemia (in this case, as there are multiple rib fractures, it could be due to hemothorax) would occur earlier and would be associated in the first place with hemodynamic instability.", "type": "CRITICAL CARE", "options": {"1": "Chest wall instability due to multiple fractures.", "2": "Aspiration respiratory infection.", "3": "Alteration of gas exchange due to pulmonary contusion.", "4": "Post-traumatic hypovolemia.", "5": NaN}, "correct_option": 3, "explanations": {"1": {"exist": true, "char_ranges": [[193, 503]], "word_ranges": [[33, 78]], "text": "an unstable thorax due to multiple rib fractures (costal volet) would cause progressive hypoventilation with atelectasis of the pulmonary parenchyma, which would progress on the one hand to hypercapnia and respiratory acidosis, and on the other, to later hypoxemia due to infection associated with atelectasis."}, "2": {"exist": true, "char_ranges": [[504, 757]], "word_ranges": [[78, 116]], "text": "Respiratory infection due to aspiration would also occur later, and does not appear in all cases of severe chest trauma, only if there has been a decrease in the level of consciousness (associated TBI, uncontrolled intubation with bronchoaspiration...)."}, "3": {"exist": true, "char_ranges": [[0, 161]], "word_ranges": [[0, 27]], "text": "Pulmonary contusion is the most serious lesion and the one with the worst prognosis in thoracic trauma. It is also the injury that causes hypoxemia the earliest."}, "4": {"exist": true, "char_ranges": [[758, 960]], "word_ranges": [[116, 147]], "text": "Post-traumatic hypovolemia (in this case, as there are multiple rib fractures, it could be due to hemothorax) would occur earlier and would be associated in the first place with hemodynamic instability."}, "5": {"exist": false, "char_ranges": [], "word_ranges": [], "text": ""}}} {"id": 572, "year": 2022, "question_id_specific": 195, "full_question": "A 66-year-old woman with type 2 diabetes mellitus. When assessing her renal function, she presents a G3a/A1 stage. To which values does this stage correspond, the most frequent in patients with diabetic nephropathy?", "full_answer": "Easy question. Stage G3a corresponds to a filtration rate between 45-59 ml/min. Stage A1 corresponds to albuminuria less than 30 mg/ml. Therefore the correct option is 1.", "type": "NEPHROLOGY", "options": {"1": "Glomerular filtration rate 45-59 ml/min/1.73 m² and albuminuria <30 mg/ml.", "2": "Glomerular filtration rate 30-44 ml/min/1.73 m² and albuminuria <30 mg/ml.", "3": "Glomerular filtration rate 45-59 ml/min/1.73 m² and albuminuria 30-300 mg/ml.", "4": "Glomerular filtration rate 15-29 ml/min/1.73 m² and albuminuria <30 mg/ml.", "5": NaN}, "correct_option": 1, "explanations": {"1": {"exist": true, "char_ranges": [[15, 170]], "word_ranges": [[2, 27]], "text": "Stage G3a corresponds to a filtration rate between 45-59 ml/min. Stage A1 corresponds to albuminuria less than 30 mg/ml. Therefore the correct option is 1."}, "2": {"exist": false, "char_ranges": [], "word_ranges": [], "text": ""}, "3": {"exist": false, "char_ranges": [], "word_ranges": [], "text": ""}, "4": {"exist": false, "char_ranges": [], "word_ranges": [], "text": ""}, "5": {"exist": false, "char_ranges": [], "word_ranges": [], "text": ""}}} {"id": 180, "year": 2013, "question_id_specific": 232, "full_question": "A 29-year-old patient comes to your office with a diagnosis of severe bone marrow aplasia. What is the treatment of choice?", "full_answer": "Transfusing this patient and stuffing him with antibiotics is giving him bread for today and hunger for tomorrow. We want something more lasting. Androgens and platelet transfusions don't fix the problem either. We are left with the other three options. An autologous transplant is not reasonable, since the bone marrow of a patient with bone marrow aplasia is less than 25%, so little can be obtained. Therefore, we are inclined towards allogeneic bone marrow transplantation if he has an HLA-identical sibling, since this is the treatment of choice according to the protocol of the Spanish Society of Hematology and Hemotherapy for patients under 40 years of age with severe bone marrow aplasia. Correct answer, 3.", "type": "HEMATOLOGY", "options": {"1": "Periodic transfusions and antibiotics.", "2": "Androgens and platelet transfusions.", "3": "Allogeneic bone marrow transplantation if HLA identical sibling.", "4": "Autologous bone marrow transplantation to avoid rejection.", "5": "Cyclosporin A and antithymocyte globulin."}, "correct_option": 3, "explanations": {"1": {"exist": true, "char_ranges": [[0, 113]], "word_ranges": [[0, 18]], "text": "Transfusing this patient and stuffing him with antibiotics is giving him bread for today and hunger for tomorrow."}, "2": {"exist": true, "char_ranges": [[146, 211]], "word_ranges": [[23, 32]], "text": "Androgens and platelet transfusions don't fix the problem either."}, "3": {"exist": true, "char_ranges": [[414, 697]], "word_ranges": [[67, 112]], "text": "we are inclined towards allogeneic bone marrow transplantation if he has an HLA-identical sibling, since this is the treatment of choice according to the protocol of the Spanish Society of Hematology and Hemotherapy for patients under 40 years of age with severe bone marrow aplasia."}, "4": {"exist": true, "char_ranges": [[254, 402]], "word_ranges": [[40, 66]], "text": "An autologous transplant is not reasonable, since the bone marrow of a patient with bone marrow aplasia is less than 25%, so little can be obtained."}, "5": {"exist": false, "char_ranges": [], "word_ranges": [], "text": ""}}} {"id": 270, "year": 2015, "question_id_specific": 135, "full_question": "A young man comes to the emergency room with a second-degree flame burn of 10% of the body surface, affecting the right arm in an extensive and circular manner. There is no arterial pulse in the hand measured by Doppler. What is the treatment of choice?", "full_answer": "The question is tricky: it speaks of a burn of 10% of the body surface and the treatment of choice for second degree burns less than 20% of the BSA should be topical and monitored. However, there is a warning sign: there is no arterial pulse and the burn is circumferential to the limb. This is an emergency that, if allowed to evolve, will compromise the viability of the affected limb and end in amputation: an emergency escharotomy must be performed in order to relieve the pressure of the third space on the arterial vascular trunk.", "type": "DERMATOLOGY AND PLASTIC SURGERY", "options": {"1": "Occlusive sulfadiazine-arginine cures and depth assessment at one week.", "2": "Lymphatic drainage and assess a vascular by-pass.", "3": "Escharotomy or emergency decompression incisions.", "4": "Expectant management.", "5": "Amputation of the extremity."}, "correct_option": 3, "explanations": {"1": {"exist": false, "char_ranges": [], "word_ranges": [], "text": ""}, "2": {"exist": false, "char_ranges": [], "word_ranges": [], "text": ""}, "3": {"exist": true, "char_ranges": [[190, 536]], "word_ranges": [[36, 96]], "text": "there is a warning sign: there is no arterial pulse and the burn is circumferential to the limb. This is an emergency that, if allowed to evolve, will compromise the viability of the affected limb and end in amputation: an emergency escharotomy must be performed in order to relieve the pressure of the third space on the arterial vascular trunk."}, "4": {"exist": false, "char_ranges": [], "word_ranges": [], "text": ""}, "5": {"exist": false, "char_ranges": [], "word_ranges": [], "text": ""}}} {"id": 368, "year": 2016, "question_id_specific": 121, "full_question": "While you are on call in the Emergency Department of your hospital, you have to attend a 64-year-old patient with acute respiratory failure. His clinical condition is critical, with low oxygen saturation and hemodynamic instability. An urgent chest X-ray was performed showing atelectasis of 2/3 of the right lung. Orotracheal intubation and assisted ventilation were performed, with Fi02 of 1.0. Subsequent arterial blood gas analysis showed pH 7.23, Pa02 60 mmHg and PaC02 30 mmHg. What was the cause of the hypoxemia?", "full_answer": "FiO2 of 1 and hyperventilation with mechanical ventilation, demonstrated by the low PCO2, fail to sufficiently raise PO2, due to a V/Q disturbance, with probable shunt in the area of pulmonary atelectasis, probably acute, since the lung has not been able to create compensatory mechanisms to limit perfusion in the poorly ventilated areas.", "type": "PNEUMOLOGY AND THORACIC SURGERY", "options": {"1": "Short circuit.", "2": "Hypoventilation.", "3": "Low inspired 02 pressure.", "4": "Neuromuscular disease.", "5": NaN}, "correct_option": 1, "explanations": {"1": {"exist": true, "char_ranges": [[0, 339]], "word_ranges": [[0, 53]], "text": "FiO2 of 1 and hyperventilation with mechanical ventilation, demonstrated by the low PCO2, fail to sufficiently raise PO2, due to a V/Q disturbance, with probable shunt in the area of pulmonary atelectasis, probably acute, since the lung has not been able to create compensatory mechanisms to limit perfusion in the poorly ventilated areas."}, "2": {"exist": false, "char_ranges": [], "word_ranges": [], "text": ""}, "3": {"exist": false, "char_ranges": [], "word_ranges": [], "text": ""}, "4": {"exist": false, "char_ranges": [], "word_ranges": [], "text": ""}, "5": {"exist": false, "char_ranges": [], "word_ranges": [], "text": ""}}} {"id": 167, "year": 2013, "question_id_specific": 85, "full_question": "If in a patient with chronic heart failure we detect prominent v waves in the jugular venous pulse and on cardiac auscultation a holosystolic murmur is auscultated in the area of the xiphoid appendage that is accentuated with deep inspiration. What is the valvulopathy responsible for this physical examination?", "full_answer": "Basic semiology. However, emergency reasoning: systolic murmur, something that in systole had to be closed and is not, or had to open, and does not: options 1, 3 and 5. And if we see repercussion in the jugular pulse, it must be in the right cavities. Apart from the fact that the v wave appears during systole, while the atria are filling: if the RV flow rises to the RA, what will happen is that the v wave will be very large?", "type": "CARDIOLOGY AND CARDIOVASCULAR SURGERY", "options": {"1": "Mitral insufficiency.", "2": "Pulmonary insufficiency.", "3": "Tricuspid insufficiency.", "4": "Aortic insufficiency.", "5": "Aortic stenosis."}, "correct_option": 3, "explanations": {"1": {"exist": false, "char_ranges": [], "word_ranges": [], "text": ""}, "2": {"exist": false, "char_ranges": [], "word_ranges": [], "text": ""}, "3": {"exist": true, "char_ranges": [[173, 428]], "word_ranges": [[31, 82]], "text": "if we see repercussion in the jugular pulse, it must be in the right cavities. Apart from the fact that the v wave appears during systole, while the atria are filling: if the RV flow rises to the RA, what will happen is that the v wave will be very large?"}, "4": {"exist": false, "char_ranges": [], "word_ranges": [], "text": ""}, "5": {"exist": false, "char_ranges": [], "word_ranges": [], "text": ""}}} {"id": 154, "year": 2012, "question_id_specific": 75, "full_question": "A 42-year-old female patient reports pain with inflammatory features and swelling in both wrists, 2nd and 3rd metacarpophalangeal and proximal interphalangeal joints bilaterally and left ankle of 4 months of evolution accompanied by morning stiffness of more than one hour duration. Hand X-ray shows an erosion in the styloid process of the ulna in the right carpus. Laboratory tests showed Hb: 10 g/dL with ESR of 45 mm in the first hour, CRP 16 mg/L, rheumatoid factor 160 IU/ML. After 6 months of treatment with indomethacin and methotrexate, the patient persists with pain and swelling of both carpals, morning stiffness lasting 30 minutes and a CBC showing an ESR 30 mm in the first hour and a CRP 9 mg/dL. Regarding the attitude to take, which of the following is true:", "full_answer": "This is a patient with rheumatoid arthritis. In my opinion, as in these six months she has shown a clear clinical and biological improvement (decrease in acute phase reactants), I would maintain the therapeutic approach taken and wait for a new evaluation. However, I consider that this question could have another valid answer, 3. It depends a little on the attitude of each rheumatologist. If what you are looking for is a remission of the disease as soon as possible, you could choose to consider associating an anti-TNF alpha?", "type": "RHEUMATOLOGY", "options": {"1": "Suspend the prescribed treatment due to lack of response and initiate prednisone at high doses for symptom control only.", "2": "Maintain the therapeutic attitude taken since we have only been on it for 6 months and it would be necessary to wait a minimum of 9 months to evaluate therapeutic response.", "3": "If there is no medical contraindication, consider adding an anti-TNF alpha to the treatment.", "4": "Start a second disease-modifying drug as soon as possible, since it would not be possible to start treatment with biologic therapy alone after methotrexate.", "5": "Consider starting treatment with anti-CD20 therapy associated with methotrexate."}, "correct_option": 2, "explanations": {"1": {"exist": false, "char_ranges": [], "word_ranges": [], "text": ""}, "2": {"exist": true, "char_ranges": [[60, 256]], "word_ranges": [[10, 42]], "text": "as in these six months she has shown a clear clinical and biological improvement (decrease in acute phase reactants), I would maintain the therapeutic approach taken and wait for a new evaluation."}, "3": {"exist": true, "char_ranges": [[392, 530]], "word_ranges": [[64, 89]], "text": "If what you are looking for is a remission of the disease as soon as possible, you could choose to consider associating an anti-TNF alpha?"}, "4": {"exist": false, "char_ranges": [], "word_ranges": [], "text": ""}, "5": {"exist": false, "char_ranges": [], "word_ranges": [], "text": ""}}} {"id": 176, "year": 2013, "question_id_specific": 57, "full_question": "A 56-year-old woman with a history of well-controlled schizophrenia and no toxic habits. Admitted for middle lobe pneumonia with a small associated metaneumonic pleural effusion and on treatment with levofloxacin 500 mg/24h . She presented good clinical evolution except for persistent febrile fever and leukocytosis on the sixth day of treatment. Microbiological studies are not available. The most appropriate course of action is:", "full_answer": "As long as fever persists in an evolving pneumonia, there is a risk of empyematization, especially if a small effusion has already occurred at the beginning of the picture. In principle, antibiotic treatment is correct and before considering changes in treatment, the possibility of empyematization should be evaluated.", "type": "PNEUMOLOGY", "options": {"1": "The evolution is normal, treatment should be maintained until completing 10 days.", "2": "It is considered a therapeutic failure and antibiotic treatment should be modified.", "3": "Perform thoracentesis to rule out empyema.", "4": "Add corticosteroids at a dose of 0.5 mg/Kg/day to antibiotic treatment.", "5": "Bronchoscopy with biopsy, aspiration and bronchoalveolar lavage."}, "correct_option": 3, "explanations": {"1": {"exist": false, "char_ranges": [], "word_ranges": [], "text": ""}, "2": {"exist": false, "char_ranges": [], "word_ranges": [], "text": ""}, "3": {"exist": true, "char_ranges": [[173, 319]], "word_ranges": [[29, 48]], "text": "In principle, antibiotic treatment is correct and before considering changes in treatment, the possibility of empyematization should be evaluated."}, "4": {"exist": false, "char_ranges": [], "word_ranges": [], "text": ""}, "5": {"exist": false, "char_ranges": [], "word_ranges": [], "text": ""}}} {"id": 130, "year": 2012, "question_id_specific": 82, "full_question": "An 18-year-old female patient with a history of absences between 6 and 9 years of age, generalized tonic-clonic seizures of recent onset and violent jumping of the upper limbs at breakfast. The clinical manifestations worsen with nighttime weekend outings. An EEG shows acute polyp spike discharges at 6 cycles/second. The most likely diagnosis is:", "full_answer": "A patient of adolescent age with myoclonus (\"violent jumps\") at breakfast time will almost always lead us to juvenile myoclonic epilepsy (answer 5 correct), a very well characterized entity. Among other features is the previous history of seizures or absences, the worsening with nighttime departures, and the EEG with generalized polyp spike discharges at a higher frequency than that of absences. Of the remaining answers, Lennox Gastaut develops at an early age with other symptomatology, 4 is the same as absences, and 3 usually presents with complex partial seizures and either way, the EEG is clearly distinct from a temporal focus.", "type": "NEUROLOGY AND NEUROSURGERY", "options": {"1": "Great epileptic disease.", "2": "Lennox-Gastaut syndrome.", "3": "Symptomatic epilepsy due to mesial temporal sclerosis.", "4": "Small atypical disease.", "5": "Juvenile myoclonic epilepsy."}, "correct_option": 5, "explanations": {"1": {"exist": false, "char_ranges": [], "word_ranges": [], "text": ""}, "2": {"exist": true, "char_ranges": [[425, 491]], "word_ranges": [[65, 75]], "text": "Lennox Gastaut develops at an early age with other symptomatology,"}, "3": {"exist": true, "char_ranges": [[523, 571]], "word_ranges": [[82, 89]], "text": "3 usually presents with complex partial seizures"}, "4": {"exist": true, "char_ranges": [[492, 518]], "word_ranges": [[75, 81]], "text": "4 is the same as absences,"}, "5": {"exist": true, "char_ranges": [[0, 156]], "word_ranges": [[0, 24]], "text": "A patient of adolescent age with myoclonus (\"violent jumps\") at breakfast time will almost always lead us to juvenile myoclonic epilepsy (answer 5 correct),"}}} {"id": 276, "year": 2016, "question_id_specific": 75, "full_question": "Indicate the clinical situation in relation to hepatitis B virus infection in a 5-year-old patient from Nigeria, with normal physical examination and the following serology for hepatitis B: HBsAg + / ANTI-HBs - / HbeAg - / ANTI-HBe + / ANTI-HBc IgM - / ANTI-HBc IgG + / DNA HBV +:", "full_answer": "Having antibodies against the core implies natural contact, and that it is IgG, which is not acute. The absence of e antigen rules out active replication. And the persistence of surface antigen (HBsAg) and virus DNA indicates that it is still present. Together, these indicate an asymptomatic carrier. Carriers of the pre-core mutant usually have recurrent symptomatic periods and increased liver inflammation. There would be insufficient data to rule it out completely, but it is not among the answers and indicates a completely normal physical examination.", "type": "DIGESTIVE SYSTEM", "options": {"1": "Acute infection.", "2": "Chronic infection.", "3": "Vaccinated patient.", "4": "Asymptomatic carrier.", "5": NaN}, "correct_option": 4, "explanations": {"1": {"exist": false, "char_ranges": [], "word_ranges": [], "text": ""}, "2": {"exist": false, "char_ranges": [], "word_ranges": [], "text": ""}, "3": {"exist": false, "char_ranges": [], "word_ranges": [], "text": ""}, "4": {"exist": true, "char_ranges": [[0, 301]], "word_ranges": [[0, 48]], "text": "Having antibodies against the core implies natural contact, and that it is IgG, which is not acute. The absence of e antigen rules out active replication. And the persistence of surface antigen (HBsAg) and virus DNA indicates that it is still present. Together, these indicate an asymptomatic carrier."}, "5": {"exist": false, "char_ranges": [], "word_ranges": [], "text": ""}}} {"id": 183, "year": 2013, "question_id_specific": 64, "full_question": "A patient on enteral nutritional support presents 72 hours after starting enteral nutrition with a CBC showing hypophosphoremia and hypokalemia, with clinical signs of heart failure. The patient is diagnosed with refeeding syndrome. Indicate which of the following is NOT considered a risk factor for a patient presenting with this condition:", "full_answer": "Refeeding syndrome occurs in patients with previous malnutrition exposed to either oral, enteral or parenteral nutritional therapy. All responses are causes of malnutrition except 3.", "type": "ENDOCRINOLOGY", "options": {"1": "Previous caloric malnutrition.", "2": "Anorexia nervosa.", "3": "Non-morbid obesity.", "4": "Elderly.", "5": "Prolonged vomiting and diarrhea."}, "correct_option": 3, "explanations": {"1": {"exist": true, "char_ranges": [[0, 182]], "word_ranges": [[0, 25]], "text": "Refeeding syndrome occurs in patients with previous malnutrition exposed to either oral, enteral or parenteral nutritional therapy. All responses are causes of malnutrition except 3."}, "2": {"exist": true, "char_ranges": [[0, 182]], "word_ranges": [[0, 25]], "text": "Refeeding syndrome occurs in patients with previous malnutrition exposed to either oral, enteral or parenteral nutritional therapy. All responses are causes of malnutrition except 3."}, "3": {"exist": true, "char_ranges": [[0, 182]], "word_ranges": [[0, 25]], "text": "Refeeding syndrome occurs in patients with previous malnutrition exposed to either oral, enteral or parenteral nutritional therapy. All responses are causes of malnutrition except 3."}, "4": {"exist": true, "char_ranges": [[0, 182]], "word_ranges": [[0, 25]], "text": "Refeeding syndrome occurs in patients with previous malnutrition exposed to either oral, enteral or parenteral nutritional therapy. All responses are causes of malnutrition except 3."}, "5": {"exist": true, "char_ranges": [[0, 182]], "word_ranges": [[0, 25]], "text": "Refeeding syndrome occurs in patients with previous malnutrition exposed to either oral, enteral or parenteral nutritional therapy. All responses are causes of malnutrition except 3."}}} {"id": 115, "year": 2012, "question_id_specific": 98, "full_question": "A 32-year-old woman with cerebral palsy from childbirth comes to the emergency department for a few days of dark urine associated with an episode of high fever and dry cough. On admission, the CBC showed 16900 leukocytes/mm3 (85% S, 11% L, 4% M), hemoglobin 6.3 g/dL; MCV 109 fl, 360000 platelets/mm3. In the biochemistry LDH 2408; bilirubin 6.8 mg/dl, (unconjugated bilirubin 6.1 mg/dl), normal GOT and GPT. The morphological study of blood showed macrocytic anisocytosis with frequent spherocytic forms and polychromatophilia without blasts. The irregular antibody study is positive for panagglutinin, making crossmatching difficult. What would be your suspicion and the most appropriate treatment?", "full_answer": "A long statement full of rather confusing data. We can attack that question in two ways: by looking at the key data and going all the way to the diagnosis we are considering or by discarding one by one. The choluria, LDH, presence of panagglutinin and the peripheral blood smear \"smells\" like hemolytic anemia. Answer 1 cannot be because a marrow aplasia does not explain the choluria, the elevation of LDH, nor in the study of irregular antibodies is positive in the form of panagglutinin; an aplasia is a marrow failure characterized by a total or partial disappearance of hemopoietic progenitors. In addition, pancytopenia is not observed, which is what would incline us more towards this pathology. Answer 2 is not possible either, since a spherocytosis does not justify the presence of panagglutinin. Answer 3 is the one I consider correct: an autoimmune hemolytic anemia would justify the data given: elevated LDH and bilirubinemia due to red cell destruction, polychromatophilia, spherocytosis and anisocytosis because the marrow is working hard to try to compensate for the anemia, which is regenerative. The study of irregular antibodies and the presence of panagglutinin also supports this response, since the binding of an antibody to the hematocyte promotes its lysis and destruction. The girl presents cough and fever, consistent with a respiratory infection. And the initial treatment is corticosteroids. This is confirmed by Sans Sabrafens in his book \"Clinical Hematology\" [1]. Answer 4 gives rise to considerable doubt; it is not possible because a pernicious anemia does not justify the presence of panagglutinin although it would justify the elevation of LDH and bilirubin; moreover, it is an intramedullary, arregenerative hemolysis, there is no reticulocytosis or release into the blood of immature forms in an attempt to compensate and fix the situation. Answer 5 is not true because no blasts are seen in the blood nor does it explain the presence of panagglutinin.", "type": "HEMATOLOGY", "options": {"1": "Medullary aplasia and immunotherapy with thymoglobulin and cyclosporine.", "2": "Hereditary spherocytosis and splenectomy.", "3": "Autoimmune hemolytic anemia associated with respiratory infection and corticosteroids.", "4": "Pernicious anemia and periodic injections of vitamin B12.", "5": "Acute leukemia and chemotherapy."}, "correct_option": 3, "explanations": {"1": {"exist": true, "char_ranges": [[311, 490]], "word_ranges": [[54, 85]], "text": "Answer 1 cannot be because a marrow aplasia does not explain the choluria, the elevation of LDH, nor in the study of irregular antibodies is positive in the form of panagglutinin;"}, "2": {"exist": true, "char_ranges": [[703, 805]], "word_ranges": [[117, 133]], "text": "Answer 2 is not possible either, since a spherocytosis does not justify the presence of panagglutinin."}, "3": {"exist": true, "char_ranges": [[806, 1112]], "word_ranges": [[133, 179]], "text": "Answer 3 is the one I consider correct: an autoimmune hemolytic anemia would justify the data given: elevated LDH and bilirubinemia due to red cell destruction, polychromatophilia, spherocytosis and anisocytosis because the marrow is working hard to try to compensate for the anemia, which is regenerative."}, "4": {"exist": true, "char_ranges": [[1494, 1876]], "word_ranges": [[236, 296]], "text": "Answer 4 gives rise to considerable doubt; it is not possible because a pernicious anemia does not justify the presence of panagglutinin although it would justify the elevation of LDH and bilirubin; moreover, it is an intramedullary, arregenerative hemolysis, there is no reticulocytosis or release into the blood of immature forms in an attempt to compensate and fix the situation."}, "5": {"exist": true, "char_ranges": [[1877, 1988]], "word_ranges": [[296, 317]], "text": "Answer 5 is not true because no blasts are seen in the blood nor does it explain the presence of panagglutinin."}}} {"id": 75, "year": 2012, "question_id_specific": 65, "full_question": "A patient admitted for acute pancreatitis starts with tachypnea, tachycardia, sweating and progressive cyanosis. PaO2 is 55 mm Hg (PaO2/FiO2 ratio<200). CXR shows bilateral alveolar pulmonary infiltrates. Pulmonary capillary wedge pressure is normal. Oxygen therapy does not improve the situation. What is the most probable diagnosis:", "full_answer": "Direct response question. The question tells the story of a patient with acute pancreatitis who is developing acute respiratory failure. The statement lists the criteria for ARDS (acute respiratory distress syndrome): PaO2/fiO2 < 200 + bilateral pulmonary infiltrates + PCP<18. So the diagnosis is respiratory distress and therefore the correct answer is option 5.", "type": "ANESTHESIOLOGY, CRITICAL CARE AND EMERGENCIES", "options": {"1": "Nosocomial pneumonia.", "2": "Cardiac failure.", "3": "Carcinomatous lymphangitis.", "4": "Pulmonary thromboembolism.", "5": "Respiratory distress."}, "correct_option": 5, "explanations": {"1": {"exist": false, "char_ranges": [], "word_ranges": [], "text": ""}, "2": {"exist": false, "char_ranges": [], "word_ranges": [], "text": ""}, "3": {"exist": false, "char_ranges": [], "word_ranges": [], "text": ""}, "4": {"exist": false, "char_ranges": [], "word_ranges": [], "text": ""}, "5": {"exist": true, "char_ranges": [[137, 364]], "word_ranges": [[20, 54]], "text": "The statement lists the criteria for ARDS (acute respiratory distress syndrome): PaO2/fiO2 < 200 + bilateral pulmonary infiltrates + PCP<18. So the diagnosis is respiratory distress and therefore the correct answer is option 5."}}} {"id": 191, "year": 2013, "question_id_specific": 125, "full_question": "A 13-month-old infant comes to the emergency department with fever up to 39ºC of 48h of evolution with no other associated symptoms. Examination by organs and devices with no significant findings, highlighting good general condition. You were going to discharge him home but the Pediatrics attending on duty asks for a systematic urine and urine culture by catheterization. The urine shows leukocyturia ++, hematuria + and nitrites ++ and the urine Gram-negative bacilli are observed. In the blood analysis there is no leukocytosis and the C-reactive protein is 50 mg/l. The attending now tells you that the child does not need to be admitted and to prescribe an oral antibiotic. State the least appropriate empirical antibiotic treatment in this case:", "full_answer": "Other drugs are a good choice in urinary tract infections caused by large negative bacilli (most likely: E. coli). The presence of leukocytes + nitrites in the urine is very suggestive of UTI. And if the urine Gram detects G- bacteria, it is probably a UTI due to E Coli. Many E Coli are resistant to amoxicillin.", "type": "PEDIATRICS", "options": {"1": "Amoxicillin.", "2": "Amoxicillin-clavulanic acid.", "3": "Cefuroxime-axetil.", "4": "Cotrimoxazole.", "5": "Cefixime."}, "correct_option": 1, "explanations": {"1": {"exist": true, "char_ranges": [[115, 313]], "word_ranges": [[19, 57]], "text": "The presence of leukocytes + nitrites in the urine is very suggestive of UTI. And if the urine Gram detects G- bacteria, it is probably a UTI due to E Coli. Many E Coli are resistant to amoxicillin."}, "2": {"exist": true, "char_ranges": [[0, 90]], "word_ranges": [[0, 15]], "text": "Other drugs are a good choice in urinary tract infections caused by large negative bacilli"}, "3": {"exist": true, "char_ranges": [[0, 90]], "word_ranges": [[0, 15]], "text": "Other drugs are a good choice in urinary tract infections caused by large negative bacilli"}, "4": {"exist": true, "char_ranges": [[0, 90]], "word_ranges": [[0, 15]], "text": "Other drugs are a good choice in urinary tract infections caused by large negative bacilli"}, "5": {"exist": true, "char_ranges": [[0, 90]], "word_ranges": [[0, 15]], "text": "Other drugs are a good choice in urinary tract infections caused by large negative bacilli"}}} {"id": 323, "year": 2016, "question_id_specific": 150, "full_question": "12-month-old boy, who in the health examinations practiced since birth presents right testicle in inguinal canal that is not possible to descend to the scrotum. Mark the CORRECT answer:", "full_answer": "This is cryptorchidism. It has to be treated before the age of 2 years, and hormonal treatment is currently in disuse. The treatment of first choice is orchidopexy.", "type": "UROLOGY", "options": {"1": "The most likely diagnosis is retractile testis.", "2": "Wait until two years of age for spontaneous decrease of the synovial fluid.", "3": "Human chorionic gonadotropin is the treatment of first choice.", "4": "The indication for orchidopexy should not be deferred.", "5": NaN}, "correct_option": 4, "explanations": {"1": {"exist": false, "char_ranges": [], "word_ranges": [], "text": ""}, "2": {"exist": false, "char_ranges": [], "word_ranges": [], "text": ""}, "3": {"exist": false, "char_ranges": [], "word_ranges": [], "text": ""}, "4": {"exist": true, "char_ranges": [[0, 164]], "word_ranges": [[0, 28]], "text": "This is cryptorchidism. It has to be treated before the age of 2 years, and hormonal treatment is currently in disuse. The treatment of first choice is orchidopexy."}, "5": {"exist": false, "char_ranges": [], "word_ranges": [], "text": ""}}} {"id": 311, "year": 2016, "question_id_specific": 217, "full_question": "An 84-year-old woman presents with loss of vision in the left eye of 4 days of evolution accompanied by metamorphopsia. The macula shows abundant hard exudates, two small deep hemorrhages and a localized neurosensory retinal detachment. In the contralateral eye there are abundant soft drusen. Which of the following diagnoses do you think is the most likely?", "full_answer": "None of the other three options give that clinical picture (DPV is usually asymptomatic, CRVO produces abrupt and total or near total loss of vision with the typical \"cherry red spot\" in the fundus, and both an arteritic and non-arteritic NOIA would produce papillary edema usually with peripapillary but not macular hemorrhages, and none of these 3 options have any association with drusen in the contralateral eye). AMD in its exudative form presents with both this clinical presentation (loss of vision and metamorphopsia) and the characteristic fundus described in the question, in addition to the fact that drusen are usually observed in the contralateral eye.", "type": "OPHTHALMOLOGY", "options": {"1": "Acute posterior vitreous detachment.", "2": "Exudative age-related macular degeneration (AMD).", "3": "Central retinal artery obstruction.", "4": "Non-arteritic anterior ischemic optic neuropathy.", "5": NaN}, "correct_option": 2, "explanations": {"1": {"exist": true, "char_ranges": [[334, 417]], "word_ranges": [[53, 67]], "text": "none of these 3 options have any association with drusen in the contralateral eye)."}, "2": {"exist": true, "char_ranges": [[418, 665]], "word_ranges": [[67, 105]], "text": "AMD in its exudative form presents with both this clinical presentation (loss of vision and metamorphopsia) and the characteristic fundus described in the question, in addition to the fact that drusen are usually observed in the contralateral eye."}, "3": {"exist": true, "char_ranges": [[334, 417]], "word_ranges": [[53, 67]], "text": "none of these 3 options have any association with drusen in the contralateral eye)."}, "4": {"exist": true, "char_ranges": [[334, 417]], "word_ranges": [[53, 67]], "text": "none of these 3 options have any association with drusen in the contralateral eye)."}, "5": {"exist": false, "char_ranges": [], "word_ranges": [], "text": ""}}} {"id": 137, "year": 2012, "question_id_specific": 164, "full_question": "Given a direct trauma to the right side of the face after which the patient presents unilateral palpebral hematoma, diplopia in the vertical gaze and difficulty in opening the mouth, which of the following statements is true?", "full_answer": "Diplopia on vertical gaze is highly suggestive that the bony walls of the orbit are affected. Lefort type I fracture does not affect the orbital contents. Knowing this information, the only correct answer is 2. If we were in doubt about whether or not Lefort's type I fracture affects the orbit, just look at the answers. Whoever posted the question wanted to make sure that the only valid answer was number 2; that is why they added the tag line \"but just in case we check it with a CT scan\".", "type": "OTORHINOLARYNGOLOGY AND MAXILLOFACIAL SURGERY", "options": {"1": "We are with great probability in front of a Lefort I type fracture of the middle third of the face.", "2": "This is probably a unilateral orbitomalar fracture. The diagnosis would ideally be verified by CT (computed axial tomography).", "3": "It is a fracture of the base of the skull at the level of the carotid foramen.", "4": "The probable diagnosis is fracture with dislocation of the mandibular condyle.", "5": "A mandibular fracture is probably associated with a Lefort I type midface fracture."}, "correct_option": 2, "explanations": {"1": {"exist": true, "char_ranges": [[94, 154]], "word_ranges": [[16, 26]], "text": "Lefort type I fracture does not affect the orbital contents."}, "2": {"exist": true, "char_ranges": [[0, 93]], "word_ranges": [[0, 16]], "text": "Diplopia on vertical gaze is highly suggestive that the bony walls of the orbit are affected."}, "3": {"exist": false, "char_ranges": [], "word_ranges": [], "text": ""}, "4": {"exist": false, "char_ranges": [], "word_ranges": [], "text": ""}, "5": {"exist": false, "char_ranges": [], "word_ranges": [], "text": ""}}} {"id": 309, "year": 2016, "question_id_specific": 215, "full_question": "A 47-year-old man with myopia magna, who underwent cataract surgery 2 years ago, comes to the emergency room reporting a profound and painless loss of vision in his right eye. Which of the following diagnoses can cause this symptomatology?", "full_answer": "This question could be controversial. Option 1 is strongly discarded because endophthalmitis typically presents with a lot of pain. Option 3 of wet form of age-related macular degeneration is also ruled out, because it usually occurs in older patients, although what a patient with myopia magna may present with is myopic macular degeneration (with a similar clinical picture to AMD, but this is a different pathology). A posterior vitreous detachment (PVD) is usually not very symptomatic, producing myodesopsias but no loss of vision or pain; however, some PVDs are of the hemorrhagic type, by traction of the vessels, producing a vitreous hemorrhage that would cause loss of vision. In any case, due to the history of myopia magna and previous intraocular surgery, the first diagnosis to rule out would be retinal detachment, since these are two risk factors for this pathology. Technically, both option 2 and 4 (an acute hemorrhagic DPV) could cause these symptoms, but the most correct option is option 2 (and I am sure it is the one the examiner wants answered).", "type": "OPHTHALMOLOGY", "options": {"1": "Post-surgical endophthalmitis.", "2": "Retinal detachment.", "3": "Age-related macular degeneration, wet form.", "4": "Posterior vitreous detachment.", "5": NaN}, "correct_option": 2, "explanations": {"1": {"exist": true, "char_ranges": [[38, 131]], "word_ranges": [[5, 19]], "text": "Option 1 is strongly discarded because endophthalmitis typically presents with a lot of pain."}, "2": {"exist": true, "char_ranges": [[882, 1068]], "word_ranges": [[141, 175]], "text": "Technically, both option 2 and 4 (an acute hemorrhagic DPV) could cause these symptoms, but the most correct option is option 2 (and I am sure it is the one the examiner wants answered)."}, "3": {"exist": true, "char_ranges": [[132, 252]], "word_ranges": [[19, 39]], "text": "Option 3 of wet form of age-related macular degeneration is also ruled out, because it usually occurs in older patients,"}, "4": {"exist": true, "char_ranges": [[882, 1068]], "word_ranges": [[141, 175]], "text": "Technically, both option 2 and 4 (an acute hemorrhagic DPV) could cause these symptoms, but the most correct option is option 2 (and I am sure it is the one the examiner wants answered)."}, "5": {"exist": false, "char_ranges": [], "word_ranges": [], "text": ""}}} {"id": 279, "year": 2016, "question_id_specific": 32, "full_question": "A 59-year-old woman presenting with chronic watery diarrhea of 4 months' evolution. In the endoscopy, the mucosa did not show relevant aspects. In particular, no ulcers or friable areas were observed. A biopsy of the transverse colon was performed. Histopathology revealed a thickened area below the superficial lining epithelium, which was more evident by Masson's trichrome technique and involved epithelial atrophy and denudation. There was also a clear increase in intraepithelial lymphocyte density. The diagnosis of the intestinal lesion is?", "full_answer": "In this case, the presence of thickening of the subepithelial collagen layer, more evident with Masson's trichrome (special stain that allows differentiation of the collagen fibers), is pathognomonic of collagenous colitis. Atrophy together with epithelial denudation are characteristic of this disease, which does not usually cause macroscopic alterations of the mucosa. An increased density of intraepithelial lymphocytes and the absence of mention of architectural alterations of the crypts (characteristic of ulcerative colitis and Crohn's disease) support this diagnosis.", "type": "PATHOLOGICAL ANATOMY", "options": {"1": "Chronic ulcerative colitis.", "2": "Pseudomembranous colitis.", "3": "Collagenous colitis.", "4": "Fibrosing Crohn's disease.", "5": NaN}, "correct_option": 3, "explanations": {"1": {"exist": false, "char_ranges": [], "word_ranges": [], "text": ""}, "2": {"exist": false, "char_ranges": [], "word_ranges": [], "text": ""}, "3": {"exist": true, "char_ranges": [[14, 223]], "word_ranges": [[3, 31]], "text": "the presence of thickening of the subepithelial collagen layer, more evident with Masson's trichrome (special stain that allows differentiation of the collagen fibers), is pathognomonic of collagenous colitis."}, "4": {"exist": false, "char_ranges": [], "word_ranges": [], "text": ""}, "5": {"exist": false, "char_ranges": [], "word_ranges": [], "text": ""}}} {"id": 437, "year": 2018, "question_id_specific": 118, "full_question": "Luis is a 25-year-old young man who underwent splenectomy after a bicycle accident 1 year ago. He has a dog that bit him 24 hours ago and has caused a small wound on his right hand. He went to his health center (located 3 hours from the nearest hospital) for fever of 39ºC, pain in the wound and general malaise. On examination, BP 100/60 mm Hg, HR 110 beats per minute, slight swelling in the wound without pus. Which of the following actions is most indicated at this time?", "full_answer": "Since a splenectomized patient is considered a \"special\" patient, he should receive antibiotic treatment as soon as possible, even if the wound does not appear infected. It is necessary to make sure that the dog is yours (you should ask if the animal is correctly vaccinated) and if the patient is correctly vaccinated (5 doses of tetanus, it would not require tetanus); make a notification of Animal Aggression, for observation of this and take measures in case this is necessary.", "type": "EPIDEMIOLOGY AND PREVENTIVE MEDICINE", "options": {"1": "Send to hospital for rabies and tetanus vaccination and keep under observation.", "2": "Clean the wound and administer intramuscular nonspecific ganunaglobulin.", "3": "Give 400 mg of oral moxifloxacin and send to the hospital.", "4": "Give clindamycin 600 mg oral every 8 hours and observation.", "5": NaN}, "correct_option": 3, "explanations": {"1": {"exist": false, "char_ranges": [], "word_ranges": [], "text": ""}, "2": {"exist": false, "char_ranges": [], "word_ranges": [], "text": ""}, "3": {"exist": true, "char_ranges": [[0, 169]], "word_ranges": [[0, 26]], "text": "Since a splenectomized patient is considered a \"special\" patient, he should receive antibiotic treatment as soon as possible, even if the wound does not appear infected."}, "4": {"exist": false, "char_ranges": [], "word_ranges": [], "text": ""}, "5": {"exist": false, "char_ranges": [], "word_ranges": [], "text": ""}}} {"id": 112, "year": 2012, "question_id_specific": 154, "full_question": "A 67-year-old patient with a history of menopause at 55 years of age, 3 pregnancies with 3 euthyroid deliveries, type 2 diabetes of 6 years of evolution, treatment with nifedipone for hypertension. She consulted for intermittent scanty metrorrhagia of 2 months of evolution. The gynecological examination showed external genitalia without lesions, an atrophic cervix, a normal uterus and appendages on palpation and a normal cytological study of the cervix. The transvaginal ultrasound study shows a 7 mm hyperechogenic endometrium. Which of the following tests is the most appropriate and most sensitive to establish a diagnosis?", "full_answer": "The correct answer is 3. The study that gives us more information about endometrial pathology is hysteroscopy, which allows us to perform a directed endometrial biopsy. Endometrial cytology is performed blindly and has a large number of false negatives. MRI will help us to stage endometrial cancer.", "type": "GYNECOLOGY AND OBSTETRICS", "options": {"1": "Conization of the cervix.", "2": "Endometrial cytology.", "3": "Hysteroscopy and endometrial biopsy.", "4": "Magnetic resonance imaging of the pelvis.", "5": "Examination under anesthesia of the genital tract and biopsy of the cervix and endometrium."}, "correct_option": 3, "explanations": {"1": {"exist": false, "char_ranges": [], "word_ranges": [], "text": ""}, "2": {"exist": true, "char_ranges": [[169, 253]], "word_ranges": [[26, 39]], "text": "Endometrial cytology is performed blindly and has a large number of false negatives."}, "3": {"exist": true, "char_ranges": [[25, 168]], "word_ranges": [[5, 26]], "text": "The study that gives us more information about endometrial pathology is hysteroscopy, which allows us to perform a directed endometrial biopsy."}, "4": {"exist": true, "char_ranges": [[254, 299]], "word_ranges": [[39, 47]], "text": "MRI will help us to stage endometrial cancer."}, "5": {"exist": false, "char_ranges": [], "word_ranges": [], "text": ""}}} {"id": 374, "year": 2016, "question_id_specific": 101, "full_question": "A 52-year-old man comes to the Emergency Department with headache and fever (37.8°C) of 2 days' evolution. In the last few hours, he also had difficulty in nomination and comprehension. The examination did not show nuchal rigidity, the most striking finding being the presence of a mixed aphasia. The cranial CT shows a faint hypodensity in the left temporal lobe without mass effect and without contrast uptake. Which of the following statements is correct?", "full_answer": "The picture described is typical of a viral meningoencephalitis, more specifically VHZ, with significant tropism by the temporal lobe, causing febrile fever and in many cases aphasia, so the correct option would be 2, since the cerebrospinal fluid normally shows a moderate lymphocytic pleocytosis, with normal glycorrhachia. The rest are not correct because: The first one because it is not a bacterial meningitis that would present with more fever, the third one because limbic encephalitis usually presents afebrile with behavioral changes and the last one because the image they describe is not an abscess, well delimited.", "type": "NEUROLOGY", "options": {"1": "Bacterial meningitis is the first diagnostic impression and treatment with 3rd generation cephalosporin should be initiated as soon as possible.", "2": "Most likely this patient's CSF shows a lymphocyte-predominant pleocytosis with normal glycorrhachia.", "3": "We would suspect limbic encephalitis.", "4": "This is an early stage brain abscess.", "5": NaN}, "correct_option": 2, "explanations": {"1": {"exist": true, "char_ranges": [[360, 450]], "word_ranges": [[53, 69]], "text": "The first one because it is not a bacterial meningitis that would present with more fever,"}, "2": {"exist": true, "char_ranges": [[0, 325]], "word_ranges": [[0, 47]], "text": "The picture described is typical of a viral meningoencephalitis, more specifically VHZ, with significant tropism by the temporal lobe, causing febrile fever and in many cases aphasia, so the correct option would be 2, since the cerebrospinal fluid normally shows a moderate lymphocytic pleocytosis, with normal glycorrhachia."}, "3": {"exist": true, "char_ranges": [[451, 542]], "word_ranges": [[69, 81]], "text": "the third one because limbic encephalitis usually presents afebrile with behavioral changes"}, "4": {"exist": true, "char_ranges": [[547, 626]], "word_ranges": [[82, 96]], "text": "the last one because the image they describe is not an abscess, well delimited."}, "5": {"exist": false, "char_ranges": [], "word_ranges": [], "text": ""}}} {"id": 116, "year": 2012, "question_id_specific": 101, "full_question": "A 25-year-old woman who wishes to become pregnant and wants to know what treatment she should take during the eventual pregnancy, as she is a heterozygous factor V Leiden carrier. She has never had any thrombotic phenomena. The determination of this factor was performed as a family study after an episode of pulmonary embolism in a sibling. What treatment should be advised?", "full_answer": "Reading the answers, the first thing that is clear to us is that acenocoumarol should NOT be given to a pregnant woman because of the risk of teratogenicity. Anyone who does so during residency should be punished with 100 slaps on the back. We discard answer 5. Second: if treatment is given, it should be during the entire pregnancy, prothrombotic status is not only during the puerperium, so answer 2 discarded. Advise a woman not to become pregnant when there are treatments that ensure a very low risk of thrombosis and/or fetal involvement? Answer 1 discarded. We are left with the possibility of not giving any treatment because it is low risk or administering low-dose aspirin. It is true that factor V Leiden in heterozygosis is classified as low risk, who has not presented a previous thrombotic episode nor is there a combination with another thrombophilia; if we look at the CHEST guide, VIII edition and review what was said at the last congress of the Spanish Society of Gynecology and Obstetrics, we see that aspirin is used in case of phospholipid syndrome and that in the case of this woman, without previous thrombosis, without previous abortions (at least two are needed) and also being heterozygous, treatment is not necessary. Correct answer, 3.", "type": "HEMATOLOGY", "options": {"1": "Since pregnancy is a prothrombotic state, there would be a high risk of venous thromboembolism, so pregnancy should be discouraged.", "2": "Treatment with low molecular weight heparin at prophylactic doses should be carried out in the immediate puerperium, with optional follow-up during pregnancy.", "3": "Factor V Leiden in heterozygosis is a low-risk thrombophilia and there is no need for any treatment in pregnancy and puerperium.", "4": "Low-dose aspirin should be advised during pregnancy and puerperium.", "5": "Treatment with antivitamin K drugs (acenocoumarol) during pregnancy."}, "correct_option": 3, "explanations": {"1": {"exist": true, "char_ranges": [[414, 545]], "word_ranges": [[71, 93]], "text": "Advise a woman not to become pregnant when there are treatments that ensure a very low risk of thrombosis and/or fetal involvement?"}, "2": {"exist": true, "char_ranges": [[270, 390]], "word_ranges": [[48, 67]], "text": "if treatment is given, it should be during the entire pregnancy, prothrombotic status is not only during the puerperium,"}, "3": {"exist": true, "char_ranges": [[1011, 1246]], "word_ranges": [[172, 210]], "text": "we see that aspirin is used in case of phospholipid syndrome and that in the case of this woman, without previous thrombosis, without previous abortions (at least two are needed) and also being heterozygous, treatment is not necessary."}, "4": {"exist": true, "char_ranges": [[1011, 1246]], "word_ranges": [[172, 210]], "text": "we see that aspirin is used in case of phospholipid syndrome and that in the case of this woman, without previous thrombosis, without previous abortions (at least two are needed) and also being heterozygous, treatment is not necessary."}, "5": {"exist": true, "char_ranges": [[65, 157]], "word_ranges": [[13, 28]], "text": "acenocoumarol should NOT be given to a pregnant woman because of the risk of teratogenicity."}}} {"id": 473, "year": 2020, "question_id_specific": 141, "full_question": "The most likely diagnosis of a 74-year-old patient who since two months ago begins with lumbar pain radiating to lower limbs, neurogenic claudication and limitation to extension the trunk is:", "full_answer": "We are being shown a picture of lumbar canal stenosis (Answer 4 correct). The patient shows the characteristic clinical picture: lumbar pain that is relieved by flexing the trunk forward, radicular pain with lower limb involvement and neurogenic claudication. Low back pain in a patient over 60 years of age accompanied by leg weakness when walking, which has to be interrupted at a certain distance, called neurogenic claudication, is the typical picture of stenosis. The patient is relieved by flexing the spine forward because it widens the lumbar canal and worsens with extension...", "type": "ORTHOPEDIC SURGERY AND TRAUMATOLOGY", "options": {"1": "L4-L5 disc herniation.", "2": "Lumbar vertebral fracture.", "3": "L5-S1 vertebral instability.", "4": "Lumbar canal stenosis.", "5": NaN}, "correct_option": 4, "explanations": {"1": {"exist": false, "char_ranges": [], "word_ranges": [], "text": ""}, "2": {"exist": false, "char_ranges": [], "word_ranges": [], "text": ""}, "3": {"exist": false, "char_ranges": [], "word_ranges": [], "text": ""}, "4": {"exist": true, "char_ranges": [[0, 259]], "word_ranges": [[0, 39]], "text": "We are being shown a picture of lumbar canal stenosis (Answer 4 correct). The patient shows the characteristic clinical picture: lumbar pain that is relieved by flexing the trunk forward, radicular pain with lower limb involvement and neurogenic claudication."}, "5": {"exist": false, "char_ranges": [], "word_ranges": [], "text": ""}}} {"id": 34, "year": 2011, "question_id_specific": 66, "full_question": "A 51-year-old woman comes to the emergency department with a sudden decrease in visual acuity, severe headache, nausea and vomiting. Hypotensive and afebrile. She presented right ophthalmoparesis due to involvement of the third cranial nerve. A cranial CT scan shows a mass in the hyperdense selar region with erosion of the anterior clinoid processes. What is the best approach to follow?", "full_answer": "Pituitary apoplexy is a syndrome characterized by the sudden onset of headache accompanied by ocular motility disturbances and a variable degree of pituitary insufficiency. The cause is usually a rapidly growing pituitary mass secondary to a vascular event (infarction or hemorrhage). General symptoms may range from vomiting or nausea to meningeal irritation. Compression on the chiasm and optic nerve can cause various campimetric deficits (usually greater bitemporal hemianopsia in the superior quadrants) and decreased visual acuity, even blindness. If the mass grows lateral to the sella turcica it can compress the oculomotor nerves, with the common motor ocular nerve being the most frequently affected, because of its more medial course in the cavernous sinus. The test of choice for the diagnosis of pituitary apoplexy is MRI, as CT may not distinguish the region clearly enough to distinguish degenerative or cystic changes from previous bleedings. The finding of erosion of the anterior clinoid processes in the skull CT may support a chronic lesion, which as a result of an acute vascular phenomenon has caused the symptoms. The management of these patients involves strict control of water and electrolyte disturbances and correction of hormonal deficits. In unstable situations as in the case described, with significant visual disturbances, surgical intervention is usually of choice, with transsphenoidal decompression.", "type": "NEUROLOGY AND NEUROSURGERY", "options": {"1": "Suspect chemical meningitis derived from a ruptured epidermoid tumor and start immediate treatment with corticosteroids.", "2": "It would indicate the performance of a cerebral angiography to rule out an aneurysm, since it is most likely that we are facing a case of subarachnoid hemorrhage and the mass that is evident in the CT is a thrombosed parasellar aneurysm.", "3": "Admission to ICU and treatment of the shock suffered by the patient and once stabilized perform brain MRI for scheduled surgery.", "4": "Urgent biochemistry and hemogram, initiation of high-dose corticosteroid therapy and urgent transsphenoidal surgery.", "5": "Lumbar puncture to rule out bacterial meningitis after starting empirical antibiotherapy. Once the patient was stabilized, study of the selar mass."}, "correct_option": 5, "explanations": {"1": {"exist": false, "char_ranges": [], "word_ranges": [], "text": ""}, "2": {"exist": false, "char_ranges": [], "word_ranges": [], "text": ""}, "3": {"exist": false, "char_ranges": [], "word_ranges": [], "text": ""}, "4": {"exist": false, "char_ranges": [], "word_ranges": [], "text": ""}, "5": {"exist": true, "char_ranges": [[1269, 1435]], "word_ranges": [[193, 214]], "text": "In unstable situations as in the case described, with significant visual disturbances, surgical intervention is usually of choice, with transsphenoidal decompression."}}} {"id": 343, "year": 2016, "question_id_specific": 158, "full_question": "A 24-year-old woman, primigestation, suffers a spontaneous abortion at 7 weeks gestation. The anatomopathological study of the abortive remains indicates molar disease. We should inform you that:", "full_answer": "The answer is 2. According to the SEGO (Spanish Society of Gynecology and Obstetrics) in its 2005 protocol \"Gestational trophoblastic disease\", the risk of malignization, i.e. of developing a gestational trophoblastic neoplasm, ranges from 5 to 20% depending on whether it is a partial mole or a complete mole, respectively. According to this protocol: \"Patients will be monitored weekly with hcg dosage until it becomes undetectable, for three consecutive times. After that the monitoring will be monthly, for six months and then every two months for another six months. Based on the available data, a follow-up of three to six months is recommended for partial mole and 12 months for complete mole.\" Therefore, subsequent controls are strictly necessary.The risk of a new molar gestation, although increased compared to the normal population, is not 50%.", "type": "GYNECOLOGY AND OBSTETRICS", "options": {"1": "The risk of a new molar gestation in a future pregnancy is 50%.", "2": "You should not become pregnant until periodic controls and have spent one year with negative BHCG levels.", "3": "Subsequent controls are not necessary if the evacuation of the trophoblastic tissue was complete.", "4": "It is necessary to perform periodic controls since 40% of the cases will develop a gestational trophoblastic neoplasia.", "5": NaN}, "correct_option": 2, "explanations": {"1": {"exist": false, "char_ranges": [], "word_ranges": [], "text": ""}, "2": {"exist": true, "char_ranges": [[144, 571]], "word_ranges": [[21, 89]], "text": "the risk of malignization, i.e. of developing a gestational trophoblastic neoplasm, ranges from 5 to 20% depending on whether it is a partial mole or a complete mole, respectively. According to this protocol: \"Patients will be monitored weekly with hcg dosage until it becomes undetectable, for three consecutive times. After that the monitoring will be monthly, for six months and then every two months for another six months."}, "3": {"exist": false, "char_ranges": [], "word_ranges": [], "text": ""}, "4": {"exist": false, "char_ranges": [], "word_ranges": [], "text": ""}, "5": {"exist": false, "char_ranges": [], "word_ranges": [], "text": ""}}} {"id": 210, "year": 2014, "question_id_specific": 187, "full_question": "A 27-year-old woman, 10 weeks pregnant, with persistent severe allergic asthma. She is currently adequately controlled with daily inhaled budesonide and rescue inhaled salbutamol on demand. She comes to your office concerned about the possible teratogenic effects of her anti-asthma medication. Which of the following would be the correct approach?", "full_answer": "#ERROR!", "type": "GYNECOLOGY AND OBSTETRICS", "options": {"1": "Given that asthma improves during pregnancy in most patients, the best thing for the patient and the fetus is to suspend anti-asthma treatment.", "2": "Discontinue budesonide because it has been associated with an increased risk of fetal malformations and replace it with an oral anti-leukotriene (montelukast).", "3": "Withdraw current treatment and replace it with oral prednisone at the lowest possible dose.", "4": "Maintain the current treatment and reassure the patient about its side effects and the need for adequate asthma control during pregnancy.", "5": "Replace budesonide with an anti-IgE monoclonal antibody (omalizumab) because of its greater safety in pregnancy since it is not a drug."}, "correct_option": 4, "explanations": {"1": {"exist": false, "char_ranges": [], "word_ranges": [], "text": ""}, "2": {"exist": false, "char_ranges": [], "word_ranges": [], "text": ""}, "3": {"exist": false, "char_ranges": [], "word_ranges": [], "text": ""}, "4": {"exist": false, "char_ranges": [], "word_ranges": [], "text": ""}, "5": {"exist": false, "char_ranges": [], "word_ranges": [], "text": ""}}} {"id": 187, "year": 2013, "question_id_specific": 68, "full_question": "A 52-year-old woman from a village on the Costa Brava notices an otherwise asymptomatic lump in the anterior region when applying cream to her neck; she goes to her general practitioner who confirms the presence of a firm, smooth mass, 2 cm in maximum diameter, which rises with swallowing. No palpable lymphadenopathy. What tests would you order at the outset?", "full_answer": "It refers to a thyroid nodule, and the indicated test is a FNA (fine needle puncture). I have read the answer Emilio has written on the challenge; the question is in the endocrine block and it seems he is trying to make it clear that it is a nodule. Ultrasound facilitates FNA but is not mandatory, a 2 cm thyroid nodule can be punctured without echo. It is important to have an analysis of thyroid function prior to the puncture; but since he is talking about an asymptomatic patient, it seems clear that she is not hyperthyroid. I do not think it is contested and I think it is a question that most will have answered well.", "type": "ENDOCRINOLOGY", "options": {"1": "A determination of thyroglobulin in blood.", "2": "A cervical CT scan.", "3": "A determination of circulating antithyroid antibodies (antithyroglubulin and antiperoxidase).", "4": "A fine needle puncture.", "5": "A determination of free T3."}, "correct_option": 4, "explanations": {"1": {"exist": false, "char_ranges": [], "word_ranges": [], "text": ""}, "2": {"exist": false, "char_ranges": [], "word_ranges": [], "text": ""}, "3": {"exist": false, "char_ranges": [], "word_ranges": [], "text": ""}, "4": {"exist": true, "char_ranges": [[0, 86]], "word_ranges": [[0, 16]], "text": "It refers to a thyroid nodule, and the indicated test is a FNA (fine needle puncture)."}, "5": {"exist": false, "char_ranges": [], "word_ranges": [], "text": ""}}} {"id": 406, "year": 2016, "question_id_specific": 33, "full_question": "A 67-year-old woman diagnosed with an infiltrating ductal carcinoma of the breast with no family history of neoplasia. What additional studies should be performed on the tumor because of its clinical and therapeutic implications:", "full_answer": "Currently the molecular classification of breast cancer is based on the study of hormone receptors, HER2 and the tumor cell proliferation index (Ki67). We have four fundamental types: Luminal (A: HR+, HER2- and low Ki67 and B: HR+, HER2+/ - and high Ki67), HER2 + (HR -, HER2 + and high Ki67) and Basal Like or triple negative (HR -, HER2 - and high Ki67).", "type": "ONCOLOGY (ECTOPIC)", "options": {"1": "Complete phenotypic study by flow cytometry.", "2": "Study of hormone receptors and HER2.", "3": "Study of hormone receptors, ecadherin and study of first degree relatives.", "4": "BRCA l-2 study and study of first-degree relatives.", "5": NaN}, "correct_option": 2, "explanations": {"1": {"exist": false, "char_ranges": [], "word_ranges": [], "text": ""}, "2": {"exist": true, "char_ranges": [[0, 151]], "word_ranges": [[0, 23]], "text": "Currently the molecular classification of breast cancer is based on the study of hormone receptors, HER2 and the tumor cell proliferation index (Ki67)."}, "3": {"exist": false, "char_ranges": [], "word_ranges": [], "text": ""}, "4": {"exist": false, "char_ranges": [], "word_ranges": [], "text": ""}, "5": {"exist": false, "char_ranges": [], "word_ranges": [], "text": ""}}} {"id": 526, "year": 2021, "question_id_specific": 54, "full_question": "An 86-year-old woman in whom nonvalvular atrial fibrillation has been detected. She has a CHADS2 score of 3 points. In the literature, similar patients on warfarin therapy have a stroke risk of 2.2% versus 5.2% in patients without warfarin. What would be the number needed to treat (NNT) to prevent embolic stroke with anticoagulation therapy?", "full_answer": "The NNT (number needed to treat), means how many patients we would need to treat with an intervention to obtain a benefit; in this question how many patients do we need to treat with warfarin to prevent a stroke. To calculate this we use the following formula: NNT= 1/RRA (RRA: absolute risk reduction). RRA= Io - Ie (Io: incidence in those not exposed to warfarin; Ie: incidence in those exposed to warfarin). RRA: 5,2%- 2,2%= 3= 0,03. NNT: 1/0.03= 33.3.", "type": "PREVENTIVE MEDICINE", "options": {"1": "3", "2": "19,2.", "3": "33,3.", "4": "49,5.", "5": NaN}, "correct_option": 3, "explanations": {"1": {"exist": false, "char_ranges": [], "word_ranges": [], "text": ""}, "2": {"exist": false, "char_ranges": [], "word_ranges": [], "text": ""}, "3": {"exist": true, "char_ranges": [[0, 455]], "word_ranges": [[0, 80]], "text": "The NNT (number needed to treat), means how many patients we would need to treat with an intervention to obtain a benefit; in this question how many patients do we need to treat with warfarin to prevent a stroke. To calculate this we use the following formula: NNT= 1/RRA (RRA: absolute risk reduction). RRA= Io - Ie (Io: incidence in those not exposed to warfarin; Ie: incidence in those exposed to warfarin). RRA: 5,2%- 2,2%= 3= 0,03. NNT: 1/0.03= 33.3."}, "4": {"exist": false, "char_ranges": [], "word_ranges": [], "text": ""}, "5": {"exist": false, "char_ranges": [], "word_ranges": [], "text": ""}}} {"id": 6, "year": 2011, "question_id_specific": 45, "full_question": "A 30-year-old man with a family history of a father who died at 38 years of age from colon cancer. A colonoscopy is performed and shows hundreds of adenomas throughout the colon. Which of the following statements is false?", "full_answer": "Patient with familial adenomatous polyposis. All statements are correct except 2. Treatment should be surgical when polyposis is observed. At 40 years of age is the usual point of incidence of cancer.", "type": "DIGESTIVE", "options": {"1": "The patient has familial adenomatous polyposis.", "2": "The most appropriate management is annual follow-up colonoscopy and colectomy at age 40.", "3": "If the patient does not undergo surgical treatment, he/she will almost certainly develop colorectal cancer.", "4": "First degree relatives should be studied.", "5": "The patient's children have a 50% risk of suffering the same disease."}, "correct_option": 2, "explanations": {"1": {"exist": true, "char_ranges": [[0, 200]], "word_ranges": [[0, 32]], "text": "Patient with familial adenomatous polyposis. All statements are correct except 2. Treatment should be surgical when polyposis is observed. At 40 years of age is the usual point of incidence of cancer."}, "2": {"exist": true, "char_ranges": [[0, 200]], "word_ranges": [[0, 32]], "text": "Patient with familial adenomatous polyposis. All statements are correct except 2. Treatment should be surgical when polyposis is observed. At 40 years of age is the usual point of incidence of cancer."}, "3": {"exist": true, "char_ranges": [[0, 200]], "word_ranges": [[0, 32]], "text": "Patient with familial adenomatous polyposis. All statements are correct except 2. Treatment should be surgical when polyposis is observed. At 40 years of age is the usual point of incidence of cancer."}, "4": {"exist": true, "char_ranges": [[0, 200]], "word_ranges": [[0, 32]], "text": "Patient with familial adenomatous polyposis. All statements are correct except 2. Treatment should be surgical when polyposis is observed. At 40 years of age is the usual point of incidence of cancer."}, "5": {"exist": true, "char_ranges": [[0, 200]], "word_ranges": [[0, 32]], "text": "Patient with familial adenomatous polyposis. All statements are correct except 2. Treatment should be surgical when polyposis is observed. At 40 years of age is the usual point of incidence of cancer."}}} {"id": 573, "year": 2022, "question_id_specific": 92, "full_question": "A 75-year-old woman brought for consultation by her family because they have been finding her depressed and with memory lapses for months. They are concerned that she may have Alzheimer's disease. The patient refers that she does not think anything is wrong with her and that she is as usual. In what order do the following symptoms generally occur in the progression of Alzheimer's disease:", "full_answer": "I answer this question thinking more about what they want to ask than what they are really asking. There is heterogeneity in the development of Alzheimer's disease symptoms. There are behavioral variants, but in the MIR they usually want you to think of other types of dementias in the face of behavioral symptoms. In fact, the appearance of these symptoms in early stages should alert us and open the range of diagnostic possibilities. Therefore, we discard 2 and 4. Between 1 and 3, I choose 3 for the same reason and because of the emphasis on the decline of functional independence that occurs in these patients.", "type": "NEUROLOGY", "options": {"1": "Mood changes, behavioral symptoms, cognitive deficits.", "2": "Behavioral symptoms, motor symptoms, decline of functional independence.", "3": "Mood changes, cognitive deficit, decline of functional independence.", "4": "Behavioral symptoms, mood changes, motor symptoms.", "5": NaN}, "correct_option": 3, "explanations": {"1": {"exist": true, "char_ranges": [[468, 616]], "word_ranges": [[79, 106]], "text": "Between 1 and 3, I choose 3 for the same reason and because of the emphasis on the decline of functional independence that occurs in these patients."}, "2": {"exist": true, "char_ranges": [[174, 467]], "word_ranges": [[28, 79]], "text": "There are behavioral variants, but in the MIR they usually want you to think of other types of dementias in the face of behavioral symptoms. In fact, the appearance of these symptoms in early stages should alert us and open the range of diagnostic possibilities. Therefore, we discard 2 and 4."}, "3": {"exist": true, "char_ranges": [[468, 616]], "word_ranges": [[79, 106]], "text": "Between 1 and 3, I choose 3 for the same reason and because of the emphasis on the decline of functional independence that occurs in these patients."}, "4": {"exist": true, "char_ranges": [[174, 467]], "word_ranges": [[28, 79]], "text": "There are behavioral variants, but in the MIR they usually want you to think of other types of dementias in the face of behavioral symptoms. In fact, the appearance of these symptoms in early stages should alert us and open the range of diagnostic possibilities. Therefore, we discard 2 and 4."}, "5": {"exist": false, "char_ranges": [], "word_ranges": [], "text": ""}}} {"id": 164, "year": 2013, "question_id_specific": 123, "full_question": "An 80-year-old patient with a history of hypertension and on treatment with enalapril and spironolactone comes to the hospital with asthenia and severe muscle weakness. Blood pressure is 110/70 mmHg. In the ECG, there are sharp and elevated T waves, ventricular extrasystoles and short QT. What is the most likely diagnosis?", "full_answer": "The correct answer is: 2. Hyperkalemia. Spironolactone, as a potassium-sparing diuretic, inhibits the action of aldosterone at the renal level, being responsible for a decrease in potassium excretion. Hyperkalemia produces repolarization disturbances that result in the installation of a large, symmetrical, narrow-based T, visible in the precordial leads. The QT space is shortened. These anomalies appear with a kalemia of around 5.5 to 6 mmol/l. Above 6.5 mmol/L, electrocardiographic changes are constant, and are dominated by conduction disturbances (making ventricular extrasystoles possible).", "type": "NEPHROLOGY", "options": {"1": "Hypercalcemia.", "2": "Hyperkalemia.", "3": "Hypomagnesemia.", "4": "Hypocalcemia.", "5": "Hypernatremia."}, "correct_option": 2, "explanations": {"1": {"exist": false, "char_ranges": [], "word_ranges": [], "text": ""}, "2": {"exist": true, "char_ranges": [[201, 356]], "word_ranges": [[28, 48]], "text": "Hyperkalemia produces repolarization disturbances that result in the installation of a large, symmetrical, narrow-based T, visible in the precordial leads."}, "3": {"exist": false, "char_ranges": [], "word_ranges": [], "text": ""}, "4": {"exist": false, "char_ranges": [], "word_ranges": [], "text": ""}, "5": {"exist": false, "char_ranges": [], "word_ranges": [], "text": ""}}} {"id": 208, "year": 2014, "question_id_specific": 183, "full_question": "Woman 40 weeks gestation in labor with 6 cm dilation. She presents a decelerative fetal pattern in cardiotocographic recordings, so it is decided to perform a fetal blood smear to assess fetal well-being. Result 7.22. The correct conduct is:", "full_answer": "- pH less than 7.20 Pathological value. Indication for fetal extraction by the quickest route, in this case a cesarean section. - pH between 7.20-7.25 Prepathological value. Repeat microtome in 15-20 min. - pH greater than 7.25. Value within the limits of normality. Observation.", "type": "GYNECOLOGY AND OBSTETRICS", "options": {"1": "Severe acidosis. Urgent cesarean section.", "2": "Pre-pathological value repeat microtome 15-20 minutes.", "3": "Moderate acidosis. Repeat microtome in 1-2 hours.", "4": "Value in normal limits, leave natural evolution of labor.", "5": "Repeat at the time of possible error in obtaining the shot."}, "correct_option": 2, "explanations": {"1": {"exist": false, "char_ranges": [], "word_ranges": [], "text": ""}, "2": {"exist": true, "char_ranges": [[130, 204]], "word_ranges": [[22, 32]], "text": "pH between 7.20-7.25 Prepathological value. Repeat microtome in 15-20 min."}, "3": {"exist": false, "char_ranges": [], "word_ranges": [], "text": ""}, "4": {"exist": false, "char_ranges": [], "word_ranges": [], "text": ""}, "5": {"exist": false, "char_ranges": [], "word_ranges": [], "text": ""}}} {"id": 232, "year": 2014, "question_id_specific": 103, "full_question": "A 19-year-old girl with no medical history of interest, except for a self-limited case of influenza 3 weeks earlier, who comes to the emergency department with petechiae and ecchymosis of spontaneous onset. On physical examination the patient was in good general condition, afebrile, normotensive and oriented in time and space. There were petechiae scattered throughout the lower extremities and abdomen and small ecchymoses in decubitus areas. No lymphadenopathy or splenomegaly were palpated. Laboratory tests showed the following findings: Hb 12.6 g/dL, leukocytes 5,500/mm3, platelets 7000/mm3. The peripheral blood smear showed normal erythrocyte morphology, normal differential leukocyte count and the planetary count was concordant with the autoanalyzer figure without platelet aggregates. Biochemistry and proteinogram, beta 2 microglobulin and LDH are normal. Which of the following do you think is the most appropriate initial treatment?", "full_answer": "Idiopathic thrombocytopenic purpura, but with very low platelets, so replenishment is necessary. The proposed steroid dose is too low.", "type": "HEMATOLOGY", "options": {"1": "Platelet transfusion.", "2": "Weekly Rituximab.", "3": "Cyclophosphamide in 4-day pulses every 21 days.", "4": "Daily plasmapheresis.", "5": "Prednisone at 1 mg/day for 2-3 weeks."}, "correct_option": 1, "explanations": {"1": {"exist": true, "char_ranges": [[0, 96]], "word_ranges": [[0, 12]], "text": "Idiopathic thrombocytopenic purpura, but with very low platelets, so replenishment is necessary."}, "2": {"exist": false, "char_ranges": [], "word_ranges": [], "text": ""}, "3": {"exist": false, "char_ranges": [], "word_ranges": [], "text": ""}, "4": {"exist": false, "char_ranges": [], "word_ranges": [], "text": ""}, "5": {"exist": true, "char_ranges": [[97, 134]], "word_ranges": [[12, 19]], "text": "The proposed steroid dose is too low."}}} {"id": 64, "year": 2011, "question_id_specific": 210, "full_question": "A 42-year-old woman with breast cancer has metastases at the level of the intervertebral foramina between the 4th and 5th cervical vertebrae and between the 4th and 5th thoracic vertebrae. Which spinal nerves will be injured?", "full_answer": "The trick to this question is that there are 8 cervical roots for 7 vertebrae, therefore the C1 root exits ABOVE the atlas, and so on up to C8 which exits BETWEEN C7 and T1. The thoracic roots come out respectively below the vertebra with their same numbering (nerve T1 under T1, T2 under T2, etc.). Therefore, between C4 and C5 the 5th cervical nerve exits, and between T4 and T5 the 4th thoracic nerve exits. The correct answer is 4.", "type": "ANATOMY", "options": {"1": "Fourth cervical and fourth thoracic nerves.", "2": "Fifth cervical and fifth thoracic nerves.", "3": "Fourth cervical and fifth thoracic nerves.", "4": "Fifth cervical and fourth thoracic nerve.", "5": "Third cervical and fourth thoracic nerve."}, "correct_option": 4, "explanations": {"1": {"exist": false, "char_ranges": [], "word_ranges": [], "text": ""}, "2": {"exist": false, "char_ranges": [], "word_ranges": [], "text": ""}, "3": {"exist": false, "char_ranges": [], "word_ranges": [], "text": ""}, "4": {"exist": false, "char_ranges": [], "word_ranges": [], "text": ""}, "5": {"exist": false, "char_ranges": [], "word_ranges": [], "text": ""}}} {"id": 280, "year": 2016, "question_id_specific": 32, "full_question": "A 59-year-old woman presents with chronic watery diarrhea of 4 months' duration. On endoscopy, the mucosa showed no relevant features. In particular, no ulcers or friable areas were observed. A biopsy of the transverse colon was performed. Histopathology revealed a thickened area below the superficial lining epithelium, which was more evident by Masson's trichrome technique and involved epithelial atrophy and denudation. There was also a clear increase in intraepithelial lymphocyte density. The diagnosis of the intestinal lesion is?", "full_answer": "Everything we are told in this case is typical of COLLAGENOUS COLITIS. In principle, the clinical and endoscopic findings may leave us a little as at the beginning, but histology is little more than the definition of this pathology. The specific criterion for the diagnosis of collagenous colitis is the additional presence of an irregular band of collagen just below the surface epithelium of the colonic mucosa in continuity with the basement membrane, visible with trichrome staining (stains type I collagen fibers), which traps superficial capillaries producing lesions in the surface epithelium. In addition, it is accompanied by a chronic inflammatory infiltrate in the lamina propria, composed mainly of lymphocytes, plasma cells and eosinophils.", "type": "PATHOLOGICAL ANATOMY", "options": {"1": "Chronic ulcerative colitis.", "2": "Pseudomembranous colitis.", "3": "Collagenous colitis.", "4": "Fibrosing Crohn's disease.", "5": NaN}, "correct_option": 3, "explanations": {"1": {"exist": false, "char_ranges": [], "word_ranges": [], "text": ""}, "2": {"exist": false, "char_ranges": [], "word_ranges": [], "text": ""}, "3": {"exist": true, "char_ranges": [[233, 600]], "word_ranges": [[39, 92]], "text": "The specific criterion for the diagnosis of collagenous colitis is the additional presence of an irregular band of collagen just below the surface epithelium of the colonic mucosa in continuity with the basement membrane, visible with trichrome staining (stains type I collagen fibers), which traps superficial capillaries producing lesions in the surface epithelium."}, "4": {"exist": false, "char_ranges": [], "word_ranges": [], "text": ""}, "5": {"exist": false, "char_ranges": [], "word_ranges": [], "text": ""}}} {"id": 545, "year": 2022, "question_id_specific": 122, "full_question": "A 78-year-old patient with no past history who consults for asthenia and dyspnea of 3 months of evolution, which has progressed to rest in the last few days. In the previous days he also refers chest pain with anginal characteristics with small efforts. On examination, blood pressure of 110/80 mmHg, heart rate of 85 bpm and auscultation with a rough systolic murmur in the right second intercostal space and crackles in both lung bases. Which of the following is the most likely diagnosis?", "full_answer": "Aortic stenosis.", "type": "CARDIOLOGY", "options": {"1": "Mitral insufficiency.", "2": "Aortic stenosis.", "3": "Aortic insufficiency.", "4": "Dilated cardiomyopathy of ischemic origin.", "5": NaN}, "correct_option": 2, "explanations": {"1": {"exist": false, "char_ranges": [], "word_ranges": [], "text": ""}, "2": {"exist": true, "char_ranges": [[0, 16]], "word_ranges": [[0, 2]], "text": "Aortic stenosis."}, "3": {"exist": false, "char_ranges": [], "word_ranges": [], "text": ""}, "4": {"exist": false, "char_ranges": [], "word_ranges": [], "text": ""}, "5": {"exist": false, "char_ranges": [], "word_ranges": [], "text": ""}}} {"id": 257, "year": 2014, "question_id_specific": 95, "full_question": "In a 30-year-old female patient a calcium level of 11 mg/dl (normal less than 10.5 mg/dl) is found during a routine company examination. PTH determination was 45 pg/ml (VN 10-55 pg/ml). The history is unremarkable except for the fact that the mother and paternal grandfather were diagnosed with hyperparathyroidism and underwent surgery, although they remained hypercalcemic. Which test is most useful in confirming the diagnosis?", "full_answer": "Calcium/creatinine urine ratio. This is familial hypocalciuric hypercalcemia or benign familial hypercalcemia. Autosomal dominant disorder. Asymptomatic. Not cured after surgical treatment; it is due to mutation of the calcium-sensitive receptor. Difficult question since it is necessary to know data of a disease that is not usually asked in the MIR.", "type": "ENDOCRINOLOGY", "options": {"1": "25-OH D.", "2": "1,25-OH 2D.", "3": "Urine calcium/creatinine ratio.", "4": "Phosphate tubular reabsorption.", "5": "PTHrP."}, "correct_option": 3, "explanations": {"1": {"exist": false, "char_ranges": [], "word_ranges": [], "text": ""}, "2": {"exist": false, "char_ranges": [], "word_ranges": [], "text": ""}, "3": {"exist": true, "char_ranges": [[32, 246]], "word_ranges": [[3, 30]], "text": "This is familial hypocalciuric hypercalcemia or benign familial hypercalcemia. Autosomal dominant disorder. Asymptomatic. Not cured after surgical treatment; it is due to mutation of the calcium-sensitive receptor."}, "4": {"exist": false, "char_ranges": [], "word_ranges": [], "text": ""}, "5": {"exist": false, "char_ranges": [], "word_ranges": [], "text": ""}}} {"id": 548, "year": 2022, "question_id_specific": 126, "full_question": "A 58-year-old male with a history of arterial hypertension of 6 years of evolution, who consults for poor control of blood pressure figures despite receiving treatment with an angiotensin-converting enzyme inhibitor, a diuretic and a calcium antagonist. On consultation she presented with blood pressure of 149/100 mmHg. Laboratory tests: creatinine 1.2 mg/dl, potassium 2.2 mEq/l and compensated metabolic alkalosis; the rest of the biochemical study, blood count, coagulation and urinary sediment were normal. Mark the correct statement:", "full_answer": "CT scan is part of the diagnostic workup in case there is biochemical confirmation.", "type": "CARDIOLOGY", "options": {"1": "The origin of hypertension in this case is excessive aldosterone secretion caused by autonomic hyperfunction of the adrenal medulla.", "2": "In most cases the anatomical substrate is bilateral hyperplasia of the adrenal cortex.", "3": "CT scan is part of the diagnostic study in case of biochemical confirmation.", "4": "Spironolactone is contraindicated in the management of this pathology.", "5": NaN}, "correct_option": 3, "explanations": {"1": {"exist": false, "char_ranges": [], "word_ranges": [], "text": ""}, "2": {"exist": false, "char_ranges": [], "word_ranges": [], "text": ""}, "3": {"exist": true, "char_ranges": [[0, 83]], "word_ranges": [[0, 14]], "text": "CT scan is part of the diagnostic workup in case there is biochemical confirmation."}, "4": {"exist": false, "char_ranges": [], "word_ranges": [], "text": ""}, "5": {"exist": false, "char_ranges": [], "word_ranges": [], "text": ""}}} {"id": 318, "year": 2016, "question_id_specific": 140, "full_question": "You evaluate a 66-year-old patient with groin pain accentuated by prolonged standing for a few days per month. A plain radiograph of the hips shows narrowing of the femoroacetabular joint space, sclerosis and ostephytes.", "full_answer": "It is a coxarthrosis. The surgical criteria depends on the patient's symptoms and failure of medical treatment. No medication slows down or modifies the course of the disease. An MRI is not necessary, it does not contribute anything. Treatment is medical and sequential, starting with the first analgesic step of the WHO.", "type": "RHEUMATOLOGY", "options": {"1": "I make the diagnosis of coxarthrosis and send to the traumatologist to place a hip prosthesis.", "2": "Start treatment with weak opioids that have shown evidence in stopping the progression of the disease.", "3": "Treatment with paracetamol, explaining that the evolution is very variable and the surgical indication depends on the functionality and pain control.", "4": "Because of the radiological features described, I need a hip MRI before making a therapeutic decision.", "5": NaN}, "correct_option": 3, "explanations": {"1": {"exist": false, "char_ranges": [], "word_ranges": [], "text": ""}, "2": {"exist": true, "char_ranges": [[112, 175]], "word_ranges": [[17, 28]], "text": "No medication slows down or modifies the course of the disease."}, "3": {"exist": true, "char_ranges": [[234, 321]], "word_ranges": [[38, 52]], "text": "Treatment is medical and sequential, starting with the first analgesic step of the WHO."}, "4": {"exist": true, "char_ranges": [[176, 233]], "word_ranges": [[28, 38]], "text": "An MRI is not necessary, it does not contribute anything."}, "5": {"exist": false, "char_ranges": [], "word_ranges": [], "text": ""}}} {"id": 147, "year": 2012, "question_id_specific": 157, "full_question": "A 19-year-old female university student comes to the clinic accompanied by her parents, reporting feeling progressively more asthenic in the last two months, with loss of appetite and weight and with greater difficulty in concentrating on her studies. The anamnesis also highlights that she has lost interest in going out with friends, has ideas of death without self-harming ideation and pessimistic cognitions of the future. Her weight is 90% of that considered ideal for age and gender. She does not present weight phobia or body image distortion. The most appropriate diagnosis is:", "full_answer": "As soon as we hear about weight we go to anorexia nervosa, but anorexia nervosa has three features: significant self-induced weight loss, body image distortion and amenorrhea. So we remove 1. One loses weight when one gets sad, worries, etc... 3 does not make sense in the MIR because of the clinic -in reality the history is different and in psychiatry everything is evolution-. 4 has to meet temporal criteria that are not met here. So we are between 2 and 5. Depression or anxiety? There is no anxiety clinic and she also meets the criteria for depression. The poor girl has been diagnosed with depression. The answer is 5 (another option is that she was studying the MIR).", "type": "PSYCHIATRY", "options": {"1": "Anorexia nervosa.", "2": "Anxiety disorder.", "3": "Borderline personality disorder.", "4": "Dysthymia.", "5": "Major depressive disorder."}, "correct_option": 5, "explanations": {"1": {"exist": true, "char_ranges": [[100, 191]], "word_ranges": [[18, 31]], "text": "significant self-induced weight loss, body image distortion and amenorrhea. So we remove 1."}, "2": {"exist": true, "char_ranges": [[485, 559]], "word_ranges": [[85, 98]], "text": "There is no anxiety clinic and she also meets the criteria for depression."}, "3": {"exist": true, "char_ranges": [[244, 379]], "word_ranges": [[40, 64]], "text": "3 does not make sense in the MIR because of the clinic -in reality the history is different and in psychiatry everything is evolution-."}, "4": {"exist": true, "char_ranges": [[380, 434]], "word_ranges": [[64, 75]], "text": "4 has to meet temporal criteria that are not met here."}, "5": {"exist": true, "char_ranges": [[485, 559]], "word_ranges": [[85, 98]], "text": "There is no anxiety clinic and she also meets the criteria for depression."}}} {"id": 530, "year": 2021, "question_id_specific": 167, "full_question": "In a patient with known advanced chronic kidney disease (CKD G4, eGFR 20 ml/min) who comes to the emergency department for general weakness and is found to have severe hyperkalemia (K 7 mEq/l) with electrocardiographic abnormalities. What would be the first measure to take?", "full_answer": "We describe the case of a patient with a hyperkalemic emergency (potassium >6.5 mmol/l with ECG alterations). The first step is the administration of iv calcium gluconate to counteract the cardiac toxicity of hyperkalemia (option 3 correct), giving us more time to initiate hypokalemic treatments (initially saline glucose with insulin, salbutamol, furosemide, and assessing dialysis if there is no good response).", "type": "NEPHROLOGY", "options": {"1": "Administration of cation exchange resins.", "2": "Placement of a catheter to initiate dialysis.", "3": "Administration of intravenous calcium gluconate.", "4": "Administration of oral furosemide.", "5": NaN}, "correct_option": 3, "explanations": {"1": {"exist": false, "char_ranges": [], "word_ranges": [], "text": ""}, "2": {"exist": false, "char_ranges": [], "word_ranges": [], "text": ""}, "3": {"exist": true, "char_ranges": [[0, 241]], "word_ranges": [[0, 37]], "text": "We describe the case of a patient with a hyperkalemic emergency (potassium >6.5 mmol/l with ECG alterations). The first step is the administration of iv calcium gluconate to counteract the cardiac toxicity of hyperkalemia (option 3 correct),"}, "4": {"exist": false, "char_ranges": [], "word_ranges": [], "text": ""}, "5": {"exist": false, "char_ranges": [], "word_ranges": [], "text": ""}}} {"id": 494, "year": 2020, "question_id_specific": 157, "full_question": "A 26-year-old woman consults for a feeling of generalized weakness that has progressively set in over the course of three weeks, becoming particularly intense in the last two days. For the last couple of years, she reports episodes of joint pain in the hands that have required the use of anti-inflammatory drugs, as well as the appearance of an erythematous lesion of unclear cause in the neckline area, mainly in summer. In the physical examination there was only an evident cutaneous-mucosal pallor and a heart rate of 100 bpm. The hemogram shows: Hb 6 gr/dL, Hto 27 %, MCV 105 fL, 3,420 leukocytes/mm3 (2300 neutrophils/mm3, 800 lymphocytes/mm3, 250 monocytes/mm3, 50 eosinophils/mm3, 20 basophils/mm3), platelets 170,000/mm3. Biochemistry: AST 30 IU/L, ALT 35 IU/L, GGT 59 IU/L, alkaline phosphatase 105 IU/L, LDH 490 IU/L, urea 20 mg/dL, creatinine 0.8 mg/dL. Taking into account the available information, indicate which of the following additional analytical parameters you would need to know in order to make the most appropriate immediate decision:", "full_answer": "They comment on the case of a young woman with symptoms of systemic disease of probable autoimmune origin (lupus) who presents with macrocytic anemia, options 2 and 4 are options to identify the autoimmune disease but what they ask us is what to do to treat the anemia, so we should suspect that the anemia is secondary to the autoimmune disease and therefore request a direct Coombs test, since if it is positive the patient, except in extreme cases, will not be transfused and the primary cause should be treated. It also speaks of rapid onset, which in the case of vitamin B12 is more progressive.", "type": "HEMATOLOGY", "options": {"1": "Vitamin B12.", "2": "Antinuclear antibodies.", "3": "Direct Coombs' test.", "4": "Anti-DNA antibodies.", "5": NaN}, "correct_option": 3, "explanations": {"1": {"exist": false, "char_ranges": [], "word_ranges": [], "text": ""}, "2": {"exist": true, "char_ranges": [[0, 269]], "word_ranges": [[0, 48]], "text": "They comment on the case of a young woman with symptoms of systemic disease of probable autoimmune origin (lupus) who presents with macrocytic anemia, options 2 and 4 are options to identify the autoimmune disease but what they ask us is what to do to treat the anemia,"}, "3": {"exist": true, "char_ranges": [[273, 515]], "word_ranges": [[49, 90]], "text": "we should suspect that the anemia is secondary to the autoimmune disease and therefore request a direct Coombs test, since if it is positive the patient, except in extreme cases, will not be transfused and the primary cause should be treated."}, "4": {"exist": true, "char_ranges": [[0, 269]], "word_ranges": [[0, 48]], "text": "They comment on the case of a young woman with symptoms of systemic disease of probable autoimmune origin (lupus) who presents with macrocytic anemia, options 2 and 4 are options to identify the autoimmune disease but what they ask us is what to do to treat the anemia,"}, "5": {"exist": false, "char_ranges": [], "word_ranges": [], "text": ""}}} {"id": 117, "year": 2012, "question_id_specific": 112, "full_question": "A 64-year-old male presents with fever, cough, dyspnea and right pleuritic pain of 1 week's evolution. Chest X-ray shows a loculated right pleural effusion occupying two thirds of the hemithorax. During thoracentesis, a yellowish fluid is drawn and analysis shows: leukocytes 15,000/uL, 92% neutrophils, glucose 30 mg/dL, pH 7, lactate dehydrogenase 3500 U/L, adenosine deaminase 45 U/L and absence of germs on GRAM stain. What is the most appropriate next course of action in this patient?", "full_answer": "In this question there should not be much doubt either (by the way, I think it is more of Pneumology than mine, but well, with this fever my friend Emilienko has deviated), since the clinical picture points to a parapneumonic pleural effusion, the macroscopic characteristics of the fluid seem so and the analysis of it shows us that the effusion is also complicated, almost on the verge of being an empyema (very high LDH, cellularity with predominance of PMN, glucose consumption). In addition, with this pH (and knowing that the effusion is already loculated), the endothoracic tube is more than indicated.", "type": "INFECTOLOGY", "options": {"1": "Intravenous antibiotic therapy.", "2": "Intravenous antibiotic therapy and repeat diagnostic thoracentesis in 24 hours.", "3": "Intravenous antibiotic therapy and repeat diagnostic thoracentesis if there is no improvement in 48 hours.", "4": "Intravenous antibiotic therapy and evacuative (therapeutic) thoracentesis if any germ is isolated in the pleural fluid culture.", "5": "Intravenous antibiotic therapy and placement of a chest tube or catheter to drain all pleural fluid."}, "correct_option": 5, "explanations": {"1": {"exist": false, "char_ranges": [], "word_ranges": [], "text": ""}, "2": {"exist": false, "char_ranges": [], "word_ranges": [], "text": ""}, "3": {"exist": false, "char_ranges": [], "word_ranges": [], "text": ""}, "4": {"exist": false, "char_ranges": [], "word_ranges": [], "text": ""}, "5": {"exist": true, "char_ranges": [[179, 609]], "word_ranges": [[33, 101]], "text": "the clinical picture points to a parapneumonic pleural effusion, the macroscopic characteristics of the fluid seem so and the analysis of it shows us that the effusion is also complicated, almost on the verge of being an empyema (very high LDH, cellularity with predominance of PMN, glucose consumption). In addition, with this pH (and knowing that the effusion is already loculated), the endothoracic tube is more than indicated."}}} {"id": 27, "year": 2011, "question_id_specific": 203, "full_question": "A 52-year-old patient is admitted to the Hospital for severe pneumonia. With appropriate antibiotic treatment, the patient's respiratory symptoms improved. After 4 days of stay in the ward, his evolution is complicated by the appearance of a severe diarrhea. What is the most frequent microorganism responsible for this condition?", "full_answer": "In this case, given the antibiotic treatment given and the severity of the diarrhea, it seems that we are dealing with a Clostridium infection. Option 2 is not very credible, in fact, as much as patients with CAP may have associated diarrhea, it is unlikely that a bacterium is sensitive to an antibiotic at the beginning of the picture and stops being so after 3 days.", "type": "INFECTIOUS", "options": {"1": "Salmonella enterica.", "2": "The bacterium itself causing pneumonia that has become resistant to the antibiotic.", "3": "Campylobacter jejuni.", "4": "Clostridium difficile.", "5": "Yersinia enterocolitica."}, "correct_option": 4, "explanations": {"1": {"exist": false, "char_ranges": [], "word_ranges": [], "text": ""}, "2": {"exist": true, "char_ranges": [[144, 368]], "word_ranges": [[24, 66]], "text": "Option 2 is not very credible, in fact, as much as patients with CAP may have associated diarrhea, it is unlikely that a bacterium is sensitive to an antibiotic at the beginning of the picture and stops being so after 3 days."}, "3": {"exist": false, "char_ranges": [], "word_ranges": [], "text": ""}, "4": {"exist": true, "char_ranges": [[14, 142]], "word_ranges": [[3, 24]], "text": "given the antibiotic treatment given and the severity of the diarrhea, it seems that we are dealing with a Clostridium infection."}, "5": {"exist": false, "char_ranges": [], "word_ranges": [], "text": ""}}} {"id": 471, "year": 2020, "question_id_specific": 139, "full_question": "A 7-year-old boy brought to the emergency department after falling off a swing onto his right hand. He has no history of interest. He presents with fork dorsum deformity of the wrist and functional impotence, with normal distal neurovascular status. What lesion would you expect to find on the urgent x-ray you request?", "full_answer": "With the deformity and pain in the distal radius, we expect a fracture at this level, so we mark 3. However, I see it as unquestionable, given that it could also have a concomitant fracture in the ulnar green stalk, although the main lesion is that of the radius.", "type": "ORTHOPEDIC SURGERY AND TRAUMATOLOGY", "options": {"1": "Fracture - dislocation of Monteggia.", "2": "Radial head fracture.", "3": "Epiphysiolysis of the distal radius.", "4": "Fracture in green stem of ulna.", "5": NaN}, "correct_option": 3, "explanations": {"1": {"exist": false, "char_ranges": [], "word_ranges": [], "text": ""}, "2": {"exist": false, "char_ranges": [], "word_ranges": [], "text": ""}, "3": {"exist": true, "char_ranges": [[0, 99]], "word_ranges": [[0, 20]], "text": "With the deformity and pain in the distal radius, we expect a fracture at this level, so we mark 3."}, "4": {"exist": false, "char_ranges": [], "word_ranges": [], "text": ""}, "5": {"exist": false, "char_ranges": [], "word_ranges": [], "text": ""}}} {"id": 327, "year": 2016, "question_id_specific": 81, "full_question": "A 30-year-old woman comes to the clinic reporting anxiety, weight loss of about 6 kg and a feeling of \"nervousness\" in the last three months. In the physical examination she has tachycardia, hyperreflexia and absence of goiter. In the blood test TSH values are < 0.01 microU/mL, T4 is elevated and thyroglobulin levels are low. A scintigraphy shows an absence of uptake in the thyroid region. What seems to you the most likely diagnosis?", "full_answer": "Answer 2 incorrect: Graves' disease is characterized by hyperthyroidism and one or more of the following features: goiter, exophthalmos and pretibial myxedema. Answer 4 incorrect: The most obvious symptom of subacute thyroiditis is neck pain. Sometimes, the pain may extend to the jaw or ears. Painful enlargement of the thyroid gland can last for weeks or months. And you usually have low serum thyroid stimulating hormone (TSH). High serum free T4 level. High serum thyroglobulin level. Incorrect answer 4: Very rare tumor.", "type": "ENDOCRINOLOGY", "options": {"1": "Factitious thyrotoxicosis.", "2": "Hyperthyroidism due to Graves' disease.", "3": "Ovarian teratoma (ovarian stromal tumor).", "4": "Subacute thyroiditis.", "5": NaN}, "correct_option": 1, "explanations": {"1": {"exist": false, "char_ranges": [], "word_ranges": [], "text": ""}, "2": {"exist": true, "char_ranges": [[0, 159]], "word_ranges": [[0, 22]], "text": "Answer 2 incorrect: Graves' disease is characterized by hyperthyroidism and one or more of the following features: goiter, exophthalmos and pretibial myxedema."}, "3": {"exist": false, "char_ranges": [], "word_ranges": [], "text": ""}, "4": {"exist": true, "char_ranges": [[160, 293]], "word_ranges": [[22, 45]], "text": "Answer 4 incorrect: The most obvious symptom of subacute thyroiditis is neck pain. Sometimes, the pain may extend to the jaw or ears."}, "5": {"exist": false, "char_ranges": [], "word_ranges": [], "text": ""}}} {"id": 450, "year": 2018, "question_id_specific": 154, "full_question": "A man presents on neurological examination with a thermoalgesic sensory deficit in the left leg associated with a loss of vibratory and positional sensitivity in the right leg. At the same time he presents with clumsiness and loss of distal strength in the right leg and a right plantar cutaneous reflex in extension. Which of the following statements is correct:", "full_answer": "They describe a typical hemisection or Brown-Sequard picture. Ipsilateral posterior motor and chordal involvement with contralateral pain and temperature involvement.", "type": "NEUROSURGERY", "options": {"1": "It is a centromedullary syndrome type syringomyelia.", "2": "It is a hemimedullary syndrome.", "3": "It is a pattern of transverse medullary lesion.", "4": "It is a pattern of lateral bulbar lesion.", "5": NaN}, "correct_option": 2, "explanations": {"1": {"exist": false, "char_ranges": [], "word_ranges": [], "text": ""}, "2": {"exist": true, "char_ranges": [[0, 166]], "word_ranges": [[0, 20]], "text": "They describe a typical hemisection or Brown-Sequard picture. Ipsilateral posterior motor and chordal involvement with contralateral pain and temperature involvement."}, "3": {"exist": false, "char_ranges": [], "word_ranges": [], "text": ""}, "4": {"exist": false, "char_ranges": [], "word_ranges": [], "text": ""}, "5": {"exist": false, "char_ranges": [], "word_ranges": [], "text": ""}}} {"id": 265, "year": 2014, "question_id_specific": 137, "full_question": "65-year-old man with a history of pancreatic neoplasm undergoing chemotherapy. What diagnostic test is more cost-effective to confirm the diagnostic suspicion?", "full_answer": "Dopler ultrasound is the most accurate test and allows subsequent non-invasive monitoring of the evolution of the disease.", "type": "PNEUMOLOGY", "options": {"1": "D-dimer.", "2": "Magnetic resonance imaging.", "3": "Phlebography.", "4": "Venous Doppler ultrasound.", "5": "Helical CT."}, "correct_option": 4, "explanations": {"1": {"exist": false, "char_ranges": [], "word_ranges": [], "text": ""}, "2": {"exist": false, "char_ranges": [], "word_ranges": [], "text": ""}, "3": {"exist": false, "char_ranges": [], "word_ranges": [], "text": ""}, "4": {"exist": true, "char_ranges": [[0, 122]], "word_ranges": [[0, 18]], "text": "Dopler ultrasound is the most accurate test and allows subsequent non-invasive monitoring of the evolution of the disease."}, "5": {"exist": false, "char_ranges": [], "word_ranges": [], "text": ""}}} {"id": 189, "year": 2013, "question_id_specific": 71, "full_question": "A 10-year-old boy with brief episodes of distractions (< 1 minute) in which he does not respond to calls and blinks. An EEG shows spike-wave discharges at 3 cycles per second. First-line elective treatment would with:", "full_answer": "The seizures described and the EEG correspond to typical absence seizures and the initial treatment of choice is valproic acid.", "type": "PEDIATRICS", "options": {"1": "Valproate.", "2": "Carbamazepine.", "3": "Phenytoin.", "4": "Gabapentin.", "5": "Clonazepam."}, "correct_option": 1, "explanations": {"1": {"exist": true, "char_ranges": [[0, 127]], "word_ranges": [[0, 20]], "text": "The seizures described and the EEG correspond to typical absence seizures and the initial treatment of choice is valproic acid."}, "2": {"exist": false, "char_ranges": [], "word_ranges": [], "text": ""}, "3": {"exist": false, "char_ranges": [], "word_ranges": [], "text": ""}, "4": {"exist": false, "char_ranges": [], "word_ranges": [], "text": ""}, "5": {"exist": false, "char_ranges": [], "word_ranges": [], "text": ""}}} {"id": 508, "year": 2021, "question_id_specific": 148, "full_question": "A 72-year-old woman with no past history of interest. After a complicated renal colic, a 2 cm cystic lesion in the tail of the pancreas, together with multiple bilateral renal cystic lesions, is found by chance in the abdominal CT scan. Endoscopic ultrasonography shows a polycystic lesion formed by multiple vesicles with central calcification in the tail of the pancreas with no connection to the duct of Wirsung. Fluid analysis is compatible with a serous cystadenoma. Of the following, which is the most correct attitude regarding the management of this patient:", "full_answer": "Serous cystoadenoma of the pancreas is a benign, often polycystic entity (also called microcystic adenoma) formed by collagen-producing (non-mucinous) cells. It is usually asymptomatic and is discovered as an incidental finding. Given its benign nature, surgery is not recommended, and is reserved only for symptomatic patients or those in whom the nature of the lesion is in doubt after a complete study with CT, MRI and echoendoscopy with biopsy. It is frequently sporadic, but can be associated with Von-Hippel-Lindau syndrome.", "type": "DIGESTIVE", "options": {"1": "Surgical resection (corporocaudal pancreatectomy).", "2": "Endoscopic ultrasonography-guided puncture and ethanolization of the puncture.", "3": "Follow-up of the lesion by MRI.", "4": "Kidney-pancreas transplant.", "5": NaN}, "correct_option": 3, "explanations": {"1": {"exist": true, "char_ranges": [[229, 281]], "word_ranges": [[31, 39]], "text": "Given its benign nature, surgery is not recommended,"}, "2": {"exist": true, "char_ranges": [[229, 281]], "word_ranges": [[31, 39]], "text": "Given its benign nature, surgery is not recommended,"}, "3": {"exist": true, "char_ranges": [[229, 281]], "word_ranges": [[31, 39]], "text": "Given its benign nature, surgery is not recommended,"}, "4": {"exist": true, "char_ranges": [[229, 281]], "word_ranges": [[31, 39]], "text": "Given its benign nature, surgery is not recommended,"}, "5": {"exist": false, "char_ranges": [], "word_ranges": [], "text": ""}}} {"id": 151, "year": 2012, "question_id_specific": 70, "full_question": "A 25-year-old female patient with a history of skin rash after sun exposure and occasional polyarthritis in the joints of the hands, controlled with non-steroidal anti-inflammatory drugs, presents with general malaise, progressive feeling of generalized weakness and pallor for the last 15 days. Laboratory tests showed hemoglobin 7 g/dL, MCV 108 mm/h, 150,000 platelets/mm3, 3000 leukocytes/mm3, elevated LDH, undetectable haptoglobin. In the case of choosing a single diagnostic test indicate which of the following determinations should be performed first:", "full_answer": "The clinical and biological picture of this patient raises suspicion of hemolytic anemia in the context of systemic lupus erythematosus. If we have to choose only one diagnostic test, and this should be the first, we are interested in confirming the presence of an autoimmune hemolytic anemia with the direct Coombs' test in order to initiate treatment. The other tests can wait...", "type": "RHEUMATOLOGY", "options": {"1": "Direct Coombs test.", "2": "Antinuclear antibodies (ANA).", "3": "Vitamin B12.", "4": "Ferritin.", "5": "Folic acid."}, "correct_option": 1, "explanations": {"1": {"exist": true, "char_ranges": [[0, 353]], "word_ranges": [[0, 57]], "text": "The clinical and biological picture of this patient raises suspicion of hemolytic anemia in the context of systemic lupus erythematosus. If we have to choose only one diagnostic test, and this should be the first, we are interested in confirming the presence of an autoimmune hemolytic anemia with the direct Coombs' test in order to initiate treatment."}, "2": {"exist": true, "char_ranges": [[354, 381]], "word_ranges": [[57, 62]], "text": "The other tests can wait..."}, "3": {"exist": true, "char_ranges": [[354, 381]], "word_ranges": [[57, 62]], "text": "The other tests can wait..."}, "4": {"exist": true, "char_ranges": [[354, 381]], "word_ranges": [[57, 62]], "text": "The other tests can wait..."}, "5": {"exist": true, "char_ranges": [[354, 381]], "word_ranges": [[57, 62]], "text": "The other tests can wait..."}}} {"id": 404, "year": 2016, "question_id_specific": 139, "full_question": "A 70-year-old woman with a history of anorexia, weight loss, discomfort in the musculature and proximal joints plus pain in the temporomandibular region who comes to the emergency department for unilateral loss of vision (hand movement), sudden and painless onset (afferent pupillary defect). What test would you request first for diagnostic purposes?", "full_answer": "This is a woman with abrupt, painless and severe loss (she only sees the movement of the hands) of vision in one eye. She does not give us much information from the ocular examination, only that there is an afferent pupillary defect. We have to rely on systemic symptoms to diagnose the problem. This is an elderly woman with typical symptoms of polymyalgia rheumatica, and the pain in the temporomandibular region is very suggestive. It is giant cell arteritis or temporal arteritis. The most common ocular manifestation is anterior ischemic optic neuropathy. In the fundus we would certainly see a papillary edema of that eye, with a gypsum white coloration. The afferent pupillary defect is very characteristic of these severe ischemic optic neuropathies. Suspecting giant cell arteritis, there are three analytical markers that are characteristically high: CRP, ESR and platelet count. We will first request one of these parameters and treat it often without waiting for biopsy confirmation. Therefore, option 2 is the correct one.", "type": "OPHTHALMOLOGY (ECTOPIC)", "options": {"1": "Lumbar puncture.", "2": "C-reactive protein.", "3": "Magnetic Resonance Angiography.", "4": "Carotid ultrasound.", "5": NaN}, "correct_option": 2, "explanations": {"1": {"exist": false, "char_ranges": [], "word_ranges": [], "text": ""}, "2": {"exist": true, "char_ranges": [[759, 889]], "word_ranges": [[123, 141]], "text": "Suspecting giant cell arteritis, there are three analytical markers that are characteristically high: CRP, ESR and platelet count."}, "3": {"exist": false, "char_ranges": [], "word_ranges": [], "text": ""}, "4": {"exist": false, "char_ranges": [], "word_ranges": [], "text": ""}, "5": {"exist": false, "char_ranges": [], "word_ranges": [], "text": ""}}} {"id": 215, "year": 2014, "question_id_specific": 106, "full_question": "A 68-year-old patient consults for edema and asthenia. Laboratory tests showed creatinine 5 mg/dL, hemoglobin 10 g/dL and marked hypogammaglobulinemia in serum at the expense of IgG, IgA and IgM. A urinalysis reveals the presence of kappa light chains. What is your diagnostic suspicion?", "full_answer": "This question is mixed hemato-nephro. It is complicated, so we are going to rule out options one by one. Option 2, nephrotic syndrome, apart from being broad and nonspecific, would not be assessable, since they do not give us the quantification of urine protein (they only tell us that urinalysis reveals the presence of kappa light chains, not the amount). Option 3 would be unlikely; in amyloidosis, the light chains that are usually deposited are of the lambda type. Option 4 also seems unlikely, since in IgA myeloma we would expect a monoclonal peak of this Ig, which in this case does not appear. The doubt that arises is between 1 and 5; I would opt for 1, because this entity fulfills the characteristics described (absence of monoclonal peak in most cases, acute renal failure and presence of kappa chains in urine), being light chain myeloma a somewhat broader entity, but option 5 could also be considered valid.", "type": "NEPHROLOGY", "options": {"1": "Kappa light chain deposition disease.", "2": "Nephrotic syndrome.", "3": "Amyloidosis.", "4": "IgA myeloma with Bence-Jones proteinuria.", "5": "Light chain myeloma."}, "correct_option": 1, "explanations": {"1": {"exist": true, "char_ranges": [[645, 825]], "word_ranges": [[113, 142]], "text": "I would opt for 1, because this entity fulfills the characteristics described (absence of monoclonal peak in most cases, acute renal failure and presence of kappa chains in urine),"}, "2": {"exist": true, "char_ranges": [[105, 357]], "word_ranges": [[19, 60]], "text": "Option 2, nephrotic syndrome, apart from being broad and nonspecific, would not be assessable, since they do not give us the quantification of urine protein (they only tell us that urinalysis reveals the presence of kappa light chains, not the amount)."}, "3": {"exist": true, "char_ranges": [[358, 469]], "word_ranges": [[60, 79]], "text": "Option 3 would be unlikely; in amyloidosis, the light chains that are usually deposited are of the lambda type."}, "4": {"exist": true, "char_ranges": [[470, 602]], "word_ranges": [[79, 104]], "text": "Option 4 also seems unlikely, since in IgA myeloma we would expect a monoclonal peak of this Ig, which in this case does not appear."}, "5": {"exist": true, "char_ranges": [[826, 878]], "word_ranges": [[142, 150]], "text": "being light chain myeloma a somewhat broader entity,"}}} {"id": 264, "year": 2014, "question_id_specific": 135, "full_question": "A 24-year-old woman is found lying in the street by passers-by. On arrival of the emergency team she was found with an oxygen saturation of 88% breathing room air and on examination with pinpoint pupils. She was transferred to the emergency room of the nearest hospital, where the baseline arterial blood gases showed: pH 7.25, PaC02 60 mmHg, Pa02 58 mmHg, bicarbonate 26 mEq/1 and base excess of -1. In blood sodium was 137 mEq/1 and chloride 100 mEq/1:", "full_answer": "The acidosis with elevated PCO2, hypoxemia and normal bicarbonate suggests an acute respiratory acidosis of rapid onset probably due to drug intoxication.", "type": "PNEUMOLOGY", "options": {"1": "Partial respiratory failure.", "2": "Metabolic acidosis.", "3": "Pure respiratory acidosis.", "4": "Respiratory alkalosis due to lack of chloride.", "5": "Blood gases can only be venous blood."}, "correct_option": 3, "explanations": {"1": {"exist": false, "char_ranges": [], "word_ranges": [], "text": ""}, "2": {"exist": false, "char_ranges": [], "word_ranges": [], "text": ""}, "3": {"exist": true, "char_ranges": [[0, 154]], "word_ranges": [[0, 22]], "text": "The acidosis with elevated PCO2, hypoxemia and normal bicarbonate suggests an acute respiratory acidosis of rapid onset probably due to drug intoxication."}, "4": {"exist": false, "char_ranges": [], "word_ranges": [], "text": ""}, "5": {"exist": false, "char_ranges": [], "word_ranges": [], "text": ""}}} {"id": 594, "year": 2022, "question_id_specific": 80, "full_question": "A 2-month-old infant with an upper respiratory tract cold of 3 days of evolution, who begins with moderate respiratory distress and pulmonary auscultation with expiratory wheezing. Oxygen saturation is 89 %. Respiratory syncytial virus is isolated in the nasopharyngeal exudate. Which of the following treatments do you consider more indicated for this condition?", "full_answer": "We are presented with a case of RSV-positive bronchiolitis. According to current recommendations, it would be indicated to initiate supplemental oxygen therapy.", "type": "PEDIATRICS", "options": {"1": "Oral rivabirin.", "2": "Nebulized salbutamol.", "3": "Supplemental oxygen.", "4": "Intravenous corticosteroids.", "5": NaN}, "correct_option": 3, "explanations": {"1": {"exist": false, "char_ranges": [], "word_ranges": [], "text": ""}, "2": {"exist": false, "char_ranges": [], "word_ranges": [], "text": ""}, "3": {"exist": true, "char_ranges": [[0, 160]], "word_ranges": [[0, 22]], "text": "We are presented with a case of RSV-positive bronchiolitis. According to current recommendations, it would be indicated to initiate supplemental oxygen therapy."}, "4": {"exist": false, "char_ranges": [], "word_ranges": [], "text": ""}, "5": {"exist": false, "char_ranges": [], "word_ranges": [], "text": ""}}} {"id": 236, "year": 2014, "question_id_specific": 108, "full_question": "A 67-year-old female patient on ticlopidine treatment comes to the emergency department with headache, asthenia and petechiae in the lower extremities. Laboratory tests showed hemoglobin 8.2 g/dL, MCV 100 fl, platelets 25000/ul and leukocytes 7500/ul with normal formula. The reticulocyte count was elevated and the blood smear showed numerous schistocytes. Coagulation studies (APTT, PT and fibrinogen) are normal. The biochemistry shows LDH 2700 IU/l and bilirubin 2.6 mg/dl. What is the most likely diagnosis?", "full_answer": "Typical of ticlopidine. Not only thrombocytopenia but also headache, schistocytes, high LDH and high bilirubin.", "type": "HEMATOLOGY", "options": {"1": "Autoimmune thrombocytopenic purpura.", "2": "Thrombotic thrombocytopenic purpura.", "3": "Bone marrow aplasia.", "4": "Drug-induced thrombopenia.", "5": "Disseminated intravascular coagulation."}, "correct_option": 2, "explanations": {"1": {"exist": false, "char_ranges": [], "word_ranges": [], "text": ""}, "2": {"exist": true, "char_ranges": [[0, 111]], "word_ranges": [[0, 15]], "text": "Typical of ticlopidine. Not only thrombocytopenia but also headache, schistocytes, high LDH and high bilirubin."}, "3": {"exist": false, "char_ranges": [], "word_ranges": [], "text": ""}, "4": {"exist": false, "char_ranges": [], "word_ranges": [], "text": ""}, "5": {"exist": false, "char_ranges": [], "word_ranges": [], "text": ""}}}